Med Surg Final Exam

¡Supera tus tareas y exámenes ahora con Quizwiz!

10. What should the nurse teach the patient before fluorescein angiography? a. Hold a card and fixate on the center dot. b. Report any burning or pain at the IV site. c. Remain still while the cornea is anesthetized. d. Let the examiner know when images shown appear clear.

ANS: B

18. Which instruction should the nurse include in a teaching plan for a patient with herpes simplex keratitis? a. Wash hands frequently and avoid touching the eyes. b. Apply antibiotic drops to the eye several times daily. c. Apply a new occlusive dressing to the affected eye at bedtime. d. Use corticosteroid ophthalmic ointment to decrease inflammation.

ANS: A

21. An unresponsive patient who has type 2 diabetes is brought to the emergency department and diagnosed with hyperosmolar hyperglycemia syndrome (HHS). What should the nurse anticipate doing? a. Giving 50% dextrose b. Inserting an IV catheter c. Initiating O2 by nasal cannula d. Administering glargine (Lantus) insulin

ANS: B

12. A patient receives aspart (NovoLog) insulin at 8:00 AM. At which time would the nurse anticipate the highest risk for hypoglycemia? a. 10:00 AM b. 12:00 AM c. 2:00 PM d. 4:00 PM

ANS: A

10. How should the nurse document a loud humming sound auscultated over the patient's abdominal aorta? a. Thrill b. Bruit c. Murmur d. Normal finding

ANS: B

12. Which blood pressure (BP) finding by the nurse indicates that no changes in therapy are needed for a 48-yr-old patient with newly diagnosed hypertension? a. 98/56 mm Hg b. 128/76 mm Hg c. 128/92 mm Hg d. 142/78 mm Hg

ANS: B

1. Which assessment finding should alert the nurse to ask the patient about alcohol use? a. Low blood pressure b. Decreased heart rate c. Elevated temperature d. Abdominal tenderness

ANS: D Abdominal pain associated with gastrointestinal tract and liver dysfunction is common in patients with chronic alcohol use. The other problems are not associated with alcohol use.

10. A patient with atopic dermatitis has been using a high-potency topical corticosteroid ointment for several weeks. The nurse should assess for which adverse effect? a. Thinning of the affected skin b. Alopecia of the affected area c. Dryness and scaling in the area d. Reddish-brown skin discoloration

ANS: A

11. The nurse has received the laboratory results for a patient who developed chest pain 4 hours ago and may be having a myocardial infarction. Which laboratory test result should be most helpful in indicating myocardial damage? a. Troponins b. Myoglobin c. Homocysteine (Hcy) d. Creatine kinase-MB (CK-MB)

ANS: A

11. What is the priority problem for a patient experiencing an acute attack with Meniere's disease? a. Being at risk for falls b. Imbalanced nutritional intake c. Difficulty performing self-care d. Impaired verbal communication

ANS: A

13. The nurse hears a murmur between the S1 and S2 heart sounds at the patient's left fifth intercostal space and midclavicular line. How will the nurse record this information? a. Systolic murmur heard at mitral area. b. Systolic murmur heard at Erb's point. c. Diastolic murmur heard at aortic area. d. Diastolic murmur heard at the point of maximal impulse.

ANS: A

13. Which patient action indicates an accurate understanding of the nurse's teaching about the use of an insulin pump? a. The patient programs the pump for an insulin bolus after eating. b. The patient changes the location of the insertion site every week. c. The patient takes the pump off at bedtime and starts it again each morning. d. The patient plans a diet with more calories than usual when using the pump.

ANS: A

13. The nurse working in the clinic receives telephone calls from several patients who want appointments as soon as possible. Which patient should be seen first? a. 71-yr-old who has noticed increasing loss of peripheral vision b. 74-yr-old who has difficulty seeing well enough to drive at night c. 60-yr-old who is reporting dry eyes with decreased tear formation d. 64-yr-old who states that it is becoming difficult to read news print

ANS: A

1. A young adult patient who is being seen in the clinic has excessive secretion of the anterior pituitary hormones. Which laboratory test result should the nurse expect? a. Increased urinary cortisol b. Decreased serum thyroxine c. Elevated serum aldosterone d. Low urinary catecholamines

ANS: A

14. A patient with diabetes is starting on intensive insulin therapy. Which type of insulin will the nurse discuss using for mealtime coverage? a. Lispro (Humalog) b. Glargine (Lantus) c. Detemir (Levemir) d. NPH (Humulin N)

ANS: A

14. The nurse on the intermediate care unit received change-of-shift report on four patients with hypertension. Which patient should the nurse assess first? a. 48-yr-old with a blood pressure of 160/92 mm Hg who reports chest pain b. 52-yr-old with a blood pressure of 198/90 mm Hg who has intermittent claudication c. 50-yr-old with a blood pressure of 190/104 mm Hg who has a creatinine of 1.7 mg/dL d. 43-yr-old with a blood pressure of 172/98 mm Hg whose urine shows microalbuminuria.

ANS: A

15. The nurse is reviewing the laboratory test results for a patient who has recently been diagnosed with hypertension. Which result is most important to communicate to the health care provider? a. Serum creatinine of 2.8 mg/dL b. Serum potassium of 4.5 mEq/L c. Serum hemoglobin of 14.7 g/dL d. Blood glucose level of 96 mg/dL

ANS: A

18. In preparation for discharge, the nurse teaches a patient with chronic stable angina how to use the prescribed short-acting and long-acting nitrates. Which patient statement indicates that the teaching has been effective? a. "I will sit down before I put the nitroglycerin under my tongue." b. "I will check my pulse rate before I take any nitroglycerin tablets." c. "I will put the nitroglycerin patch on as soon as I get any chest pain." d. "I will remove the nitroglycerin patch before taking sublingual nitroglycerin."

ANS: A

18. The standard policy on the cardiac unit states, "Notify the health care provider for mean arterial pressure (MAP) less than 70 mm Hg." Which patient should the nurse call the health care provider about? a. Postoperative patient with a BP of 116/42 mm Hg. b. Newly admitted patient with a BP of 150/87 mm Hg. c. Patient with left ventricular failure who has a BP of 110/70 mm Hg. d. Patient with a myocardial infarction who has a BP of 140/86 mm Hg.

ANS: A

2. Which information should the nurse include when teaching a patient who has just received a prescription for ciprofloxacin (Cipro) to treat a urinary tract infection? a. Use a sunscreen with a high SPF when exposed to the sun. b. Sun exposure may decrease the effectiveness of the medication. c. Photosensitivity may result in an artificial-looking tan appearance. d. Wear sunglasses to avoid eye damage while taking this medication.

ANS: A

20. The nurse is preparing to teach a 43-yr-old man who is newly diagnosed with type 2 diabetes about home management of the disease. Which action should the nurse take first? a. Assess the patient's perception of what it means to have diabetes. b. Ask the patient's family to participate in the diabetes education program. c. Demonstrate how to check glucose using capillary blood glucose monitoring. d. Discuss the need for the patient to actively participate in diabetes management.

ANS: A

21. After having a myocardial infarction (MI) and successful percutaneous coronary intervention, the patient states, "It was just a little chest pain. As soon as I get out of here, I'm going for my vacation as planned." Which reply would be most appropriate for the nurse to make? a. "What do you think caused your chest pain?" b. "Where are you planning to go for your vacation?" c. "Sometimes plans need to change after a heart attack." d. "Recovery from a heart attack takes at least a few weeks."

ANS: A

22. The nurse is reviewing the laboratory results for newly admitted patients on the cardiovascular unit. Which laboratory result is most important to communicate rapidly to the health care provider? a. High troponin I level b. Increased triglyceride level c. Very low homocysteine level d. Elevated C-reactive protein level

ANS: A

22. The nurse working in the dermatology clinic assesses a young adult female patient who has severe cystic acne. Which assessment finding is of concern related to the patient's prescribed isotretinoin? a. The patient recently had an intrauterine device removed. b. The patient already has some acne scarring on her forehead. c. The patient has also used topical antibiotics to treat the acne. d. The patient has a strong family history of rheumatoid arthritis.

ANS: A

22. Which action should the nurse take first to assist a patient with newly diagnosed stage 1 hypertension in making needed dietary changes? a. Collect a detailed diet history. b. Provide a list of low-sodium foods. c. Help the patient make an appointment with a dietitian. d. Teach the patient about foods that are high in potassium.

ANS: A

25. Which question during the assessment of a patient who has diabetes will help the nurse identify autonomic neuropathy? a. "Do you feel bloated after eating?" b. "Have you seen any skin changes?" c. "Do you need to increase your insulin dosage when you are stressed?" d. "Have you noticed any painful new ulcerations or sores on your feet?"

ANS: A

26. A patient who has recently started taking pravastatin (Pravachol) and niacin reports several symptoms to the nurse. Which information is most important to communicate to the health care provider? a. Generalized muscle aches and pains b. Dizziness with rapid position changes c. Nausea when taking the drugs before meals d. Flushing and pruritus after taking the drugs

ANS: A

26. Which information will the nurse include in teaching a female patient who has peripheral arterial disease, type 2 diabetes, and sensory neuropathy of the feet and legs? a. Choose flat-soled leather shoes. b. Set heating pads on a low temperature. c. Use a callus remover for corns or calluses. d. Soak feet in warm water for an hour each day.

ANS: A

27. A patient who is being admitted to the emergency department with intermittent chest pain gives the following list of daily medications to the nurse. Which medication has the most immediate implications for the patient's care? a. Tadalafil (Cialas) b. Furosemide (Lasix) c. Warfarin (Coumadin) d. Diltiazem (Cardizem)

ANS: A

33. When admitting a patient with a non-ST-segment-elevation myocardial infarction (NSTEMI) to the intensive care unit, which action should the nurse perform first? a. Attach the heart monitor. b. Obtain the blood pressure. c. Assess the peripheral pulses. d. Auscultate the breath sounds.

ANS: A

14. What should the nurse assess to evaluate the effectiveness of treatment for the patient's myopia and presbyopia? a. Strength of the eye muscles. b. Both near and distant vision. c. Cloudiness in the eye lenses. d. Intraocular pressure changes.

ANS: B

34. A 27-yr-old patient admitted with diabetic ketoacidosis (DKA) has a serum glucose level of 732 mg/dL and serum potassium level of 3.1 mEq/L. Which action prescribed by the health care provider should the nurse take first? a. Place the patient on a cardiac monitor. b. Administer IV potassium supplements. c. Ask the patient about home insulin doses. d. Start an insulin infusion at 0.1 units/kg/hr.

ANS: A

35. A patient with diabetic ketoacidosis is brought to the emergency department. Which prescribed action should the nurse implement first? a. Infuse 1 L of normal saline per hour. b. Give sodium bicarbonate 50 mEq IV push. c. Administer regular insulin 10 U by IV push. d. Start a regular insulin infusion at 0.1 units/kg/hr.

ANS: A

36. A patient had a non-ST-segment-elevation myocardial infarction (NSTEMI) 3 days ago. Which nursing intervention is appropriate for the registered nurse (RN) to delegate to an experienced licensed practical/vocational nurse (LPN/VN)? a. Reinforcement of teaching about the prescribed medications b. Evaluation of the patient's response to walking in the hallway c. Completion of the referral form for a home health nurse follow-up d. Education of the patient about the pathophysiology of heart disease

ANS: A

37. A female patient is scheduled for an oral glucose tolerance test. Which information from the patient's health history is important for the nurse to communicate to the health care provider regarding this test? a. The patient uses oral contraceptives. b. The patient runs several days a week. c. The patient has been pregnant three times. d. The patient has a family history of diabetes.

ANS: A

4. A patient in the dermatology clinic has a thin, scaly erythematous plaque on the right cheek. Which action should the nurse take? a. Prepare the patient for a skin biopsy. b. Teach the use of corticosteroid cream. c. Explain how to apply tretinoin (Retin-A) to the face. d. Discuss the need for topical application of antibiotics.

ANS: A

4. Assessment of a patient's visual acuity reveals that the left eye can see at 20 feet what a person with normal vision can see at 50 feet and the right eye can see at 20 feet what a person with normal vision can see at 40 feet. Which finding should the nurse record? a. OS 20/50; OD 20/40 b. OU 20/40; OS 50/20 c. OD 20/40; OS 20/50 d. OU 40/20; OD 50/20

ANS: A

4. The nurse is assessing a patient who was recently treated with amoxicillin for acute otitis media of the right ear. Which finding is a priority to report to the health care provider? a. The patient has a temperature of 100.6° F. b. The patient report frequent "popping" in the ear. c. Clear fluid is visible through the tympanic membrane. d. The patient frequently asks the nurse to repeat information.

ANS: A

44. The nurse has administered 4 oz of orange juice to an alert patient whose blood glucose was 62 mg/dL. Fifteen minutes later, the blood glucose is 67 mg/dL. Which action should the nurse take next? a. Give the patient 4 to 6 oz more orange juice. b. Administer the PRN glucagon (Glucagon) 1 mg IM. c. Have the patient eat some peanut butter with crackers. d. Notify the health care provider about the hypoglycemia.

ANS: A

45. Which nursing action can the nurse delegate to experienced unlicensed assistive personnel (UAP) who are working in the diabetic clinic? a. Measure the ankle-brachial index. b. Check for changes in skin pigmentation. c. Assess for unilateral or bilateral foot drop. d. Ask the patient about symptoms of depression.

ANS: A

5. A 65-yr-old patient is being evaluated for glaucoma. Which information given by the patient has implications for the patient's treatment plan? a. "I take metoprolol (Lopressor) for angina." b. "I take aspirin when I have a sinus headache." c. "I have had frequent episodes of conjunctivitis." d. "I have not had an eye examination for 10 years."

ANS: A

5. A patient who has Ménière's disease is admitted with vertigo, nausea, and vomiting. Which nursing intervention will be included in the care plan? a. Dim the lights in the patient's room. b. Encourage increased oral fluid intake. c. Change the patient's position every 2 hours. d. Keep the head of the bed elevated 45 degrees.

ANS: A

6. Propranolol (Inderal) is prescribed for a patient diagnosed with hypertension. The nurse should consult with the health care provider before giving this drug when the patient reveals a history of: a. asthma. b. daily alcohol use. c. peptic ulcer disease. d. myocardial infarction (MI).

ANS: A

6. The nurse is assessing a 55-yr-old female patient with type 2 diabetes who has a body mass index (BMI) of 31 kg/m2 .Which goal in the plan of care is most important for this patient? a. The patient will reach a glycosylated hemoglobin level of less than 7%. b. The patient will follow a diet and exercise plan that results in weight loss. c. The patient will choose a diet that distributes calories throughout the day. d. The patient will state the reasons for eliminating simple sugars in the diet.

ANS: A

6. The nurse is testing the visual acuity of a patient in the outpatient clinic. Which instructions should the nurse give for this test? a. "Stand 20 feet away from the wall chart." b. "Look at an object far away and then near to you." c. "Follow the examiner's finger with your eyes only." d. "Look straight ahead while I check your eyes with a light."

ANS: A

6. Which instructions should the nurse include in the teaching plan for a patient with impetigo? a. Clean the crusted areas with soap and water. b. Spread alcohol-based cleansers on the lesions. c. Avoid use of antibiotic ointments on the lesions. d. Use petroleum jelly (Vaseline) to soften crusty areas.

ANS: A

7. A patient who underwent eye surgery must wear an eye patch until the scheduled postoperative clinic visit. Which patient problem will the nurse address in the plan of care? a. Risk for falls b. Difficulty coping c. Disturbed body image d. Inability to care for home

ANS: A

7. The nurse notes white lesions that resemble milk curds in the back of a patient's throat. Which question by the nurse is appropriate at this time? a. "Are you taking any medications?" b. "Do you have a productive cough?" c. "How often do you brush your teeth?" d. "Have you had an oral herpes infection?"

ANS: A

15. A patient reports dizziness when bending over and of nausea and dizziness associated with physical activities. What exam should the nurse expect to prepare the patient to undergo? a. Tympanometry b. Rotary chair testing c. Pure-tone audiometry d. Bone-conduction testing

ANS: B

9. The nurse is assessing a 65-yr-old patient for presbyopia. Which instruction will the nurse give the patient before the test? a. "Hold this card and read the print out loud." b. "Cover one eye while reading the wall chart." c. "You'll feel a short burst of air directed at your eyeball." d. "A light will be used to look for a change in your pupils."

ANS: A

9. Which nursing action is most important in assisting an older patient who has diabetes to engage in moderate daily exercise? a. Determine what types of activities the patient enjoys. b. Remind the patient that exercise improves self-esteem. c. Teach the patient about the effects of exercise on glucose level. d. Give the patient a list of activities that are moderate in intensity.

ANS: A

10. A newly admitted patient reports waking frequently during the night. The nurse observes the patient wearing a nicotine patch on the right upper arm. Which action should the nurse take first? a. Question the patient about use of the patch at night. b. Suggest that the patient go to bed earlier in the evening. c. Ask the health care provider about prescribing a sedative drug for nighttime use. d. Remind the patient that the benefits of the patch outweigh the short-term insomnia

ANS: A Insomnia can occur when nicotine patches are used all night. This can be resolved by removing the patch in the evening. The other actions may be helpful in improving the patient's sleep, but the initial action should be to ask about nighttime use of the patch and suggest removal of the patch at bedtime.

38. The nurse in the emergency department receives arterial blood gas results for 4 recently admitted patients with obstructive pulmonary disease. The results for which patient will require the most rapid action by the nurse? a. pH 7.28, PaCO2 50 mm Hg, and PaO2 58 mm Hg b. pH 7.48, PaCO2 30 mm Hg, and PaO2 65 mm Hg c. pH 7.34, PaCO2 33 mm Hg, and PaO2 80 mm Hg d. pH 7.31, PaCO2 58 mm Hg, and PaO2 64 mm Hg

ANS: A The pH, PaCO2, and PaO2 indicate that the patient has severe uncompensated respiratory acidosis and hypoxemia. Rapid action will be required to prevent increasing hypoxemia and correct the acidosis.

37. The nurse assesses a patient with non-Hodgkin's lymphoma who is receiving an infusion of rituximab (Rituxan). Which assessment finding would require the most rapid action by the nurse? a. Shortness of breath b. Shivering and chills c. Muscle aches and pains d. Temperature of 100.2° F (37.9° C)

ANS: A Rituximab (Rituxan) is a monoclonal antibody. Shortness of breath should be investigated rapidly because anaphylaxis is a possible reaction to monoclonal antibody administration. The nurse will need to rapidly take actions such as stopping the infusion, assessing the patient further, and notifying the health care provider. The other findings will also require action by the nurse but are not indicative of life-threatening complicatio

35. A patient who is experiencing an asthma attack develops bradycardia and a decrease in wheezing. Which action should the nurse take first? a. Notify the health care provider. b. Document changes in respiratory status. c. Encourage the patient to cough and deep breathe. d. Administer IV methylprednisolone (Solu-Medrol)

ANS: A The patient's assessment indicates impending respiratory failure, and the nurse should prepare to assist with intubation and mechanical ventilation after notifying the health care provider. IV corticosteroids require several hours before having any effect on respiratory status. The patient will not be able to cough or deep breathe effectively. Documentation is not a priority at this time.

33. When caring for a patient who is pancytopenic, which action by unlicensed assistive personnel (UAP) indicates a need for the nurse to intervene? a. The UAP assists the patient to use dental floss after eating. b. The UAP adds baking soda to the patient's saline oral rinses. c. The UAP puts fluoride toothpaste on the patient's toothbrush. d. The UAP has the patient rinse after meals with a saline solution.

ANS: A Use of dental floss is avoided in patients with pancytopenia because of the risk for infection and bleeding. The other actions are appropriate for oral care of a pancytopenic pa

15. Which patient in the ear, nose, and throat clinic should the nurse assess first? a. A patient who reports having a sore throat and has a muffled voice. b. A patient with a history of a total laryngectomy whose stoma is red. c. A patient who has a "scratchy throat" and a positive rapid strep antigen test. d. A patient who is receiving radiation for throat cancer and has severe fatigue

ANS: A A muffled voice suggests a possible peritonsillar abscess that could lead to an airway obstruction requiring rapid assessment and potential treatment. A tracheal stoma is normally red. Strep throat and fatigue do not indicate life-threatening problems.

17. The nurse cares for an agitated patient who was admitted to the emergency department after taking a hallucinogenic drug and trying to jump from a third-story window. Which patient problem should the nurse assign as the highest priority? a. Anxiety b. Risk for injury c. Substance abuse d. Disturbed coping

ANS: A Although all the diagnoses may be appropriate for the patient, the highest priority is to address the patient's immediate risk for injury.

36. A patient who is experiencing an acute asthma attack is admitted to the emergency department. Which assessment should the nurse complete first? a. Listen to the patient's breath sounds. b. Ask about inhaled corticosteroid use. c. Determine when the dyspnea started. d. Measure forced expiratory volume (FEV) flow rate

ANS: A Assessment of the patient's breath sounds will help determine how effectively the patient is ventilating and whether rapid intubation may be necessary. The length of time the attack has persisted is not as important as determining the patient's status at present. Most patients having an acute attack will be unable to cooperate with an FEV measurement. It is important to know about the medications the patient is using but not as important as assessing the breath sounds.

14. A patient with acute dyspnea is scheduled for a spiral computed tomography (CT) scan. Which information obtained by the nurse is a priority to communicate to the health care provider before the CT? a. Allergy to shellfish b. Apical pulse of 104 c. Respiratory rate of 30 d. O2 saturation of 90%

ANS: A Because iodine-based contrast media is used during a spiral CT, the patient may need to have the CT scan without contrast or be premedicated before injection of the contrast media. The increased pulse, low oxygen saturation, and tachypnea all indicate a need for further assessment or intervention but do not indicate a need to modify the CT procedure.

14. A patient who uses a fentanyl (Duragesic) patch for chronic abdominal pain caused by ovarian cancer asks the nurse to administer the prescribed hydrocodone tablets, but the patient is asleep when the nurse returns with the medication. Which action is best for the nurse to take? a. Wake the patient and administer the hydrocodone. b. Suggest the use of nondrug therapies for pain relief. c. Wait until the patient wakes up and reassess the pain. d. Consult with the health care provider about the fentanyl dose

ANS: A Because patients with chronic pain frequently use withdrawal and decreased activity as coping mechanisms for pain, sleep is not an indicator that the patient is pain free. The nurse should wake the patient and administer the hydrocodone.

9. A patient who had surgery for a perforated gastric ulcer has been receiving nasogastric suction for 3 days. The patient's serum sodium level is 127 mEq/L (127 mmol/L). Which prescribed therapy should the nurse question? a. Infuse 5% dextrose in water intravenously at 125 mL/hr. b. Administer IV morphine sulfate 4 mg every 2 hours PRN. c. Give IV metoclopramide 10 mg every 6 hours PRN for nausea. d. Administer 3% saline intravenously at 50 mL/hr for a total of 200 mL.

ANS: A Because the patient's gastric suction has been depleting electrolytes, the IV solution should include electrolyte replacement. Solutions such as lactated Ringer's solution would usually be ordered for this patient. The other orders are appropriate for a postoperative patient with gastric suction.

6. A patient who is diagnosed with cervical cancer classified as Tis, N0, M0 asks the nurse what the letters and numbers mean. Which response by the nurse is accurate? a. "The cancer involves only the cervix." b. "The cancer cells look like normal cells." c. "Further testing is needed to determine the spread of the cancer." d. "It is difficult to determine the original site of the cervical cancer."

ANS: A Cancer in situ indicates that the cancer is localized to the cervix and is not invasive at this time. Cell differentiation is not indicated by clinical staging. Because the cancer is in situ, the origin is the cervix. Further testing is not indicated given that the cancer has not spread.

1. The nurse provides discharge instructions to a patient who has an immune deficiency involving the T lymphocytes. Which health screening should the nurse include in the teaching plan for this patient? a. Screening for cancers b. Screening for allergies c. Screening for antibody deficiencies d. Screening for autoimmune disorders

ANS: A Cell-mediated immunity is responsible for the recognition and destruction of cancer cells. Allergic reactions, autoimmune disorders, and antibody deficiencies are mediated primarily by B lymphocytes and humoral immunity

19. A patient arrives in the emergency department with a possible nasal fracture after being hit by a baseball. Which finding by the nurse is most important to report to the health care provider? a. Clear nasal drainage b. Report of nasal pain c. Bilateral nose swelling and bruising d. Inability to breathe through the nose

ANS: A Clear nasal drainage may indicate a meningeal tear with leakage of cerebrospinal fluid. This would place the patient at risk for complications such as meningitis. The other findings are typical with a nasal fracture and do not indicate complications.

5. A patient with chronic back pain has learned to control the pain with the use of imagery and hypnosis. The patient's spouse asks the nurse how these techniques work. Which response by the nurse is accurate? a. "The strategies work by affecting the perception of pain." b. "These techniques block the pain pathways of the nerves." c. "These strategies prevent transmission of stimuli from the back to the brain." d. "The therapies slow the release of chemicals in the spinal cord that cause pain."

ANS: A Cognitive therapies affect the perception of pain by the brain rather than affecting efferent or afferent pathways or influencing the release of chemical transmitters in the dorsal horn.

26. The nurse administers prescribed therapies for a patient with cor pulmonale and right-sided heart failure. Which assessment could be used to evaluate the effectiveness of the therapies? a. Observe for distended neck veins. b. Auscultate for crackles in the lungs. c. Palpate for heaves or thrills over the heart. d. Monitor for elevated white blood cell count

ANS: A Cor pulmonale is right ventricular failure caused by pulmonary hypertension, so clinical manifestations of right ventricular failure such as peripheral edema, jugular venous distention, and right upper-quadrant abdominal tenderness would be expected. Crackles in the lungs are likely to be heard with left-sided heart failure. Findings in cor pulmonale include evidence of right ventricular hypertrophy on electrocardiography and an increase in intensity of the second heart sound. Heaves or thrills are not common with cor pulmonale. White blood count elevation might indicate infection but is not expected with cor pulmonale.

3. A young adult patient scheduled for an annual physical examination arrives in the clinic smelling of cigarette smoke and carrying a pack of cigarettes. Which action should the nurse plan to take? a. Urge the patient to quit smoking as soon as possible. b. Wait for the patient to start a discussion about smoking. c. Avoid confronting the patient about smoking at this time. d. Explain that the "cold turkey" method is most effective to stop smoking.

ANS: A Current national guidelines indicate that health care professionals should urge patients who smoke to quit smoking at every encounter. The other actions will not help decrease the patient's health risks related to smoking.

18. A patient who has diabetes and acute abdominal pain is admitted for an exploratory laparotomy. When planning postoperative interventions to promote wound healing, what is the nurse's highest priority? a. Maintaining the patient's blood glucose within a normal range b. Ensuring that the patient has an adequate dietary protein intake c. Giving antipyretics to keep the temperature less than 102° F (38.9° C) d. Redressing the surgical incision with a dry, sterile dressing twice daily

ANS: A Elevated blood glucose will impair wound healing in multiple ways. Ensuring adequate nutrition is important for the postoperative patient, but a higher priority is blood glucose control. A temperature of 102° F will not impact wound healing. Application of a dry, sterile dressing daily may be ordered, but frequent dressing changes for a wound healing by primary intention is not necessary to promote wound healing.

18. A patient with renal failure is on a low phosphate diet. Which food should the nurse instruct unlicensed assistive personnel (UAP) to remove from the patient's food tray? a. Skim milk b. Grape juice c. Mixed green salad d. Fried chicken breast

ANS: A Foods high in phosphate include milk and other dairy products, so these are restricted on low-phosphate diets. Green, leafy vegetables; high-fat foods; and fruits and juices are not high in phosphate and are not restricte

21. A patient who has a history of ongoing opioid use is hospitalized for surgery. After a visit by a friend, the nurse finds that the patient is unresponsive with pinpoint pupils. Which prescribed medication should the nurse expect to administer? a. Naloxone b. Diazepam (Valium) c. Clonidine (Catapres) d. Methadone (Dolophine)

ANS: A The patient's assessment indicates an opioid overdose, and naloxone should be given to prevent respiratory arrest. The other medications may be used to decrease symptoms associated with opioid withdrawal but would not be appropriate for an overdose.

1. The nurse is advising a patient who was exposed 4 days ago to human immunodeficiency virus (HIV) through unprotected sexual intercourse. The patient's antigen-antibody test has just been reported as negative for HIV. What information should the nurse give to this patient? a. "You will need to be retested in 2 weeks." b. "You do not need to fear infecting others." c. "We won't know for about 10 years if you have HIV infection." d. "With no symptoms and this negative test, you do not have HIV."

ANS: A HIV screening tests detect HIV-specific antibodies or antigens. However, there may be a delay between infection and the time a screening test is able to detect HIV. The typical "window period" for antigen-antibody combination assays is approximately 2 weeks. It is not known based on this information whether the patient is infected with HIV or can infect others. It would be best practice to have him return for repeat testing in approximately 2 weeks.

43. Which finding in a patient hospitalized with bronchiectasis is most important to report to the health care provider? a. Cough productive of bloody, purulent mucus b. Report of sharp chest pain with deep breathing c. Scattered crackles and wheezes heard bilaterally d. Respiratory rate 28 breaths/min while ambulating

ANS: A Hemoptysis may indicate life-threatening hemorrhage and should be reported immediately to the health care provider. The other findings are frequently noted in patients with bronchiectasis and may need further assessment but are not indicators of life-threatening complications.

13. A patient receives 3% NaCl solution for correction of hyponatremia. Which assessment is most important for the nurse to monitor while the patient is receiving this infusion? a. Lung sounds b. Urinary output c. Peripheral pulses d. Peripheral edema

ANS: A Hypertonic solutions cause water retention, so the patient should be monitored for symptoms of fluid excess. Crackles in the lungs may indicate the onset of pulmonary edema and are a serious manifestation of fluid excess. Peripheral pulses, peripheral edema, or changes in urine output are also important to monitor when administering hypertonic solutions, but they do not indicate acute respiratory or cardiac decompensation

28. A patient has a serum calcium level of 7.0 mEq/L. Which assessment finding is most important for the nurse to report to the health care provider? a. The patient is experiencing stridor. b. The patient reports generalized fatigue. c. The patient's bowels have not moved for 4 days. d. The patient has numbness and tingling of the lips.

ANS: A Hypocalcemia can cause laryngeal stridor, which may lead to respiratory arrest. Rapid action is required to correct the patient's calcium level. The other data are also consistent with hypocalcemia, but do not indicate a need for as immediate action as laryngospasm.

16. A patient who has been receiving diuretic therapy is admitted to the emergency department with a serum potassium level of 3.0 mEq/L. The nurse should alert the health care provider immediately that the patient is on which medication? a. Digoxin (Lanoxin) 0.25 mg/day b. Ibuprofen 400 mg every 6 hours c. Lantus insulin 24 U every evening d. Metoprolol (Lopressor) 12.5 mg/day

ANS: A Hypokalemia increases the risk for digoxin toxicity, which can cause serious dysrhythmias. The nurse will need to do more assessment about the other medications, but they are not of as much concern with the potassium level.

22. Interleukin-2 (IL-2) is used as adjuvant therapy for a patient with metastatic renal cell carcinoma. Which information should the nurse include when explaining the purpose of this therapy to the patient? a. IL-2 enhances the body's immunologic response to tumor cells. b. IL-2 prevents bone marrow depression caused by chemotherapy. c. IL-2 protects normal cells from harmful effects of chemotherapy. d. IL-2 stimulates cancer cells in their resting phase to enter mitosis

ANS: A IL-2 enhances the ability of the patient's own immune response to suppress tumor cells. IL-2 does not protect normal cells from damage caused by chemotherapy, stimulate cancer cells to enter mitosis, or prevent bone marrow depression.

2. The nurse assesses the chest of a patient with pneumococcal pneumonia. Which finding would the nurse expect? a. Increased tactile fremitus b. Dry, nonproductive cough c. Hyperresonance to percussion d. A grating sound on auscultation

ANS: A Increased tactile fremitus over the area of pulmonary consolidation is expected with bacterial pneumonias. Dullness to percussion would be expected. Pneumococcal pneumonia typically presents with a loose, productive cough. Adventitious breath sounds such as crackles and wheezes are typical. A grating sound is more representative of a pleural friction rub rather than pneumonia

2. The nurse plans postoperative care for a patient who smokes 2 packs of cigarettes daily. Which goal should the nurse include in the plan of care for this patient? a. Promote sleep. b. Enhance appetite. c. Decrease diarrhea. d. Prevent sore throat.

ANS: A Insomnia is a characteristic of nicotine withdrawal. Diarrhea, sore throat, and anorexia are not symptoms associated with nicotine withdrawal.

6. The nurse is caring for a mechanically ventilated patient with a cuffed tracheostomy tube. Which action by the nurse would determine if the cuff has been properly inflated? a. Use a hand-held manometer to measure cuff pressure. b. Review the health record for the prescribed cuff pressure. c. Suction the patient through a fenestrated inner cannula to clear secretions. d. Insert the decannulation plug before removing the nonfenestrated inner cannula

ANS: A Measurement of cuff pressure using a manometer to ensure that cuff pressure is 20 mm Hg or lower will avoid compression of the tracheal wall and capillaries. Never insert the decannulation plug in a tracheostomy tube until the cuff is deflated and the nonfenestrated inner cannula is removed. Otherwise, the patient's airway is occluded. A health care provider's order is not required to determine safe cuff pressure. A nonfenestrated inner cannula must be used to suction a patient to prevent tracheal damage occurring from the suction catheter passing through the fenestrated openings.

20. An older adult who takes medications for coronary artery disease and hypertension is newly diagnosed with HIV infection and is starting antiretroviral therapy. Which information will the nurse include in patient teaching? a. Many drugs interact with antiretroviral medications. b. HIV infections progress more rapidly in older adults. c. Less frequent CD4+ level monitoring is needed in older adults. d. Hospice care is available for patients with terminal HIV infection

ANS: A The nurse will teach the patient about potential interactions between antiretrovirals and the medications that the patient is using for chronic health problems. Treatment and monitoring of HIV infection is not affected by age. A patient beginning early ART is not a candidate for hospice. Progression of HIV is not affected by age although it may be affected by chronic disease.

18. A patient who has just started taking sustained-release morphine sulfate (MS Contin) for chronic arthritic joint pain after a traumatic injury reports nausea and abdominal fullness. Which action should the nurse take initially? a. Administer the ordered antiemetic medication. b. Order the patient a clear liquid diet until the nausea decreases. c. Tell the patient that the nausea should subside in about a week. d. Consult with the health care provider about using a different opioid.

ANS: A Nausea is frequently experienced with the initiation of opioid therapy, and antiemetics usually are prescribed to treat this expected side effect. The best choice would be to administer the antiemetic medication so the patient can eat. There is no indication that a different opioid is needed, although if the nausea persists, the health care provider may order a change of opioid. Although tolerance develops and the nausea will subside in about a week, it is not appropriate to allow the patient to continue to be nauseated. A clear liquid diet may decrease the nausea but may not provide needed nutrients for injury healing

11. An older adult is receiving standard multidrug therapy for tuberculosis (TB). Which finding should the nurse report to the health care provider? a. Yellow-tinged sclera b. Orange-colored sputum c. Thickening of the fingernails d. Difficulty hearing high-pitched voices

ANS: A Noninfectious hepatitis is a toxic effect of isoniazid, rifampin, and pyrazinamide, and patients who develop hepatotoxicity will need to use other medications. Changes in hearing and nail thickening are not expected with the four medications used for initial TB drug therapy. Presbycusis is an expected finding in the older adult patient. Orange discoloration of body fluids is an expected side effect of rifampin and not an indication to call the health care provider.

31. The nurse is caring for a patient who has just had a thoracentesis. Which assessment information obtained by the nurse is a priority to communicate to the health care provider? a. O2 saturation is 88%. b. Blood pressure is 155/90 mm Hg. c. Respiratory rate is 24 breaths/min when lying flat. d. Pain level is 5 (on 0 to 10 scale) with a deep breath

ANS: A O2 saturation should improve after a thoracentesis. A saturation of 88% indicates that a complication such as pneumothorax may be occurring. The other assessment data also indicate a need for ongoing assessment or intervention, but the low O2 saturation is the priority.

18. The nurse is caring for a hospitalized older patient who has nasal packing in place after a nosebleed. Which assessment finding will require the most immediate action by the nurse? a. The oxygen saturation is 89%. b. The nose appears red and swollen. c. The patient reports level 8 (0 to 10 scale) pain. d. The patient's temperature is 100.1° F (37.8° C)

ANS: A Older patients with nasal packing are at risk of aspiration or airway obstruction. An O2 saturation of 89% should alert the nurse to further assess for these complications. The other assessment data indicate a need for nursing action but not as immediately as the low O2 saturation.

22. A patient with chronic obstructive pulmonary disease (COPD) has poor gas exchange. Which action by the nurse would support the patient's ventilation? a. Encourage the patient to sit in a chair and lean forward. b. Have the patient rest with the head elevated 15 degrees. c. Place the patient in the Trendelenburg position with pillows behind the head. d. Ask the patient to rest in bed in a high-Fowler's position with the knees flexed.

ANS: A Patients with COPD improve the mechanics of breathing by sitting up in the "tripod" position. Resting in bed with the head elevated in a semi-Fowler's position would be an alternative position if the patient was confined to bed but sitting in a chair allows better ventilation. The Trendelenburg position or sitting upright in bed with the knees flexed would decrease the patient's ability to ventilate well.

12. A new nurse performs a dressing change on a patient's stage 2 left heel pressure injury. Which action by the new nurse indicates a need for further teaching about pressure injury care? a. The new nurse cleans the injury with half-strength peroxide. b. The new nurse applies a hydrocolloid dressing on the injury. c. The new nurse irrigates the pressure injury with saline using a 30-mL syringe. d. The new nurse inserts a sterile cotton-tipped applicator into the pressure injury.

ANS: A Pressure injuries should not be cleaned with solutions that are cytotoxic, such as hydrogen peroxide. The other actions by the new nurse are appropriate.

8. A nurse obtains a health history from a patient who has a 35 pack-year smoking history. The patient reports hoarseness and tightness in the throat and difficulty swallowing. Which question is important for the nurse to ask? a. "How much alcohol do you drink in an average week?" b. "Do you have a family history of head or neck cancer?" c. "Have you had frequent streptococcal throat infections?" d. "Do you use antihistamines for upper airway congestion?"

ANS: A Prolonged alcohol use and smoking are associated with the development of laryngeal cancer, which the patient's symptoms and history suggest. Family history is not a risk factor for head or neck cancer. Frequent antihistamine use would be asked about if the nurse suspected allergic rhinitis, but the patient's symptoms are not suggestive of this diagnosis. Patients with streptococcal throat infections will also have pain and a fever.

19. Which finding is most important for the nurse to communicate to the health care provider when caring for a patient who is receiving negative-pressure wound therapy? a. Low serum albumin level b. Serosanguineous drainage c. Deep red and moist wound bed d. Cobblestone wound appearance

ANS: A Serum protein levels may decrease with negative pressure therapy, which will adversely affect wound healing. The other findings are expected with wound healing

13. A patient arrives in the emergency department with a swollen ankle after a soccer injury. Which action by the nurse is appropriate? a. Elevate the ankle above heart level. b. Apply a warm moist pack to the ankle. c. Ask the patient to try bearing weight on the ankle. d. Assess the ankle's passive range of motion (ROM).

ANS: A Soft tissue injuries are treated with rest, ice, compression, and elevation (RICE). Elevation of the ankle will decrease tissue swelling. Moving the ankle through the ROM will increase swelling and risk further injury. Cold packs should be applied the first 24 hours to reduce swelling. The nurse should not ask the patient to move or bear weight on the swollen ankle because immobilization of the inflamed or injured area promotes healing by decreasing metabolic needs of the tissues.

14. What should the nurse teach a patient who is scheduled to complete a 24-hour urine collection for 17-ketosteroids? a. To insert and maintain a retention catheter b. To keep the specimen refrigerated or on ice c. To drink at least 3 L of fluid during the 24 hours d. To void and save the specimen to start the collection

ANS: B

18. Eight years after seroconversion, a patient with human immunodeficiency virus infection has a CD4+ cell count of 800/µL and an undetectable viral load. What should be included in the plan of care at this time? a. Encourage adequate nutrition, exercise, and sleep. b. Teach about the side effects of antiretroviral agents. c. Explain opportunistic infections and antibiotic prophylaxis. d. Monitor symptoms of acquired immunodeficiency syndrome (AIDS).

ANS: A The CD4+ level for this patient is in the normal range, indicating that the patient is the stage of asymptomatic chronic infection when the body is able to produce enough CD4+ cells to maintain a normal CD4+ count. Maintaining healthy lifestyle behaviors is an important goal in this stage. AIDS and increased incidence of opportunistic infections typically develop when the CD4+ count is much lower than normal. Although the initiation of ART is highly individual, it would not be likely that a patient with a normal CD4+ level would receive ART.

21. A patient who is using both a fentanyl (Duragesic) patch and immediate-release morphine for chronic cancer pain develops new-onset confusion, dizziness, and a decrease in respiratory rate. Which action should the nurse take first? a. Remove the fentanyl patch. b. Obtain complete vital signs. c. Notify the health care provider. d. Administer prescribed PRN naloxone.

ANS: A The assessment data indicate a possible overdose of opioid. The first action should be to remove the patch. Naloxone administration in a patient who has been chronically using opioids can precipitate withdrawal and would not be the first action. Notification of the health care provider and continued monitoring are also needed, but the patient's data indicate that more rapid action is needed. The respiratory rate alone is an indicator for immediate action before obtaining blood pressure, pulse, and temperature

10. Which information about a patient population would be most useful to help the nurse plan for human immunodeficiency virus (HIV) testing needs? a. Age b. Lifestyle c. Symptoms d. Sexual orientation

ANS: A The current Centers for Disease Control and Prevention policy is to offer routine testing for HIV to all individuals age 13 to 64 years. Although lifestyle, symptoms, and sexual orientation may suggest increased risk for HIV infection, the goal is to test all individuals in this age range.

16. To evaluate the effectiveness of antiretroviral therapy (ART), which laboratory test result will the nurse review? a. Viral load testing b. Enzyme immunoassay c. Rapid HIV antibody testing d. Immunofluorescence assay

ANS: A The effectiveness of ART is measured by the decrease in the amount of virus detectable in the blood. The other tests are used to detect HIV antibodies, which remain positive even with effective ART

5. The emergency department nurse is evaluating the outcomes for a patient who has received treatment during an asthma attack. Which assessment finding is the best indicator that the therapy has been effective? a. O2 saturation is >90%. b. No wheezes are audible. c. Respiratory rate is 16 breaths/min. d. Accessory muscle use has decreased.

ANS: A The goal for treatment of an asthma attack is to keep the O2 saturation above 90%. The other patient data may occur when the patient is too fatigued to continue with the increased work of breathing required in an asthma attack

25. A patient with renal failure who arrives for outpatient hemodialysis is unresponsive to questions and has decreased deep tendon reflexes. Family members report that the patient has been taking aluminum hydroxide/magnesium hydroxide suspension (Maalox) at home for indigestion. Which action should the nurse take first? a. Notify the patient's health care provider. b. Obtain an order to draw a potassium level. c. Review the last magnesium level on the patient's chart. d. Teach the patient about magnesium-containing antacids

ANS: A The health care provider should be notified immediately. The patient has a history and manifestations consistent with hypermagnesemia. The nurse should check the chart for a recent serum magnesium level and make sure that blood is sent to the laboratory for immediate electrolyte and chemistry determinations. Dialysis should correct the high magnesium levels. The patient needs teaching about the risks of taking magnesium-containing antacids. Monitoring of potassium levels also is important for patients with renal failure, but the patient's current symptoms are not consistent with hyperkalemia

2. A patient with an open leg lesion has a white blood cell (WBC) count of 13,500/µL and a band count of 11%. What prescribed action should the nurse take first? a. Obtain cultures of the wound. b. Begin antibiotic administration. c. Continue to monitor the wound for drainage. d. Redress the wound with wet-to-dry dressings.

ANS: A The increase in WBC count with the increased bands (shift to the left) indicates that the patient probably has a bacterial infection, and the nurse should obtain wound cultures. Antibiotic therapy and/or dressing changes may be started, but cultures should be done first. The nurse will continue to monitor the wound, but additional actions are needed as well.

20. The nurse assesses a patient with chronic obstructive pulmonary disease (COPD) who has been admitted after increasing dyspnea over the past 3 days. Which finding is important for the nurse to report to the health care provider? a. Respirations are 36 breaths/min. b. Anterior-posterior chest ratio is 1:1. c. Lung expansion is decreased bilaterally. d. Hyperresonance to percussion is present.

ANS: A The increase in respiratory rate indicates respiratory distress and a need for rapid interventions such as administration of O2 or medications. The other findings are common chronic changes occurring in patients with COPD.

14. Employee health test results reveal a tuberculosis (TB) skin test of 16-mm induration and a negative chest x-ray for a staff nurse working on the pulmonary unit. The nurse has no symptoms of TB and has never had a positive TB skin test before. Which information should the occupational health nurse plan to teach the staff nurse? a. Use and side effects of isoniazid b. Standard four-drug therapy for TB c. Need for annual repeat TB skin testing d. Bacille Calmette-Guérin (BCG) vaccine

ANS: A The nurse is considered to have a latent TB infection and should be treated with INH daily for 6 to 9 months. The four-drug therapy would be appropriate if the nurse had active TB. TB skin testing is not done for those who have already had a positive skin test result. BCG vaccine is not used in the United States for TB and would not be helpful for this individual, who already has a TB infection

15. A patient who is lethargic and with deep, rapid respirations has the following arterial blood gas (ABG) results: pH 7.32, PaO2 88 mm Hg, PaCO2 35 mm Hg, and HCO3 16 mEq/L. How should the nurse interpret these results? a. Metabolic acidosis b. Metabolic alkalosis c. Respiratory acidosis d. Respiratory alkalosis

ANS: A The pH and HCO3 indicate that the patient has a metabolic acidosis. The ABGs are inconsistent with the other responses.

13. Ten days after receiving a bone marrow transplant, a patient develops a skin rash. What should the nurse suspect is the cause of the rash? a. The donor T cells are attacking the patient's skin cells. b. The patient needs treatment to prevent hyperacute rejection. c. The patient's antibodies are rejecting the donor bone marrow. d. The patient is experiencing a delayed hypersensitivity reaction.

ANS: A The patient's history and symptoms indicate that the patient is experiencing graft-versus-host disease, in which the donated T cells attack the patient's tissues. The history and symptoms are not consistent with rejection or delayed hypersensitivity

42. A patient who was admitted the previous day with pneumonia reports a sharp pain of 7 (on 0 to 10 scale) "whenever I take a deep breath." Which action will the nurse take next? a. Auscultate for breath sounds. b. Administer as-needed morphine. c. Have the patient cough forcefully. d. Notify the patient's health care provider

ANS: A The patient's statement indicates that pleurisy or a pleural effusion may have developed, and the nurse will need to listen for a pleural friction rub and decreased breath sounds. Assessment should occur before administration of pain medications. The patient is unlikely to be able to cough forcefully until pain medication has been administered. The nurse will want to obtain more assessment data before calling the health care provider.

4. An older adult patient who is having an annual checkup tells the nurse, "I feel fine, and I don't want to pay for all these unnecessary cancer screening tests!" Which information should the nurse plan to teach this patient? a. Consequences of aging on cell-mediated immunity b. Decrease in antibody production associated with aging c. Incidence of cancer-associated infections in older adults d. Impact of poor nutrition on immune function in older adults

ANS: A The primary impact of aging on immune function is on T cells, which are important for immune surveillance and tumor immunity. Antibody function is not affected as much by aging. Poor nutrition can also contribute to decreased immunity, but there is no evidence that it is a contributing factor for this patient. Although some types of cancer are associated with specific infections, this patient does not have an active infect

13. A disoriented and agitated patient comes to the emergency department and admits using methamphetamine. Vital signs are blood pressure 164/94 mm Hg, heart rate 136 beats/min and irregular, and respirations 32 breaths/min. Which action by the nurse is most important? a. Monitor the patient's electrocardiogram. b. Reorient the patient at frequent intervals. c. Keep the patient in a quiet and darkened room. d. Obtain a health history including prior drug use

ANS: A The priority is to ensure physiologic stability given that methamphetamine use can lead to complications such as myocardial infarction. The other actions are also appropriate but are not of as high a priority.

7. A patient with a history of heavy alcohol use is diagnosed with acute gastritis. Which statement by the patient indicates a willingness to stop alcohol use? a. "I am older and wiser now, and I can change my drinking behavior." b. "Alcohol has never bothered my stomach before. I think I have the flu." c. "People say that I drink too much, but I feel pretty good most of the time." d. "My drinking is affecting my stomach, but medication will help me feel better."

ANS: A The statement "I am older and wiser now, and I can change my drinking behavior" indicates the patient expresses willingness to stop alcohol use and an initial commitment to changing alcohol intake behaviors. In the remaining statements, the patient recognizes that alcohol use is the reason for the gastritis but is not yet willing to make a change.

17. A patient hospitalized with chronic obstructive pulmonary disease (COPD) is being discharged home on O2 therapy. Which instruction should the nurse include in the discharge teaching? a. O2 use can improve the patient's quality of life. b. Travel is not possible with the use of O2 devices. c. O2 flow should be increased if the patient has more dyspnea. d. Storage of O2 requires large metal tanks that last 4 to 6 hours

ANS: A The use of home O2 improves quality of life and prognosis. Because increased dyspnea may be a symptom of an acute process such as pneumonia, the patient should notify the health care provider rather than increasing the O2 flowrate if dyspnea becomes worse. O2 can be supplied using liquid, storage tanks, or concentrators, depending on individual patient circumstances. Travel is possible using portable O2 concentrators

1. Which finding by the nurse most specifically indicates that a patient is not able to effectively clear the airway? a. Weak cough effort b. Profuse green sputum c. Respiratory rate of 28 breaths/min d. Resting pulse oximetry (SpO2) of 85%

ANS: A The weak cough effort indicates that the patient is unable to clear the airway effectively. The other data suggest problems with gas exchange and breathing pattern

33. The nurse notes a serum calcium level of 7.9 mg/dL for a patient who has chronic malnutrition. Which action should the nurse expect to take first? a. Monitor ionized calcium level. b. Give oral calcium citrate tablets. c. Check parathyroid hormone level. d. Administer vitamin D supplements

ANS: A This patient with chronic malnutrition is likely to have a low serum albumin level, which will affect the total serum calcium. A more accurate reflection of calcium balance is the ionized calcium level. Most of the calcium in the blood is bound to protein (primarily albumin). Alterations in serum albumin levels affect the interpretation of total calcium levels. Low albumin levels result in a drop in the total calcium level, although the level of ionized calcium is not affected. The other actions may be needed if the ionized calcium is also decreased.

39. Which nursing action for a patient with chronic obstructive pulmonary disease (COPD) could the nurse delegate to experienced unlicensed assistive personnel (UAP)? a. Measure O2 saturation using pulse oximetry. b. Monitor for increased O2 need with exercise. c. Teach the patient about safe use of O2 at home. d. Adjust O2 to keep saturation in prescribed parameters

ANS: A UAP can obtain O2 saturation (after being trained and evaluated in the skill). The other actions require more education and a scope of practice that licensed practical/vocational nurses (LPN/VNs) or registered nurses (RNs) would have

48. The nurse is caring for a patient who has a right-sided chest tube after a right lower lobectomy. Which nursing action can the nurse delegate to the unlicensed assistive personnel (UAP)? a. Document the amount of drainage every 8 hours. b. Obtain samples of drainage for culture from the system. c. Assess patient pain level associated with the chest tube. d. Check the water-seal chamber for the correct fluid level.

ANS: A UAP education includes documentation of intake and output. The other actions are within the scope of practice and education of licensed nursing personnel.

36. Which action should the nurse take when caring for a patient who is receiving chemotherapy and reports problems with concentration? a. Suggest use of a daily planner and encourage adequate sleep. b. Teach the patient to rest the brain by avoiding new activities. c. Teach that "chemo-brain" is a short-term effect of chemotherapy. d. Report patient symptoms immediately to the health care provider.

ANS: A Use of tools to enhance memory and concentration such as a daily planner and adequate rest are helpful for patients who develop "chemo-brain" while receiving chemotherapy. Patients should be encouraged to exercise the brain through new activities. Chemo-brain may be short or long term. There is no urgent need to report common chemotherapy side effects to the provider.

8. A young adult patient who denies any history of smoking is seen in the clinic with a new diagnosis of chronic obstructive pulmonary disease (COPD). What topic should the nurse plan to teach the patient? a. a1-antitrypsin testing b. Leukotriene modifiers c. Use of the nicotine patch d. Continuous pulse oximetry

ANS: A When COPD occurs in young patients, especially without a smoking history, a genetic deficiency in 1-antitrypsin should be suspected. Because the patient does not smoke, a nicotine patch would not be ordered. There is no indication that the patient requires continuous pulse oximetry. Leukotriene modifiers would be used in patients with asthma, not with COPD.

7. Which statement by the patient indicates that teaching has been effective for a patient scheduled for radiation therapy of the larynx? a. "I will need to buy a water bottle to carry with me." b. "I should not use any lotions on my neck and throat." c. "Until the radiation is complete, I may have diarrhea." d. "Alcohol-based mouthwashes will help clean my mouth."

ANS: A Xerostomia can be partially alleviated by drinking fluids at frequent intervals. Radiation will damage tissues at the site being radiated but should not affect the abdominal organs, so loose stools are not a usual complication of head and neck radiation therapy. Frequent oral rinsing with non-alcohol-based rinses is recommended. Prescribed lotions and sunscreen may be used on radiated skin, although they should not be used just before the radiation therapy

4. On auscultation of a patient's lungs, the nurse hears low-pitched, bubbling sounds during inhalation in the lower third bilaterally. How should the nurse document this finding? a. Inspiratory crackles at the bases b. Expiratory wheezes in both lungs c. Abnormal lung sounds in the apices of both lungs d. Pleural friction rub in the right and left lower lobes

ANS: A Crackles are low-pitched, bubbling sounds usually heard on inspiration. Wheezes are high-pitched sounds. They can be heard during the expiratory or inspiratory phase of the respiratory cycle. The lower third of both lungs are the bases, not apices. Pleural friction rubs are grating sounds that are usually heard during both inspiration and expiration

19. A patient has a magnesium level of 1.3 mg/dL. Which assessment would help the nurse identify a likely cause of this value? a. Daily alcohol intake b. Dietary protein intake c. Multivitamin with minerals d. Over-the-counter (OTC) laxative

ANS: A Hypomagnesemia is associated with alcoholism. Protein intake would not have a significant effect on magnesium level. OTC laxatives (such as milk of magnesia) and use of multivitamin/mineral supplements tend to increase magnesium levels

1. The nurse is caring for a patient living with asymptomatic chronic HIV infection (HIV). Which prophylactic measures will the nurse include in the plan of care? (Select all that apply.) a. Hepatitis B vaccine b. Pneumococcal vaccine c. Influenza virus vaccine d. Trimethoprim-sulfamethoxazole e. Varicella zoster immune globulin

ANS: A, B, C Asymptomatic chronic HIV infection is a stage between acute HIV infection and a diagnosis of symptomatic chronic HIV infection. Although called asymptomatic, symptoms (e.g., fatigue, headache, low-grade fever, night sweats) often occur. Prevention of other infections is an important intervention in patients who are living with HIV, and these vaccines are recommended as soon as the HIV infection is diagnosed. Antibiotics and immune globulin are used to prevent and treat infections that occur later in the course of the disease when the CD4+ counts have dropped or when infection has occurred

1. Which factors will the nurse consider when calculating the CURB-65 score for a patient with pneumonia? (Select all that apply.) a. Age b. Blood pressure c. Respiratory rate d. O2 saturation e. Presence of confusion f. Blood urea nitrogen (BUN) level

ANS: A, B, C, E, F Data collected for the CURB-65 are mental status (confusion), BUN (elevated), blood pressure (decreased), respiratory rate (increased), and age (65 years and older). The other information is also essential to assess but are not used for CURB-65 scoring.

2. The nurse is reviewing the medical records for five patients who are scheduled for their yearly physical examinations in September. Which patients should receive the inactivated influenza vaccination? (Select all that apply.) a. A 76-yr-old nursing home resident b. A 36-yr-old female patient who is pregnant c. A 42-yr-old patient who has a 15 pack-year smoking history d. A 30-yr-old patient who takes corticosteroids for rheumatoid arthritis e. A 24-yr-old patient who has allergies to penicillin and cephalosporins

ANS: A, B, D Individuals who are pregnant, residents of nursing homes, or are immunocompromised or who have chronic medical conditions should receive inactivated vaccine by injection. The corticosteroid use by the 30-year-old patient increases the risk for infection. Current guidelines suggest that healthy individuals between 6 months and age 49 years receive intranasal immunization with live, attenuated influenza vaccine.

1. The nurse assumes care of a patient who just returned from surgery for a total laryngectomy and radical neck dissection and notes the following problems. In which order should the nurse address the problems? (Put a comma and a space between each answer choice [A, B, C, D].) a. The patient is in a side-lying position with the head of the bed flat. b. The patient is coughing blood-tinged secretions from the tracheostomy. c. The nasogastric (NG) tube is disconnected from suction and clamped off. d. The wound drain in the neck incision contains 200 mL of bloody drainage

ANS: A, B, D, C The patient should first be placed in a semi-Fowler's position to maintain the airway and reduce incisional swelling. The blood-tinged secretions may obstruct the airway, so suctioning is the next appropriate action. Then the wound drain should be drained because the 200 mL of drainage will decrease the amount of suction in the wound drain and could lead to incisional swelling and poor healing. Finally, the NG tube should be reconnected to suction to prevent gastric dilation, nausea, and vomiting.

1. The nurse notes new onset confusion in an older patient who is normally alert and oriented. In which order should the nurse take the following actions? (Put a comma and a space between each answer choice [A, B, C, D].) a. Obtain the O2 saturation. b. Check the patient's pulse rate. c. Document the change in status. d. Notify the health care provider.

ANS: A, B, D, C Assessment for physiologic causes of new onset confusion such as pneumonia, infection, or perfusion problems should be the first action by the nurse. Airway and oxygenation should be assessed first, then circulation. After assessing the patient, the nurse should notify the health care provider. Finally, documentation of the assessments and care should be done.

3. The nurse plans a presentation for community members about how to decrease the risk for antibiotic-resistant infections. Which information will the nurse include in the teaching plan? (Select all that apply.) a. Antibiotics may sometimes be prescribed to prevent infection. b. Continue taking antibiotics until all of the prescription is gone. c. Unused antibiotics that are more than a year old should be discarded. d. Antibiotics are effective in treating influenza associated with high fevers. e. Hand washing is effective in preventing many viral and bacterial infections.

ANS: A, B, E All prescribed doses of antibiotics should be taken. In some situations, such as before surgery, antibiotics are prescribed to prevent infection. There should not be any leftover antibiotics because all prescribed doses should be taken. However, if there are leftover antibiotics, they should be discarded at once because the number left will not be enough to treat a future infection. Hand washing is considered the single most effective action in decreasing infection transmission. Antibiotics are ineffective in treating viral infections such as influenza.

2. Which activities can the nurse working in the outpatient clinic delegate to a licensed practical/vocational nurse (LPN/VN)? (Select all that apply.) a. Administer patch testing to a patient with allergic dermatitis. b. Interview a new patient about chronic health problems and allergies. c. Apply a sterile dressing after the health care provider excises a mole. d. Explain potassium hydroxide testing to a patient with a skin infection. e. Teach a patient about site care after a punch biopsy of an upper arm lesion.

ANS: A, C

1. A patient is scheduled for a computed tomography (CT) scan of the chest with contrast media. Which assessment findings should the nurse report to the health care provider before the patient goes for the CT (Select all that apply.)? a. Allergy to shellfish b. Patient reports claustrophobia c. Elevated serum creatinine level d. Recent bronchodilator inhaler use e. Inability to remove a wedding band

ANS: A, C Because the contrast media is iodine-based and may cause dehydration and decreased renal blood flow, asking about iodine allergies (such as allergy to shellfish) and monitoring renal function before the CT scan are necessary. The other actions are not contraindications for CT of the chest, although they may be for other diagnostic tests, such as magnetic resonance imaging or pulmonary spirometry.

14. The nurse is observing a student who is preparing to perform an ear examination for a 30-year-old patient. Which action by the student indicates that the nurse should intervene? a. Pulls the auricle of the ear up and posterior. b. Chooses a speculum larger than the ear canal. c. Stabilizes the hand holding the otoscope on the patient's head. d. Stops inserting the otoscope after observing impacted cerumen.

ANS: B

1. The nurse at the clinic is interviewing a 64-yr-old woman who is 5 feet, 3 inches tall and weighs 125 lb (57 kg). The patient has not seen a health care provider for 20 years. She walks 5 miles most days and has a glass of wine 2 or 3 times a week. Which topics will the nurse plan to include in patient teaching about cancer screening and decreasing cancer risk? (Select all that apply.) a. Pap testing b. Tobacco use c. Sunscreen use d. Mammography e. Colorectal screening

ANS: A, C, D, E The patient's age, gender, and history indicate a need for screening and teaching about colorectal cancer, mammography, Pap smears, and sunscreen. The patient does not use tobacco or excessive alcohol, she is physically active, and her body weight is healthy.

1. The clinic nurse is teaching a patient with acute sinusitis. Which interventions should the nurse plan to include in the teaching session? (Select all that apply.) a. Decongestants can be used to relieve swelling. b. Avoid blowing the nose to decrease the nosebleed risk. c. Taking a hot shower will increase sinus drainage and decrease pain. d. Saline nasal spray can be made at home and used to wash out secretions. e. You will be more comfortable if you keep your head in an upright position.

ANS: A, C, D, E The steam and heat from a shower will help thin secretions and improve drainage. Decongestants can be used to relieve swelling. Patients can use either over-the-counter sterile saline solutions or home-prepared saline solutions to thin and remove secretions. Maintaining an upright posture decreases sinus pressure and the resulting pain. Blowing the nose after a hot shower or using the saline spray is recommended to expel secretions.

2. A nurse assesses a postoperative patient 2 days after chest surgery. What findings indicate that the patient requires better pain management? (Select all that apply) a. Confusion b. Hypoglycemia c. Poor cough effort d. Shallow breathing e. Elevated temperature

ANS: A, C, D, E Inadequate pain control can decrease tidal volume and cough effort, leading to complications such as pneumonia with increases in temperature. Poor pain control may lead to confusion through a variety of mechanism, including hypoventilation and poor sleep quality. Stressors such as pain cause increased release of corticosteroids that can result in hyperglycemia.

1. Which statements will the nurse include when teaching a patient who is scheduled for oral glucose tolerance testing in the outpatient clinic? (Select all that apply.) a. "You will need to avoid smoking before the test." b. "Exercise should be avoided until the testing is complete." c. "Several blood samples will be obtained during the testing." d. "You should follow a low-calorie diet the day before the test." e. "The test requires that you fast for at least 8 hours before testing."

ANS: A, C, E

2. A patient develops neutropenia after receiving chemotherapy. Which information about ways to prevent infection will the nurse include in the teaching plan? (Select all that apply.) a. Cook food thoroughly before eating. b. Choose low fiber, low residue foods. c. Avoid public transportation such as buses. d. Use rectal suppositories if needed for constipation. e. Talk to the oncologist before having any dental work

ANS: A, C, E Eating only cooked food and avoiding public transportation will decrease infection risk. A high-fiber diet is recommended for neutropenic patients to decrease constipation. Because bacteria may enter the circulation during dental work or oral surgery, the patient may need to postpone dental work or take antibiotics.

1. A patient who has an infected abdominal wound develops a temperature of 104° F (40° C). All the following interventions are included in the patient's plan of care. In which order should the nurse perform the following actions? (Put a comma and a space between each answer choice [A, B, C, D]). a. Administer IV antibiotics. b. Sponge patient with cool water. c. Perform wet-to-dry dressing change. d. Administer acetaminophen (Tylenol).

ANS: A, D, B, C The first action should be to administer the antibiotic because treating the infection that has caused the fever is the most important aspect of fever management. The next priority is to lower the high fever, so the nurse should administer acetaminophen to lower the temperature set point. A cool sponge bath should be done after the acetaminophen is given to lower the temperature further. The wet-to-dry dressing change will not have an immediate impact on the infection or fever and should be done last

2. Which health promotion information should the nurse include when teaching a patient with a 42 pack-year history of cigarette smoking? (Select all that apply.) a. Resources for support in smoking cessation b. Reasons for annual sputum cytology testing c. Erlotinib (Tarceva) therapy to prevent tumor risk d. Computed tomography (CT) screening for cancer e. Importance of obtaining a yearly influenza vaccination

ANS: A, D, E Because smoking is the major cause of lung cancer, an important role for the nurse is teaching patients about the benefits of and means of smoking cessation. Screening for using low-dose CT is recommended for high-risk patients Encourage those at risk for pneumonia (e.g., those who smoke) to obtain both influenza and pneumococcal vaccines. Sputum cytology is a diagnostic test but does not prevent cancer or disease. Erlotinib may be used in patients who have lung cancer, but it is not used to reduce the risk of developing cancer.

1. In which order will the nurse take these steps to prepare NPH 20 units and regular insulin 2 units using the same syringe? (Put a comma and a space between each answer choice [A, B, C, D, E]). a. Rotate NPH vial. b. Withdraw regular insulin. c. Withdraw 20 units of NPH. d. Inject 20 units of air into NPH vial. e. Inject 2 units of air into regular insulin vial.

ANS: A, D, E, B, C

1. A patient who is receiving an IV antibiotic develops wheezes and dyspnea. In which order should the nurse implement these prescribed actions? (Put a comma and a space between each answer choice [A, B, C, D, E]). a. Discontinue the antibiotic. b. Give diphenhydramine IV. c. Inject epinephrine IM or IV. d. Prepare an infusion of dopamine. e. Apply 100% oxygen using a nonrebreather mask

ANS: A, E, C, B, D The nurse should initially discontinue the antibiotic because it is the likely cause of the allergic reaction. Next, oxygen delivery should be maximized, followed by treatment of bronchoconstriction with epinephrine administered IM or IV. Diphenhydramine will work more slowly than epinephrine but will help prevent progression of the reaction. Because the patient currently does not have evidence of hypotension, the dopamine infusion can be prepared last.

1. The nurse is developing a teaching plan for a 64-year-old patient with coronary artery disease (CAD). Which factor should the nurse focus on during the teaching session? a. Family history of coronary artery disease b. Elevated low-density lipoprotein (LDL) level c. Greater risk associated with the patient's gender d. Increased risk of cardiovascular disease with aging

ANS: B

1. The nurse is performing an eye examination on a 76-yr-old patient. Which finding indicates that the nurse should refer the patient for a more extensive assessment? a. The patient's sclerae are light yellow. b. The patient reports persistent photophobia. c. The pupil recovers slowly after responding to a bright light. d. There is a whitish gray ring encircling the periphery of the iris.

ANS: B

1. Which action should the nurse in the hypertension clinic take to obtain an accurate baseline blood pressure (BP) for a new patient? a. Deflate the BP cuff at a rate of 5 to 10 mm Hg per second. b. Have the patient sit in a chair with the feet flat on the floor. c. Assist the patient to the supine position for BP measurements. d. Obtain two BP readings in the dominant arm and average the results.

ANS: B

10. A patient who reports chronic itching of the ankles continuously scratches the area. Which assessment finding should the nurse expect? a. Hypertrophied scars on both ankles b. Thickening of the skin around the ankles c. Yellowish-brown skin around both ankles d. Complete absence of melanin in both ankles

ANS: B

10. Unlicensed assistive personnel (UAP) perform the following actions when caring for a patient with Ménière's disease who is experiencing an acute attack. Which action by UAP indicates that the nurse should intervene? a. UAP raises the side rails on the bed. b. UAP turns on the patient's television. c. UAP places an emesis basin at the bedside. d. UAP helps the patient turn to the right side.

ANS: B

10. Which patient statement to the nurse indicates a need for additional instruction in administering insulin? a. "I can buy the 0.5-mL syringes because the line markings are easier to see." b. "I need to rotate injection sites among my arms, legs, and abdomen each day." c. "I do not need to aspirate the plunger to check for blood before injecting insulin." d. "I should draw up the regular insulin first, after injecting air into the NPH bottle."

ANS: B

11. A patient has been newly diagnosed with type 2 diabetes. Which information about the patient will be most useful to the nurse who is helping the patient develop strategies for successful adaptation to this disease? a. Ideal weight b. Value system c. Activity level d. Visual changes

ANS: B

11. An older patient has been diagnosed with possible white coat hypertension. Which planned action by the nurse addresses that suspected cause of the hypertension? a. Instruct the patient about the need to decrease stress levels. b. Teach the patient how to self-monitor and record BPs at home. c. Tell the patient and caregiver that major dietary changes are needed. d. Schedule the patient for regular blood pressure (BP) checks in the clinic.

ANS: B

11. Which patient action indicates accurate understanding of the nurse's teaching about administration of aspart (NovoLog) insulin? a. The patient avoids injecting the insulin into the upper abdominal area. b. The patient cleans the skin with soap and water before insulin administration. c. The patient stores the insulin in the freezer after administering the prescribed dose. d. The patient pushes the plunger down while removing the syringe from the injection site.

ANS: B

12. When assessing a newly admitted patient, the nurse notes a murmur along the left sternal border. To obtain more information about the murmur, which action should the nurse take? a. Palpate the peripheral pulses. b. Determine the timing of the sound. c. Find the point of maximal impulse. d. Compare apical and radial pulse rates.

ANS: B

12. Which action can the nurse working in the emergency department delegate to an experienced unlicensed assistive personnel (UAP)? a. Ask a patient with decreased visual acuity about medications taken at home. b. Perform Snellen testing of visual acuity for a patient with a history of cataracts. c. Obtain information from a patient about any history of childhood ear infections. d. Inspect a patient's external ear for redness, swelling, or presence of skin lesions.

ANS: B

12. Which information about a patient who had a stapedotomy yesterday is most important for the nurse to communicate to the health care provider? a. The patient reports ear "fullness." b. Oral temperature is 100.8° F (38.1° C). c. Small amount of dried drainage on dressing. d. The patient reports that hearing has gotten worse.

ANS: B

13. Which action will the nurse take to evaluate the effectiveness of IV nitroglycerin for a patient with a myocardial infarction (MI)? a. Monitor heart rate. b. Ask about chest pain. c. Check blood pressure. d. Observe for dysrhythmias.

ANS: B

13. Which action will the nurse take when performing ear irrigation for a patient with cerumen impaction? a. Assist the patient to a supine position for the irrigation. b. Fill the irrigation syringe with body-temperature solution. c. Use a sterile applicator to clean the ear canal before irrigating. d. Occlude the ear canal completely with the syringe while irrigating.

ANS: B

14. A registered nurse (RN) is observing a student nurse who is assessing a patient. Which action observed by the RN requires immediate intervention? a. The student nurse presses on the skin over the tibia for 10 seconds to check for edema. b. The student nurse palpates both carotid arteries simultaneously to compare pulse quality. c. The student nurse documents a murmur heard along the right sternal border as a pulmonic murmur. d. The student nurse places the patient in the left lateral position to check for the point of maximal impulse.

ANS: B

15. A patient who has recently had an acute myocardial infarction (AMI) ambulates in the hospital hallway. Which data would indicate to the nurse that the patient should stop and rest? a. O2 saturation drops from 99% to 95%. b. Heart rate increases from 66 to 98 beats/min. c. Respiratory rate goes from 14 to 20 breaths/min. d. Blood pressure (BP) changes from 118/60 to 126/68 mm Hg.

ANS: B

15. Which additional information should the nurse consider when reviewing the laboratory results for a patient's total calcium level? a. The blood glucose b. The serum albumin c. The phosphate level d. The magnesium level

ANS: B

15. Which information will the nurse include when teaching a patient who has type 2 diabetes about glyburide? a. Glyburide decreases glucagon secretion from the pancreas. b. Glyburide stimulates insulin production and release from the pancreas. c. Glyburide should be taken even if the morning blood glucose level is low. d. Glyburide should not be used for 48 hours after receiving IV contrast media.

ANS: B

16. A patient with a history of hypertension treated with a diuretic and an angiotensin-converting enzyme (ACE) inhibitor arrives in the emergency department. The patient reports a severe headache and nausea and has a blood pressure (BP) of 238/118 mm Hg. Which question should the nurse ask to follow up on these findings? a. "Have you recently taken any antihistamines?" b. "Have you consistently taken your medications?" c. "Did you take any acetaminophen (Tylenol) today?" d. "Have there been recent stressful events in your life?"

ANS: B

17. A patient with atopic dermatitis has a new prescription for pimecrolimus (Elidel). After teaching the patient about the medication, which statement by the patient indicates that further teaching is needed? a. "After I apply the medication, I can get dressed as usual." b. "If the medication burns when I apply it, I will wipe it off." c. "I need to minimize time in the sun while using the Elidel." d. "I will rub the medication in gently every morning and night."

ANS: B

17. The nurse is assessing a patient who has been admitted to the intensive care unit (ICU) with a hypertensive emergency. Which finding is most important to report to the health care provider? a. Urine output over 8 hours is 250 mL less than the fluid intake. b. The patient cannot move the left arm and leg when asked to do so. c. Tremors are noted in the fingers when the patient extends the arms. d. The patient reports a headache with pain at level 7 of 10 (0 to 10 scale).

ANS: B

17. What should the nurse teach a patient with repeated hordeolum about how to prevent further infection? a. Apply cold compresses. b. Discard all used eye cosmetics. c. Wash the eyebrows with an antiseborrheic shampoo. d. Be examined for sexually transmitted infections (STIs).

ANS: B

17. When the patient turns his head quickly during the admission assessment, the nurse observes nystagmus. What is the indicated nursing action? a. Assess the patient with a Rinne test. b. Place a fall-risk bracelet on the patient. c. Ask the patient to watch the mouths of staff when they are speaking. d. Remind unlicensed assistive personnel to speak loudly to the patient.

ANS: B

17. Which information about a patient who is scheduled for an oral glucose tolerance test should the nurse consider in interpreting the test results? a. The patient reports having occasional orthostatic dizziness. b. The patient takes oral corticosteroids for rheumatoid arthritis. c. The patient has had a 10 pound weight gain in the last month. d. The patient drank several glasses of water an hour previously.

ANS: B

18. A patient with hypertension who has just started taking atenolol (Tenormin) returns to the health clinic after 2 weeks for a follow-up visit. The blood pressure (BP) is unchanged from the previous visit. Which action should the nurse take first? a. Tell the patient why a change in drug dosage is needed. b. Ask the patient if the medication is being taken as prescribed. c. Review with the patient any lifestyle changes made to help control BP. d. Teach the patient that multiple drugs are often needed to treat hypertension.

ANS: B

18. A registered nurse (RN) is caring for a patient with a goiter and possible hyperthyroidism. Which action by the RN indicates that the charge nurse needs to provide the RN with additional teaching? a. The RN checks the blood pressure in both arms. b. The RN palpates the neck to assess thyroid size. c. The RN orders saline eyedrops to lubricate the patient's bulging eyes. d. The RN lowers the thermostat to decrease the temperature in the room.

ANS: B

19. The nurse is caring for a patient during a water deprivation test. Which finding is most important for the nurse to communicate to the health care provider? a. The patient reports intense thirst. b. The patient has a 5-lb (2.3-kg) weight loss. c. The patient feels dizzy when sitting on the bed. d. The patient's urine osmolality does not increase.

ANS: B

19. The nurse is caring for a patient who is recovering from a sudden cardiac death (SCD) event and has no evidence of an acute myocardial infarction (AMI). What should the nurse anticipate teaching the patient? a. Sudden cardiac death events rarely reoccur. b. Additional diagnostic testing will be required. c. Long-term anticoagulation therapy will be needed. d. Limiting physical activity will prevent future SCD events.

ANS: B

19. When admitting a patient for a cardiac catheterization and coronary angiogram, which information about the patient is important for the nurse to communicate to the health care provider before the test? a. The patient's pedal pulses are +1. b. The patient is allergic to shellfish. c. The patient had a heart attack 1 year ago. d. The patient has not eaten anything today.

ANS: B

19. Which action by the patient who is self-monitoring blood glucose indicates a need for additional teaching? a. Washes the puncture site using warm water and soap. b. Chooses a puncture site in the center of the finger pad. c. Hangs the arm down for a minute before puncturing the site. d. Says the result of 120 mg indicates good blood sugar control.

ANS: B

2. The nurse obtains the following information from a patient newly diagnosed with elevated blood pressure. Which finding is most important to address with the patient? a. Low dietary fiber intake b. No regular physical exercise c. Drinks a beer with dinner every night d. Weight is 5 pounds above ideal weight

ANS: B

2. Which integumentary assessment data from an older patient admitted with bacterial pneumonia should be of concern to the nurse? a. Brown macules on extremities b. Reports a history of allergic rashes c. Skin wrinkled with tenting on both hands d. Longitudinal nail ridges and sparse scalp hair

ANS: B

20. A patient with a possible pituitary adenoma is scheduled for a computed tomography (CT) scan with contrast media. Which patient information is important for the nurse to communicate to the health care provider before the test? a. Bilateral poor peripheral vision b. Allergies to iodine and shellfish c. Recent weight loss of 20 pounds d. Patient reports ongoing headaches

ANS: B

20. The charge nurse observes a new registered nurse (RN) doing discharge teaching for a patient with hypertension who has a new prescription for enalapril (Vasotec). Which statement by the new nurse to the patient would require the charge nurse's intervention? a. "Make an appointment with the dietitian for teaching." b. "Increase your dietary intake of high-potassium foods." c. "Check your blood pressure at home at least once a day." d. "Move slowly when moving from lying to sitting to standing."

ANS: B

20. The nurse is caring for a patient diagnosed with adult inclusion conjunctivitis (AIC) caused by C. trachomatis. Which action should be included in the plan of care? a. Applying topical corticosteroids to decrease inflammation b. Discussing the need for sexually transmitted infection testing c. Educating about the use of antiviral eyedrops to treat the infection d. Assisting with applying for community visual rehabilitation services

ANS: B

21. The nurse and unlicensed assistive personnel (UAP) on the telemetry unit are caring for four patients. Which nursing action can be delegated to the UAP? a. Teaching a patient about exercise electrocardiography b. Attaching ECG monitoring electrodes after a patient bathes c. Monitoring a patient after a transesophageal echocardiogram d. Checking the patient's catheter site after a coronary angiogram

ANS: B

21. The nurse is caring for a patient with a possible pituitary tumor who is scheduled for a computed tomography scan with contrast. Which information about the patient is important to discuss with the health care provider before the test? a. Report of chronic headache b. History of renal insufficiency c. Recent bilateral visual field loss d. Blood glucose level of 134 mg/dL

ANS: B

21. Which assessment finding for a patient receiving IV furosemide (Lasix) to treat stage 2 hypertension is most important to report to the health care provider? a. Blood glucose level of 175 mg/dL b. Serum potassium level of 3.0 mEq/L c. Orthostatic systolic BP decrease of 12 mm Hg d. Current blood pressure (BP) reading of 168/94 mm Hg

ANS: B

21. Which equipment does the nurse need to perform a Rinne test? a. Otoscope b. Tuning fork c. Audiometer d. Ticking watch

ANS: B

21. Which topic will the nurse teach after a patient has had outpatient cataract surgery and lens implantation? a. Use of oral opioids for pain control b. Administration of corticosteroid drops c. Need for bed rest for 1 to 2 days after the surgery d. Importance of coughing and deep breathing exercises

ANS: B

22. The nurse is evaluating the effectiveness of preoperative teaching with a patient scheduled for coronary artery bypass graft (CABG) surgery using the internal mammary artery. Which patient statement indicates that additional teaching is needed? a. "They will circulate my blood with a machine during surgery." b. "I will have incisions in my leg where they will remove the vein." c. "They will use an artery near my heart to go around the area that is blocked." d. "I will need to take aspirin every day after the surgery to keep the graft open."

ANS: B

23. A patient who is recovering from an acute myocardial infarction (AMI) asks the nurse about safely resuming sexual intercourse. Which response by the nurse is best? a. "Most patients are able to enjoy intercourse without any complications." b. "Sexual activity uses about as much energy as climbing two flights of stairs." c. "The doctor will provide sexual guidelines when your heart is strong enough." d. "Holding and cuddling are good ways to maintain intimacy after a heart attack."

ANS: B

23. The nurse is caring for a 70-yr-old patient who takes hydrochlorothiazide and enalapril (Norvasc). The patient's blood pressure (BP) continues to be high. Which patient information may indicate a need for a change? a. Patient takes a daily multivitamin tablet. b. Patient uses ibuprofen to treat osteoarthritis. c. Patient checks BP daily just after getting up. d. Patient drinks wine three to four times a week.

ANS: B

24. A 35-yr-old female patient states that she is using topical fluorouracil to treat actinic keratoses on her face. Which additional assessment information will be most important for the nurse to obtain? a. History of sun exposure by the patient b. Method of contraception used by the patient c. Length of time the patient has used fluorouracil d. Appearance of the treated areas on the patient's face

ANS: B

24. Which action should the nurse take after a patient treated with intramuscular glucagon for hypoglycemia regains consciousness? a. Assess the patient for symptoms of hyperglycemia. b. Give the patient a snack of peanut butter and crackers. c. Have the patient drink a glass of orange juice or nonfat milk. d. Administer a continuous infusion of 5% dextrose for 24 hours.

ANS: B

25. How should the nurse evaluate a patient for improvement after treatment of primary open-angle glaucoma (POAG)? a. Question the patient about blurred vision. b. Note any changes in the patient's visual field. c. Ask the patient to rate the pain using a 0 to 10 scale. d. Assess the patient's depth perception when climbing stairs.

ANS: B

25. The nurse is caring for a patient who was admitted to the coronary care unit following an acute myocardial infarction (AMI) and percutaneous coronary intervention the previous day. What should teaching for this patient include today? a. Typical emotional responses to AMI b. When cardiac rehabilitation will begin c. Pathophysiology of coronary artery disease d. Information regarding discharge medications

ANS: B

39. To improve the physical activity level for a mildly obese 68-year-old patient, which action should the nurse plan to take? a. Stress that weight loss is a major benefit of increased exercise. b. Determine what kind of physical activities the patient usually enjoys. c. Tell the patient that older adults should exercise for no more than 20 minutes at a time. d. Teach the patient to include a short warm-up period at the beginning of physical activity.

ANS: B

26. A patient with glaucoma who has been using timolol (Timoptic) drops for several days tells the nurse that the eyedrops cause eye burning and visual blurriness after administration. The best response to the patient's statement is a. "Those symptoms may indicate a need for a change in dosage of the eyedrops." b. "The drops are uncomfortable, but it is important to use them to retain your vision." c. "These are normal side effects of the drug, which should be less noticeable with time." d. "Notify your health care provider so that different eyedrops can be prescribed for you."

ANS: B

27. Which statement by a patient with bacterial conjunctivitis indicates a need for further teaching? a. "I will wash my hands often during the day." b. "I will remove my contact lenses at bedtime." c. "I will not share towels with my friends or family." d. "I will monitor my family for eye redness or drainage."

ANS: B

28. A patient who has diabetes and reports burning foot pain at night receives a new prescription. Which information should the nurse teach the patient about amitriptyline? a. Amitriptyline decreases the depression caused by your foot pain. b. Amitriptyline helps prevent transmission of pain impulses to the brain. c. Amitriptyline corrects some of the blood vessel changes that cause pain. d. Amitriptyline improves sleep and makes you less aware of nighttime pain.

ANS: B

28. Which assessment finding in a patient who has had coronary artery bypass grafting using a right radial artery graft is most important for the nurse to communicate to the health care provider? a. Complaints of incisional chest pain b. Pallor and weakness of the right hand c. Fine crackles heard at both lung bases d. Redness on both sides of the sternal incision

ANS: B

29. The nurse is caring for a patient who has just arrived on the telemetry unit after having cardiac catheterization. What task should the nurse delegate to a licensed practical/vocational nurse (LPN/VN)? a. Teach the patient about the postprocedure plan of care. b. Give the scheduled aspirin and lipid-lowering medication. c. Perform the initial assessment of the catheter insertion site. d. Titrate the heparin infusion according to the agency protocol.

ANS: B

3. A patient seen in the emergency department for severe headache and acute confusion has a serum sodium level of 118 mEq/L. The nurse should anticipate the need for which diagnostic test? a. Urinary 17-ketosteroids b. Antidiuretic hormone level c. Growth hormone stimulation test d. Adrenocorticotropic hormone level

ANS: B

3. The nurse is admitting a patient who has chest pain. Which assessment data suggest that the pain is from an acute myocardial infarction? a. The pain increases with deep breathing. b. The pain has lasted longer than 30 minutes. c. The pain is relieved after the patient takes nitroglycerin. d. The pain is reproducible when the patient raises the arms.

ANS: B

3. Which assessment finding alerts the nurse to provide patient teaching about cataract development? a. Unequal pupil size b. Sensitivity to light c. Loss of peripheral vision d. History of hyperthyroidism

ANS: B

30. Which action could the registered nurse (RN) who is working in the clinic delegate to a licensed practical/vocational nurse (LPN/VN)? a. Evaluate a patient's ability to administer eyedrops. b. Check a patient's visual acuity using a Snellen chart. c. Inspect a patient's external ear for signs of irritation caused by a hearing aid. d. Teach a patient with otosclerosis about use of sodium fluoride and vitamin D.

ANS: B

31. The occupational health nurse is caring for an employee who reporting bilateral eye pain after a cleaning solution splashed into the employee's eyes. Which action will the nurse take? a. Apply cool compresses. b. Flush the eyes with saline. c. Apply antiseptic ophthalmic ointment to the eyes. d. Cover the eyes with dry sterile patches and shields.

ANS: B

33. The nurse is taking a health history from a 29-yr-old patient at the first prenatal visit. The patient reports that she has no personal history of diabetes, but her mother has diabetes. Which action will the nurse plan to take? a. Teach the patient about administering regular insulin. b. Schedule the patient for a fasting blood glucose level. c. Teach about an increased risk for fetal problems with gestational diabetes. d. Schedule an oral glucose tolerance test for the twenty-fourth week of pregnancy.

ANS: B

34. The charge nurse observes a newly hired nurse performing all the following interventions for a patient who has just undergone right cataract removal and an intraocular lens implant. Which action requires that the charge nurse intervene? a. The nurse leaves the eye shield in place. b. The nurse encourages the patient to cough. c. The nurse elevates the patient's head to 45 degrees. d. The nurse applies corticosteroid drops to the right eye.

ANS: B

35. Which nursing activity is appropriate for the registered nurse (RN) working in the eye clinic to delegate to experienced unlicensed assistive personnel (UAP)? a. Instilling antiviral drops for a patient with a corneal ulcer b. Application of a warm compress to a patient's hordeolum c. Instruction about hand washing for a patient with herpes keratitis d. Checking for eye irritation in a patient with possible conjunctivitis

ANS: B

37. Which prescribed medication should the nurse give first to a patient who has just been admitted to a hospital with acute angle-closure glaucoma? a. Morphine sulfate 4 mg IV b. Mannitol (Osmitrol) 100 mg IV c. Betaxolol (Betoptic) 1 drop in each eye d. Acetazolamide (Diamox) 250 mg orally

ANS: B

38. A 75-yr-old patient who lives alone at home tells the nurse, "I am afraid of losing my independence because my eyes don't work as well they used to." Which action should the nurse take first? a. Discuss the increased risk for falls that is associated with impaired vision. b. Ask the patient about what type of vision problems are being experienced. c. Explain that there are many ways to compensate for decreases in visual acuity. d. Suggest ways of improving the patient's safety, such as using brighter lighting.

ANS: B

38. Which laboratory value reported by the unlicensed assistive personnel (UAP) indicates an urgent need for the nurse to assess the patient? a. Bedtime glucose of 140 mg/dL b. Noon blood glucose of 52 mg/dL c. Fasting blood glucose of 130 mg/dL d. 2-hr postprandial glucose of 220 mg/dL

ANS: B

4. How should the nurse listen to auscultate for S3 or S4 gallops in the mitral area? a. Use the diaphragm of the stethoscope with the patient lying flat. b. Use the bell of the stethoscope with the patient in the left lateral position. c. Use the diaphragm of the stethoscope with the patient in a supine position. d. Use the bell of the stethoscope with the patient sitting and leaning forward.

ANS: B

40. Which patient arriving at the urgent care center will the nurse assess first? a. Patient who is reporting that the left eyelid has just started to droop b. Patient with acute right eye pain that began while using power tools c. Patient with purulent left eye discharge and conjunctival inflammation d. Patient who has redness, crusting, and swelling along the lower right lid margin

ANS: B

40. Which patient at the cardiovascular clinic requires the most immediate action by the nurse? a. Patient with type 2 diabetes whose current blood glucose level is 145 mg/dL. b. Patient with stable angina whose chest pain has recently increased in frequency. c. Patient with familial hypercholesterolemia and a total cholesterol of 465 mg/dL. d. Patient with chronic hypertension whose blood pressure today is 172/98 mm Hg.

ANS: B

42. After reviewing a patient's history, vital signs, physical assessment, and laboratory data, which information shown in the accompanying figure is most important for the nurse to communicate to the health care provider? a. Hyperglycemia b. Bilateral crackles c. Q waves on ECG d. Elevated troponin

ANS: B

43. A few weeks after an 82-yr-old patient with a new diagnosis of type 2 diabetes has been placed on metformin (Glucophage) therapy, the home health nurse makes a visit. Which finding should the nurse promptly discuss with the health care provider? a. Hemoglobin A1C level is 7.9%. b. Glomerular filtration rate is decreased. c. Last eye examination was 18 months ago. d. Patient has questions about the prescribed diet.

ANS: B

47. After change-of-shift report, which patient should the nurse assess first? a. A 19-yr-old patient with type 1 diabetes who has a hemoglobin A1C of 12% b. A 23-yr-old patient with type 1 diabetes who has a blood glucose of 40 mg/dL c. A 50-yr-old patient who uses exenatide (Byetta) and is reporting acute abdominal pain d. A 40-yr-old patient who is pregnant and whose oral glucose tolerance test is 202 mg/dL

ANS: B

5. A patient has the following risk factors for melanoma. Which risk factor should the nurse assign as the priority focus of patient teaching? a. The patient has multiple dysplastic nevi. b. The patient uses a tanning booth weekly. c. The patient is fair-skinned with blue eyes. d. The patient's mother died of malignant melanoma.

ANS: B

5. A patient with type 2 diabetes is scheduled for a follow-up visit in the clinic several months from now. Which test will the nurse schedule to evaluate the effectiveness of treatment for the patient? a. Fasting blood glucose b. Glycosylated hemoglobin c. Oral glucose tolerance test d. Urine dipstick for glucose and ketones

ANS: B

7. A patient is scheduled for a cardiac catheterization with coronary angiography. What information should the nurse provide before the procedure? a. It will be important not to move at all during the procedure. b. A flushed feeling is common when the contrast dye is injected. c. Monitored anesthesia care will be provided during the procedure. d. Arterial pressure monitoring will be needed for 24 hours after the test.

ANS: B

7. A patient who has type 1 diabetes plans to swim laps for an hour daily at 1:00 PM. What advice should the clinic nurse plan to give the patient? a. Increase the morning dose of NPH insulin (Novolin N). b. Check glucose level before, during, and after swimming. c. Time the morning insulin injection to peak while swimming. d. Delay eating the noon meal until after finishing the swimming.

ANS: B

7. During the physical examination, the nurse cannot feel the patient's thyroid gland. What action should the nurse take? a. Palpate the patient's neck more deeply. b. Document that the thyroid was nonpalpable. c. Notify the health care provider immediately. d. Teach the patient about thyroid hormone testing.

ANS: B

7. When performing a skin assessment, the nurse notes angiomas on the chest of an older patient. Which action should the nurse take next? a. Suggest an appointment with a dermatologist. b. Assess the patient for evidence of liver disease. c. Teach the patient about skin changes with aging. d. Discuss the use of sunscreen to prevent skin cancers.

ANS: B

7. Which patient statement indicates that the nurse's teaching about carvedilol (Coreg) for preventing anginal episodes has been effective? a. "Carvedilol will help my heart muscle work harder." b. "It is important not to suddenly stop taking the carvedilol." c. "I can expect to feel short of breath when taking carvedilol." d. "Carvedilol will increase the blood flow to my heart muscle." ANS: B

ANS: B

8. A patient with presbycusis is fitted with binaural hearing aids. Which information will the nurse include when teaching the patient how to use the hearing aids? a. Keep the volume low on the hearing aids for the first week. b. Experiment with volume and hearing in a quiet environment. c. Add the second hearing aid after making adjustments to the first hearing aid. d. Begin wearing the hearing aids for an hour a day, gradually increasing the use.

ANS: B

8. Which information will the nurse provide to the patient scheduled for refractometry? a. "You should not take any of your eye medicines before the examination." b. "You will need to wear sunglasses for a few hours after the examination." c. "The doctor will shine a bright light into your eye during the examination." d. "The surface of your eye will be numb while the doctor does the examination."

ANS: B

8. Which statement by the patient who has newly diagnosed type 1 diabetes indicates a need for additional instruction from the nurse? a. "I will need a bedtime snack because I take an evening dose of NPH insulin." b. "I can choose any foods, as long as I use enough insulin to cover the calories." c. "I can have an occasional beverage with alcohol if I include it in my meal plan." d. "I will eat something at meal times to prevent hypoglycemia, even if I am not hungry."

ANS: B

9. During assessment of the patient's skin, the nurse observes a similar pattern of discrete, small, raised lesions on the left and right upper back areas. Which term should the nurse use to document the distribution of these lesions? a. Confluent b. Symmetric c. Zosteriform d. Generalized

ANS: B

9. The health care provider prescribes topical 5-FU for a patient with actinic keratosis on the left cheek. Which statement should the nurse include in the patient's instructions? a. "5-FU will shrink the lesion to prepare for surgical excision." b. "Your cheek area will be eroded and take several weeks to heal." c. "You may develop nausea and anorexia, but good nutrition is important during treatment." d. "You will need to avoid crowds because of the risk for infection caused by chemotherapy."

ANS: B

9. The nurse has just finished teaching a hypertensive patient about a newly prescribed drug, ramipril (Altace). Which patient statement indicates that more teaching is needed? a. "The medication may not work well if I take aspirin." b. "I can expect some swelling around my lips and face." c. "The doctor may order a blood potassium level occasionally." d. "I will call the doctor if I notice that I have a frequent cough."

ANS: B

The nurse assesses a circular, flat, reddened lesion about 5 cm in diameter on a middle-aged patient's ankle. How should the nurse determine if the lesion is related to intradermal bleeding? a. Elevate the patient's leg. b. Press firmly on the lesion. c. Check the temperature of the skin around the lesion. d. Palpate the dorsalis pedis and posterior tibial pulses.

ANS: B

37. Which assessment finding in a patient who has received omalizumab (Xolair) is most important to report immediately to the health care provider? a. Pain at injection site b. Flushing and dizziness c. Respiratory rate 24 breaths/min\ d. Peak flow reading 75% of normal

ANS: B Flushing and dizziness may indicate that the patient is experiencing an anaphylactic reaction, and immediate intervention is needed. The other information should also be reported, but do not indicate possibly life-threatening complications of omalizumab therapy.

34. The nurse supervises the care of a patient with a temporary radioactive cervical implant. Which action by unlicensed assistive personnel (UAP), if observed by the nurse, would require an intervention? a. The UAP flushes the toilet once after emptying the patient's bedpan. b. The UAP stands by the patient's bed for 30 minutes talking with the patient. c. The UAP places the patient's bedding in the laundry container in the hallway. d. The UAP gives the patient an alcohol-containing mouthwash to use for oral care.

ANS: B Because patients with temporary implants emit radioactivity while the implants are in place, exposure to the patient is limited. Laundry and urine and feces do not have any radioactivity and do not require special precautions. Cervical radiation will not affect the oral mucosa, and alcohol-based mouthwash is not contraindicated

13. Which information is most important for the nurse to include when teaching a patient newly diagnosed with hypertension? a. Most people are able to control BP through dietary changes. b. Annual BP checks are needed to monitor treatment effectiveness. c. Hypertension is usually asymptomatic until target organ damage occurs. d. Increasing physical activity controls blood pressure (BP) for most people.

ANS: C

30. A patient has just been admitted with probable bacterial pneumonia and sepsis. Which order should the nurse implement first? a. Chest x-ray via stretcher b. Blood cultures from two sites c. Ciprofloxacin (Cipro) 400 mg IV d. Acetaminophen (Tylenol) suppositor

ANS: B Initiating antibiotic therapy rapidly is essential, but it is important to obtain the cultures before antibiotic administration. The chest x-ray and acetaminophen administration can be done last

13. Which information will the nurse include when teaching an older patient about skin care? a. Dry the skin thoroughly before applying lotions. b. Bathe and wash hair daily with soap and shampoo. c. Use warm water and a moisturizing soap when bathing. d. Use antibacterial soaps when bathing to avoid infection.

ANS: C

1. The nurse teaches a patient who has chronic bronchitis about a new prescription for combined fluticasone and salmeterol (Advair Diskus). Which patient action indicates to the nurse that teaching about medication administration has been successful? a. The patient shakes the device before use. b. The patient rapidly inhales the medication. c. The patient attaches a spacer to the device. \ d. The patient performs huff coughing after inhalation.

ANS: B The patient should inhale the medication rapidly. Otherwise the dry particles will stick to the tongue and oral mucosa and not get inhaled into the lungs. Advair Diskus is a dry powder inhaler; shaking is not recommended. Spacers are not used with dry powder inhalers. Huff coughing is a technique to move mucus into larger airways to expectorate. The patient should not huff cough or exhale forcefully after taking Advair to keep the medication in the lungs.

14. A patient with ST-segment elevation in three contiguous electrocardiographic leads is admitted to the emergency department and diagnosed as having an ST-segment-elevation myocardial infarction (STEMI). Which question should the nurse ask to determine whether the patient is a candidate for thrombolytic therapy? a. "Do you have any allergies?" b. "Do you take aspirin daily?" c. "What time did your pain begin?" d. "Can you rate the pain on a 0 to 10 scale?"

ANS: C

27. Which action can the registered nurse (RN) who is caring for a critically ill patient with multiple IV lines and medications delegate to a licensed practical/vocational nurse (LPN/VN)? a. Titrate vasoactive IV medications. b. Flush a saline lock with normal saline. c. Remove the patient's central venous catheter. d. Verify blood products prior to administration

ANS: B A LPN/VN has the education, experience, and scope of practice to flush a saline lock with normal saline. Administration of blood products, adjustment of vasoactive infusion rates, and removal of central catheters in critically ill patients require RN level education and scope of practice.

9. A patient with a large stomach tumor attached to the liver is scheduled for a debulking procedure. What should the nurse teach the patient about the outcome of this procedure? a. Pain will be relieved by cutting sensory nerves in the stomach. b. Decreasing the tumor size will improve the effects of other therapy. c. Relieving the pressure in the stomach will promote optimal nutrition. d. Tumor growth will be controlled by removing all the cancerous tissue

ANS: B A debulking surgery reduces the size of the tumor and makes radiation and chemotherapy more effective. Debulking surgeries do not control tumor growth. The tumor is debulked because it is attached to the liver, a vital organ (not to relieve pressure on the stomach). Debulking does not sever the sensory nerves, although pain may be lessened by the reduction in pressure on the abdominal organs

15. The nurse supervises a student nurse who is assigned to take care of a patient with active tuberculosis (TB). Which action, if performed by the student nurse, would require an intervention by the nurse? a. The patient is offered a tissue from the box at the bedside. b. A surgical face mask is applied before visiting the patient. c. A snack is brought to the patient from the unit refrigerator. d. Hand washing is performed before entering the patient's room

ANS: B A high-efficiency particulate-absorbing (HEPA) mask, rather than a standard surgical mask, should be used when entering the patient's room because the HEPA mask can filter out 100% of small airborne particles. Hand washing before entering the patient's room is appropriate. Because anorexia and weight loss are frequent problems in patients with TB, bringing food to the patient is appropriate. The student nurse should perform hand washing after handling a tissue that the patient has used, but no precautions are necessary when giving the patient an unused tissue.

21. The registered nurse (RN) is caring for a patient who is living with HIV and admitted with tuberculosis. Which task can the RN delegate to unlicensed assistive personnel (UAP)? a. Teach the patient how to dispose of tissues with respiratory secretions. b. Stock the patient's room with the necessary personal protective equipment. c. Interview the patient to obtain the names of family members and close contacts. d. Tell the patient's family members the reason for the use of airborne precautions.

ANS: B A patient diagnosed with tuberculosis would be placed on airborne precautions. Because all health care workers are taught about the various types of infection precautions used in the hospital, the UAP can safely stock the room with personal protective equipment. Obtaining contact information and patient teaching are higher-level skills that require RN education and scope of practice.

14. What is the most effective method to prevent the spread of infection to others when the nurse is changing the dressing over a wound infected with Staphylococcus aureus? a. Change the dressing using sterile gloves. b. Apply antibiotic ointment over the wound. c. Wash hands and properly dispose of soiled dressings. d. Soak the dressing in sterile normal saline before removal.

ANS: C

42. The nurse receives a change-of-shift report on the following patients with chronic obstructive pulmonary disease (COPD). Which patient should the nurse assess first? a. A patient with loud expiratory wheezes b. A patient with a respiratory rate of 38 breaths/min c. A patient who has a cough productive of thick, green mucus d. A patient with jugular venous distention and peripheral edema

ANS: B A respiratory rate of 38/min indicates severe respiratory distress, and the patient needs immediate assessment and intervention to prevent possible respiratory arrest. The other patients also need assessment as soon as possible, but they do not need to be assessed as urgently as the patient with tachypnea

17. A patient is admitted to the hospital with acute rejection of a kidney transplant. Which intervention will the nurse expect for this patient? a. Testing for human leukocyte antigen (HLA) match b. Administration of immunosuppressant medications c. Insertion of an arteriovenous graft for hemodialysis d. Placement of the patient on the transplant waiting

ANS: B Acute rejection is treated with the administration of additional immunosuppressant drugs such as corticosteroids. Because acute rejection is potentially reversible, there is no sign that the patient will need another transplant or hemodialysis. There is no need to repeat HLA testing.

25. A patient in the clinic with cystic fibrosis (CF) reports increased sweating and weakness during the summer months. Which action by the nurse would be most appropriate? a. Teach the patient signs of hypoglycemia. b. Have the patient add dietary salt to meals. c. Suggest decreasing intake of dietary fat and calories. d. Teach the patient about pancreatic enzyme replacement

ANS: B Added dietary salt is indicated whenever sweating is excessive, such as during hot weather, when fever is present, or from intense physical activity. The management of pancreatic insufficiency includes pancreatic enzyme replacement before each meal and snack. This patient is at risk for hyponatremia based on reported symptoms. Adequate intake of fat, calories, protein, and vitamins is important. Fat-soluble vitamins (vitamins A, D, E, and K) must be supplemented because they are malabsorbed. Use of caloric supplements improves nutritional status. Hyperglycemia caused by pancreatic insufficiency is more likely to occur than hypoglycemia

20. Which information is most important for the nurse to report to the health care provider about a patient who has been using varenicline (Chantix)? a. The patient reports headaches that occur almost daily. b. The patient reports new-onset sadness and depression. c. The patient continues to smoke a few cigarettes every day. d. The patient says, "I have decided that I am not ready to quit."

ANS: B Adverse effects of varenicline include depression and attempted suicide, which require immediate assessment and discontinuation of the drug. The other information will also be reported, but it does not indicate a life-threatening problem associated with the medication

27. A patient with chronic obstructive pulmonary disease (COPD) has coarse crackles throughout the lung fields and a chronic, nonproductive cough. Which nursing intervention will be most effective? a. Increase the O2 flow rate to the highest prescribed rate. b. Teach the patient to use a Flutter airway clearance device. c. Reinforce the ongoing use of pursed-lip breathing techniques. d. Teach the patient about consistent use of inhaled corticosteroids.

ANS: B Airway clearance devices assist with moving mucus into larger airways, where it can more easily be expectorated. The other actions may be appropriate for some patients with COPD, but they are not specific to this patient's problem of thick mucus secretions.

41. The nurse reviews the medication administration record (MAR) for a patient having an acute asthma attack. Which medication should the nurse administer first? a. Methylprednisolone (Solu-Medrol) 60 mg IV b. Albuterol (Ventolin HFA) 2.5 mg per nebulizer c. Salmeterol (Serevent) 50 mcg per dry-powder inhaler (DPI) d. Ipratropium (Atrovent) 2 puffs per metered-dose inhaler (MDI)

ANS: B Albuterol is a rapidly acting bronchodilator and is the first-line medication to reverse airway narrowing in acute asthma attacks. The other medications work more slowly

11. The nurse provides discharge instructions for a patient after a total laryngectomy. Which statement by the patient indicates that additional instruction is needed? a. "I can participate in fitness activities except swimming." b. "I must keep the stoma covered with an occlusive dressing." c. "I need to have smoke and carbon monoxide detectors installed." d. "I will wear a Medic-Alert bracelet to identify me as a neck breather."

ANS: B An occlusive dressing will completely block the patient's airway. The stoma may be covered with clothing or a loose dressing, but this is not essential. The other patient comments are all accurate and indicate that the teaching has been effective.

21. Immediately after the nurse administers an intradermal injection of an allergen on the forearm, the patient reports itching at the site, weakness, and dizziness. What action should the nurse take first? a. Apply antiinflammatory cream. b. Place a tourniquet above the site. c. Administer subcutaneous epinephrine. d. Reschedule the patient's other allergen tests.

ANS: B Application of a tourniquet will decrease systemic circulation of the allergen and should be the first reaction. The other actions may occur, but the tourniquet application slows the allergen progress into the patient's system, allowing treatment of the anaphylactic response. A local antiinflammatory cream may be applied to the site of a cutaneous test for persistent itching. Epinephrine will be needed if the allergic reaction progresses to anaphylaxis. Other testing may be delayed and rescheduled after development of anaphylaxis.

1. Which question asked by the nurse will give the most information about the patient's metastatic bone cancer pain? a. "How long have you had this pain?" b. "How would you describe your pain?" c. "How often do you take pain medication?" d. "How much medication do you take for the pain?

ANS: B Because pain is a multidimensional experience, asking a question that addresses the patient's experience with the pain will elicit more information than the more specific information asked in the other three responses. All these questions are appropriate, but the response beginning "How would you describe your pain?" is the best initial question.

15. What should the nurse teach a patient with recurrent staphylococcal and seborrheic blepharitis to do? a. Irrigate the eyes with saline solution. b. Schedule an appointment for eye surgery. c. Use a gentle baby shampoo to clean the eyelids. d. Apply cool compresses to the eyes three times daily.

ANS: C

15. Which action will the nurse implement for a patient who arrives for a calcium-scoring CT scan? a. Insert an IV catheter. b. Administer oral sedative medications. c. Teach the patient about the procedure. d. Confirm that the patient has been fasting.

ANS: C

5. A patient who collects honey to earn supplemental income has developed a hypersensitivity to bee stings. Which statement by the patient would indicate a need for additional teaching? a. "I need to find a different way to earn extra money." b. "I will take oral antihistamines before going to work." c. "I can get a prescription for epinephrine and learn to self-inject it." d. "I should wear a Medic-Alert bracelet indicating my allergy to bee stings."

ANS: B Because the patient is at risk for bee stings and the severity of allergic reactions tends to increase with added exposure to allergen, taking oral antihistamines will not adequately control the patient's hypersensitivity reaction. The other patient statements indicate a good understanding of management of the problem

34. A patient who comes to the clinic reports frequent, watery stools for 2 days. Which action should the nurse take first? a. Obtain the baseline weight. b. Check the patient's blood pressure. c. Draw blood for serum electrolyte levels. d. Ask about extremity numbness or tingling.

ANS: B Because the patient's history suggests that fluid volume deficit may be a problem, assessment for adequate circulation is the highest priority. The other actions are also appropriate but are not as essential as determining the patient's perfusion stat

30. The nurse assesses a patient who is receiving interleukin-2. Which finding should the nurse report immediately to the health care provider? a. Generalized muscle aches b. Crackles at the lung bases c. Reports of nausea and anorexia d. Oral temperature of 100.6° F (38.1° C)

ANS: B Capillary leak syndrome and acute pulmonary edema are possible toxic effects of interleukin-2. The patient may need oxygen and the nurse should rapidly notify the health care provider. The other findings are common side effects of interleukin-2.

15. After receiving a change-of-shift report, which patient should the nurse assess first? a. The patient who has multiple leg wounds with eschar to be debrided. b. The patient receiving chemotherapy who has a temperature of 102° F. c. The patient who requires analgesics before a scheduled dressing change. d. The newly admitted patient with a stage 4 pressure injury on the coccyx.

ANS: B Chemotherapy is an immunosuppressant. Even a low fever in an immunosuppressed patient is a sign of serious infection and should be treated immediately with cultures and rapid initiation of antibiotic therapy. The nurse should assess the other patients as soon as possible after assessing and implementing appropriate care for the immunosuppressed patient.

2. Which example should the nurse use to explain an infant's "passive immunity" to a new mother? a. Vaccinations b. Breastfeeding c. Stem cells in peripheral blood d. Exposure to communicable diseases

ANS: B Colostrum in breast milk provides passive immunity through antibodies from the mother. These antibodies protect the infant for a few months. However, memory cells are not retained, so the protection is not permanent. Active immunity is acquired by being immunized with vaccinations or having an infection. Stem cells are unspecialized cells used to repopulate a person's bone marrow after high-dose chemotherapy.

24. A patient who has a right-sided chest tube after a thoracotomy has continuous bubbling in the suction-control chamber of the collection device. Which action by the nurse is appropriate? a. Adjust the dial on the wall regulator. b. Continue to monitor the collection device. c. Document the presence of a large air leak. d. Notify the surgeon of a possible pneumothorax.

ANS: B Continuous bubbling is expected in the suction-control chamber and indicates that the suction-control chamber is connected to suction. An air leak would be detected in the water-seal chamber. There is no evidence of pneumothorax. Increasing or decreasing the vacuum source will not adjust the suction pressure. The amount of suction applied is regulated by the amount of water in this chamber and not by the amount of suction applied to the system.

32. A patient is receiving a 3% saline continuous IV infusion for hyponatremia. Which assessment data will require the most rapid response by the nurse? a. The patient's radial pulse is 105 beats/min. b. There are crackles throughout both lung fields. c. There is sediment and blood in the patient's urine. d. The patient's blood pressure increases to 142/94 mm H

ANS: B Crackles throughout both lungs suggest that the patient may be experiencing pulmonary edema, a life-threatening adverse effect of hypertonic solutions. The increased pulse rate and blood pressure and the appearance of the urine should also be reported, but they are not as dangerous as the presence of fluid in the alveoli.

15. The nurse teaches a patient about drug therapy after a kidney transplant. Which statement by the patient would indicate a need for further instructions? a. "I need to be monitored closely for development of cancer." b. "After a couple of years, I will be able to stop taking the cyclosporine." c. "If I develop acute rejection episode, I will need additional types of drugs." d. "The drugs are combined to inhibit different ways the kidney can be rejected."

ANS: B Cyclosporine, a calcineurin inhibitor, will need to be continued for life. The other patient statements are accurate and indicate that no further teaching is necessary about those topics.

3. A patient with multiple draining wounds is admitted for hypovolemia. What would be the most accurate way for the nurse to evaluate fluid balance? a. Skin turgor b. Daily weight c. Urine output d. Edema presence

ANS: B Daily weight is the most easily obtained and accurate means of assessing volume status. Skin turgor varies considerably with age. Considerable excess fluid volume may be present before fluid moves into the interstitial space and causes edema. Urine outputs do not take account of fluid intake or of fluid loss through insensible loss, sweating, or loss from the gastrointestinal tract or wounds.

23. The home health nurse is caring for a patient who has been receiving interferon therapy for treatment of cancer. Which statement by the patient indicates a need for further assessment? a. "I have frequent muscle aches and pains." b. "I rarely have the energy to get out of bed." c. "I experience chills after I inject the interferon." d. "I take acetaminophen (Tylenol) every 4 hours."

ANS: B Fatigue can be a dose-limiting toxicity for use of immunotherapy. Flu-like symptoms, such as muscle aches and chills, are common side effects with interferon use. Patients are advised to use acetaminophen every 4 hours.

16. A patient is admitted with tetany. Which laboratory value should the nurse plan to monitor? a. Total protein b. Blood glucose c. Ionized calcium d. Serum phosphate

ANS: C

17. A patient who has fibromyalgia reports pain at level 7 (0 to 10 scale). The patient tells the nurse, "I feel depressed because I ache too much to play golf." Which patient goal has the highest priority when the nurse is developing the treatment plan? a. The patient will report pain at a level 2 of 10. b. The patient will be able to play a round of golf. c. The patient will exhibit fewer signs of depression. d. The patient will say that the aching has decreased.

ANS: B For chronic pain, patients are encouraged to set functional goals such as being able to perform daily activities and hobbies. The patient has identified playing golf as the desired activity, so a pain level of 2 of 10 or a decrease in aching would be less useful in evaluating successful treatment. The nurse should also assess for depression, but the patient has identified the depression as being due to the inability to play golf, so the goal of being able to play golf is the most appropriate.

28. A hospitalized patient who has received chemotherapy for leukemia develops neutropenia. Which observation by the nurse indicates a need for further teaching? a. The patient ambulates around the room. b. The patient's visitors bring in fresh peaches. c. The patient cleans with a warm washcloth after having a stool. d. The patient uses soap and shampoo to shower every other day

ANS: B Fresh, thinned-skin fruits are not permitted in a neutropenic diet because of the risk of bacteria being present. The patient should ambulate in the room rather than the hospital hallway to avoid exposure to other patients or visitors. Because overuse of soap can dry the skin and increase infection risk, showering every other day is acceptable. Careful cleaning after having a bowel movement will help prevent skin breakdown and infection.

45. An experienced nurse instructs a new nurse about how to care for a patient with dyspnea caused by a pulmonary fungal infection. Which action by the new nurse indicates a need for further teaching? a. Listening to the patient's lung sounds several times during the shift b. Placing the patient on droplet precautions in a private hospital room c. Monitoring patient serology results to identify the infecting organism d. Titrating the O2 flowrate as prescribed to keep the O2 saturation over 90%

ANS: B Fungal infections are not transmitted from person to person. Therefore, no isolation procedures are necessary. The other actions by the new nurse are appropriate

28. The nurse provides dietary teaching for a patient with chronic obstructive pulmonary disease (COPD) who has a low body mass index (BMI). Which patient statement indicates that the teaching has been effective? a. "I will drink lots of fluids with my meals." b. "I can have ice cream as a snack every day." c. "I will exercise for 15 minutes before meals." d. "I will decrease my intake of beef and poultry."

ANS: B High-calorie foods such as ice cream are an appropriate snack for patients with COPD. Fluid intake of 3 L/day is recommended, but fluids should be taken between meals rather than with meals to improve oral intake of solid foods. The patient should avoid exercise for an hour before meals to prevent fatigue while eating. Meat and dairy products are high in protein and are good choices for the patient with COPD

2. A patient who has a small cell cancer of the lung develops syndrome of inappropriate antidiuretic hormone (SIADH). The nurse should notify the health care provider about which assessment finding? a. Serum hematocrit of 42% b. Serum sodium of 120 mg/dL c. Urinary output of 280 mL in 8 hours d. Reported weight gain of 2.2 pounds (1 kg)

ANS: B Hyponatremia is the most important finding to report. SIADH causes water retention and a decrease in serum sodium level. Hyponatremia can cause confusion and other central nervous system effects. A critically low value needs to be treated. At least 30 mL/hr of urine output indicates adequate kidney function. The hematocrit level is normal. Weight gain is expected with SIADH because of water retention.

8. IV potassium chloride (KCl) 60 mEq is prescribed for a patient with severe hypokalemia. Which action should the nurse take? a. Administer the KCl as a rapid IV bolus. b. Infuse the KCl at a maximum rate of 10 mEq/hr. c. Discontinue cardiac monitoring during the infusion. d. Refuse to give the KCl through a peripheral venous lin

ANS: B IV KCl is administered at a maximal rate of 10 mEq/hr. Rapid IV infusion of KCl can cause cardiac arrest. KCl can cause inflammation of peripheral veins, but it can be administered by this route. Cardiac monitoring should be continued while patient is receiving potassium because of the risk for dysrhythmias.

11. During physical assessment of a patient who has frequent nosebleeds, the nurse finds nasal sores and necrosis of the nasal septum. The nurse should ask the patient specifically about the use of which drug? a. Heroin b. Cocaine c. Tobacco d. Marijuana

ANS: B Inhaled cocaine causes ischemia of the nasal septum, leading to nasal sores and necrosis. These symptoms are not associated with the use of heroin, tobacco, or marijuana.

3. A patient with diabetes has arterial blood gas (ABG) results pH 7.28; PaCO2 34 mm Hg; PaO2 85 mm Hg; HCO3 ?2- 18 mEq/L. The nurse would expect which finding? a. Intercostal retractions b. Kussmaul respirations c. Low oxygen saturation (SpO2) d. Decreased venous O2 pressure

ANS: B Kussmaul (deep and rapid) respirations are a compensatory mechanism for metabolic acidosis. The low pH and low bicarbonate result indicate metabolic acidosis. Acidosis does not cause intercostal retractions, a low oxygen saturation rate, and a decrease in venous O2 pressure.

22. A clinic patient reports experiencing an allergic reaction to an unknown allergen several weeks ago. Which action is appropriate for the registered nurse (RN) to delegate to a licensed practical/vocational nurse (LPN/VN)? a. Perform a focused physical assessment. b. Administer a cutaneous scratch skin test. c. Obtain the health history from the patient. d. Review diagnostic study results with the patient

ANS: B LPN/VNs are educated and licensed to administer medications under the supervision of an RN. RN-level education and the scope of practice include assessment of health history, focused physical assessment, and teaching about study results

22. Which action is appropriate for the nurse to delegate to unlicensed assistive personnel (UAP)? a. Listen to a patient's lung sounds for wheezes or crackles. b. Label specimens obtained during percutaneous lung biopsy. c. Instruct a patient about how to use home spirometry testing. d. Measure induration at the site of a patient's intradermal skin test.

ANS: B Labeling of specimens at the bedside during a procedure is within the scope of practice of UAP. The other actions require nursing judgment and should be done by licensed nursing personnel

10. The nurse observes a student who is listening to a patient's lungs. Which action by the student indicates a need to review respiratory assessment skills? a. The student compares breath sounds from side to side at each level. b. The student listens during the inspiratory phase, then moves the stethoscope. c. The student starts at the apices of the lungs, moving down toward the lung bases. d. The student instructs the patient to breathe slowly and deeply through the mouth.

ANS: B Listening only during inspiration indicates the student needs a review of respiratory assessment skills. At each placement of the stethoscope, listen to at least one cycle of inspiration and expiration. During chest auscultation, instruct the patient to breathe slowly and a little deeper than normal through the mouth. Auscultation should proceed from the lung apices to the bases, comparing opposite areas of the chest, unless the patient is in respiratory distress or will tire easily

11. The nurse will perform which action for a wet-to-dry dressing change on a patient's stage 3 sacral pressure injury? a. Pour sterile saline onto the new dry dressings after packing the wound. b. Administer a prescribed PRN oral analgesic 30 minutes before the change. c. Apply antimicrobial ointment before repacking the wound with moist dressings. d. Soak the old dressings with sterile saline 30 minutes before the dressing change

ANS: B Mechanical debridement with wet-to-dry dressings is painful, and patients should receive pain medications before the dressing change begins. The new dressings are moistened with saline before being applied to the wound but not soaked after packing. Soaking the old dressings before removing them will eliminate the wound debridement that is the purpose of this type of dressing. Application of antimicrobial ointments is not indicated for a wet-to-dry dressing.

8. A patient who smokes a pack of cigarettes daily develops tachycardia and irritability on the second day after abdominal surgery. What action should the nurse take? a. Escort the patient outside where smoking is allowed. b. Request a prescription for a nicotine replacement agent. c. Tell the patient to calm down and not to think about smoking. d. Ask the patient's family to bring in chewable tobacco products.

ANS: B Nicotine replacement agents should be prescribed for patients who smoke and are hospitalized to avoid withdrawal symptoms. Allowing the patient to smoke or use other tobacco products encourages ongoing tobacco use. Telling the patient to calm down will not relieve withdrawal symptoms.

20. Which nursing action could the nurse delegate to unlicensed assistive personnel (UAP) when caring for a patient who is using a fentanyl (Duragesic) patch and a heating pad for treatment of chronic back pain? a. Check the skin under the heating pad. b. Count the respiratory rate every 2 hours. c. Ask the patient whether pain control is effective. d. Monitor sedation using the sedation assessment scale.

ANS: B Obtaining the respiratory rate is included in UAP education and scope of practice. Assessment for sedation, pain control, and skin integrity requires more education and scope of practice.

5. A pregnant woman with asymptomatic chronic human immunodeficiency virus (HIV) infection is seen at the clinic. The patient states, "I am very nervous about making my baby sick." Which information will the nurse include when teaching the patient? a. The antiretroviral medications used to treat HIV infection are teratogenic. b. Most infants born to HIV-positive mothers are not infected with the virus. c. Because it is an early stage of HIV infection, the infant will not contract HIV. d. Her newborn will be born with HIV unless she uses antiretroviral therapy (ART)

ANS: B Only 25% of infants born to HIV-positive mothers develop HIV infection, even when the mother does not use ART during pregnancy. The percentage drops to 2% when ART is used. Perinatal transmission can occur at any stage of HIV infection (although it is less likely to occur when the viral load is lower). ART can safely be used in pregnancy, although some ART drugs should be avoided

8. A patient with a deep partial thickness burn has been receiving hydromorphone through patient-controlled analgesia (PCA) for 1 week. The nurse caring for the patient during the previous shift reports that the patient wakes up frequently during the night reporting pain. What action by the nurse is appropriate? a. Administer a dose of morphine every 1 to 2 hours from the PCA machine while the patient is sleeping. b. Consult with the health care provider about using a different treatment protocol to control the patient's pain. c. Request that the health care provider order a bolus dose of morphine to be given when the patient awakens with pain. d. Teach the patient to push the button every 10 minutes for an hour before going to sleep, even if the pain is minimal.

ANS: B PCAs are best for controlling acute pain. This patient's history indicates a need for a pain management plan that will provide adequate analgesia while the patient is sleeping. Administering a dose of morphine when the patient already has severe pain will not address the problem. Teaching the patient to administer unneeded medication before going to sleep can result in oversedation and respiratory depression. It is illegal for the nurse to administer the morphine for a patient through PCA.

37. The nurse receives change-of-shift report on the following four patients. Which patient should the nurse assess first? a. A 77-yr-old patient with tuberculosis (TB) who has four medications due b. A 46-yr-old patient on bed rest who reports sudden onset of shortness of breath c. A 35-yr-old patient with pneumonia who has a temperature of 100.2° F (37.8° C) d. A 23-yr-old patient with cystic fibrosis who has pulmonary function testing scheduled

ANS: B Patients on bed rest who are immobile are at high risk for deep vein thrombosis (DVT). Sudden onset of shortness of breath in a patient with a DVT suggests a pulmonary embolism and requires immediate assessment and action such as O2 administration. The other patients should be assessed as soon as possible, but there is no indication that they may need immediate action to prevent clinical deterioration.

4. The nurse provides discharge instructions to a patient who was hospitalized for pneumonia. Which statement by the patient indicates a good understanding of the instructions? a. "I will call my health care provider if I still feel tired after a week." b. "I will continue to do deep breathing and coughing exercises at home." c. "I will schedule two appointments for the pneumonia and influenza vaccines." d. "I will cancel my follow-up chest x-ray appointment if I feel better next week.

ANS: B Patients should continue to cough and deep breathe after discharge. Fatigue is expected for several weeks. The pneumococcal and influenza vaccines can be given at the same time in different arms. A follow-up chest x-ray needs to be done in 6 to 8 weeks to evaluate resolution of pneumonia.

8. Which patient would benefit from education about HIV preexposure prophylaxis (PrEP)? a. A 23-yr-old woman living with HIV infection. b. A 52-yr-old recently single woman just diagnosed with chlamydia. c. A 33-yr-old hospice worker who received a needle stick injury 3 hours ago. d. A 60-yr-old male in a monogamous relationship with an HIV-uninfected partner.

ANS: B Preexposure prophylaxis (PrEP) is used to prevent HIV infection. Persons who would be good candidates for PrEP include individuals with a recent diagnosis of an STI and those with more than one partner. Individuals who are not on PrEP but who have a recent high-risk exposure (such as a needle stick) would be better candidates for postexposure prophylaxis (PEP). A person in a monogamous relationship with an HIV-uninfected partner is considered low-risk for HIV infection.

4. The nurse should suggest which food choice for a patient scheduled to receive external-beam radiation for abdominal cancer? a. Fruit salad b. Baked chicken c. Creamed broccoli d. Toasted wheat bread

ANS: B Protein is needed for wound healing. To minimize the diarrhea that is associated with bowel radiation, the patient should avoid foods high in roughage, such as fruits and whole grains. Lactose intolerance may develop secondary to radiation, so dairy products should also be avoided

10. The home health nurse is visiting a patient with chronic obstructive pulmonary disease (COPD). Which nursing action is appropriate to implement for a patient who has an impaired breathing pattern due to anxiety? a. Titrate O2 to keep saturation at least 90%. b. Teach the patient how to use the pursed-lip technique. c. Discuss a high-protein, high-calorie diet with the patient. d. Suggest the use of over-the-counter sedative medications

ANS: B Pursed-lip breathing techniques assist in prolonging the expiratory phase of respiration and decrease air trapping. There is no indication that the patient requires O2 therapy or an improved diet. Sedative medications should be avoided because they decrease respiratory drive.

19. A patient in the health care provider's office for allergen testing using the cutaneous scratch method develops itching and swelling at the skin site. Which action should the nurse plan to take first? a. Monitor the patient's edema. b. Administer a dose of epinephrine. c. Obtain a prescription for oral antihistamines. d. Assess the patient's use of new skin products.

ANS: B Rapid administration of epinephrine when excessive itching or swelling at the skin site is observed can prevent the progression to anaphylaxis. The initial symptoms of anaphylaxis are itching and edema at the site of the exposure. The nurse should not wait and assess for development of more edema. Hypotension, tachycardia, dilated pupils, and wheezes occur later. Exposure to skin products does not address the immediate concern of a possible anaphylactic reaction

3. A patient is being evaluated for possible atopic dermatitis. The nurse should expect elevation of which laboratory value? a. IgA b. IgE c. Basophils d. Neutrophils

ANS: B Serum IgE is elevated in an allergic response (type 1 hypersensitivity disorders). The eosinophil level will be elevated rather than neutrophil or basophil counts. IgA is in body secretions and would not be tested when evaluating a patient who has symptoms of atopic dermatitis.

12. Which patient should the nurse assess first? a. Patient with urticaria after receiving an IV antibiotic b. Patient who is sneezing after subcutaneous immunotherapy c. Patient who has graft-versus-host disease and severe diarrhea d. Patient with multiple chemical sensitivities with muscle stiffness

ANS: B Sneezing after subcutaneous immunotherapy may indicate impending anaphylaxis and assessment and emergency measures should be initiated. The other patients also have findings that need assessment and intervention by the nurse, but do not have evidence of life-threatening complications.

11. Which statement by a patient would alert the nurse to a risk for decreased immune function? a. "I had a chest x-ray 6 months ago." b. "I had my spleen removed after a car accident." c. "I take one baby aspirin every day to prevent stroke." d. "I usually eat eggs or meat for at least two meals a day."

ANS: B Splenectomy increases the risk for septicemia from bacterial infections. The patient's protein intake is good and should improve immune function. Daily aspirin use does not affect immune function. A chest x-ray does not have enough radiation to suppress immune function

17. Which nursing action could the registered nurse (RN) working in a skilled care hospital unit delegate to an experienced licensed practical/vocational nurse (LPN/VN) caring for a patient with a permanent tracheostomy? a. Assess the patient's risk for aspiration. b. Suction the tracheostomy when directed. c. Teach the patient to provide tracheostomy self-care. d. Determine the need for tracheostomy tube replacement.

ANS: B Suctioning of a stable patient can be delegated to LPNs/VNs. The RN should perform patient assessment and patient teaching

15. The nurse administers an IV vesicant chemotherapeutic agent to a patient. Which action is most important for the nurse to take? a. Infuse the medication over a short period of time. b. Stop the infusion if swelling is observed at the site. c. Administer the chemotherapy through a small-bore catheter. d. Hold the medication unless a central venous line is available

ANS: B Swelling at the site may indicate extravasation, and the IV should be stopped immediately. The medication generally should be given slowly to avoid irritation of the vein. The size of the catheter is not as important as administration of vesicants into a running IV line to allow dilution of the chemotherapy drug. These medications can be given through peripheral lines, although central vascular access devices are preferred

9. A patient who is admitted to the hospital for wound debridement admits to using fentanyl (Sublimaze) illegally. What withdrawal signs should the nurse expect? a. Tremors and seizures b. Vomiting and diarrhea c. Lethargy and disorientation d. Delusions and hallucinations

ANS: B Symptoms of opioid withdrawal include gastrointestinal symptoms such as nausea, vomiting, and diarrhea. The other symptoms are seen during withdrawal from other substances such as alcohol, sedative-hypnotics, or stimulant

1. The nurse assesses a patient's surgical wound on the first postoperative day and notes redness and warmth around the incision. Which action by the nurse is appropriate? a. Obtain wound cultures. b. Document the assessment. c. Notify the health care provider. d. Assess the wound every 2 hours.

ANS: B The incisional redness and warmth are indicators of the normal initial (inflammatory) stage of wound healing by primary intention. The nurse should document the wound appearance and continue to monitor the wound. Notification of the health care provider, assessment every 2 hours, and obtaining wound cultures are not indicated because the healing is progressing normally.

20. A student asks the nurse why a peripherally inserted central catheter is needed for a patient receiving parenteral nutrition with 25% dextrose. Which response by the nurse is accurate? a. "The prescribed infusion can be given more rapidly when the patient has a central line." b. "The hypertonic solution will be more rapidly diluted when given through a central line." c. "There is a decreased risk for infection when 25% dextrose is infused through a central line." d. "The required blood glucose monitoring is based on samples obtained from a central line."

ANS: B The 25% dextrose solution is hypertonic. Shrinkage of red blood cells can occur when solutions with dextrose concentrations greater than 10% are administered IV. Blood glucose testing is not more accurate when samples are obtained from a central line. The infection risk is higher with a central catheter than with peripheral IV lines. Hypertonic or concentrated IV solutions are not given rapidly.

18. A patient is receiving 35% O2 via a Venturi mask. Which action by the nurse will help ensure the correct dosage of O2? a. Teach the patient to keep the mask on during meals. b. Keep the air entrainment ports clean and unobstructed. c. Use a high enough flowrate to keep the bag from collapsing. d. Drain moisture condensation from the corrugated tubing hourly

ANS: B The air entrainment ports regulate the O2 percentage delivered to the patient, so they must be unobstructed. The other options refer to other types of O2 devices. A high O2 flow rate is needed when giving O2 by partial rebreather or nonrebreather masks. Draining O2 tubing is necessary when caring for a patient receiving mechanical ventilation. The mask can be changed to a nasal cannula at a prescribed setting when the patient eats

14. Which assessment finding in a patient with impaired gas exchange is most useful in evaluating the effectiveness of treatment? a. Even, unlabored respirations b. Pulse oximetry reading of 92% c. Absence of wheezes or crackles d. Respiratory rate of 18 breaths/mi

ANS: B The best data for evaluation of gas exchange are arterial blood gases (ABGs) or pulse oximetry. The other data may indicate either improvement or impending respiratory failure caused by fatigue

16. The nurse is admitting a patient diagnosed with an acute exacerbation of chronic obstructive pulmonary disease (COPD). How should the nurse determine the appropriate O2 flowrate? a. Minimize O2 use to avoid O2 dependency. b. Maintain the pulse oximetry level at 90% or greater. c. Administer O2 according to the patient's level of dyspnea. d. Avoid administration of O2 at a rate of more than 2 L/min

ANS: B The best way to determine the appropriate O2 flowrate is by monitoring the patient's oxygenation either by arterial blood gases (ABGs) or pulse oximetry. An O2 saturation of 90% indicates adequate blood O2 level without the danger of suppressing the respiratory drive. For patients with an exacerbation of COPD, an O2 flowrate of 2 L/min may not be adequate. Because O2 use improves survival rate in patients with COPD, there is no concern about O2 dependency. The patient's perceived dyspnea level may be affected by other factors (e.g., anxiety) besides blood O2 level.

1. The nurse is caring for a patient who has a massive burn injury and possible hypovolemia. Which assessment data should be of most concern to the nurse? a. Urine output is 30 mL/hr. b. Blood pressure is 90/40 mm Hg. c. Oral fluid intake is 100 mL for 8 hours. d. Skin tenting over the sternum is prolonged.

ANS: B The blood pressure indicates that the patient may be developing hypovolemic shock because of intravascular fluid loss because of the burn injury. This finding will require immediate intervention to prevent the complications associated with systemic hypoperfusion. The poor oral intake, decreased urine output, and skin tenting all indicate the need for increasing the patient's fluid intake but not as urgently as the hypotension.

16. Which information will be most important for the nurse to communicate to the health care provider about an older patient who has influenza? a. Fever of 100.4° F (38° C) b. Diffuse crackles in the lungs c. Sore throat and frequent cough d. Myalgia and persistent headache

ANS: B The crackles indicate that the patient may be developing pneumonia, a common complication of influenza, which would require aggressive treatment. Myalgia, headache, mild temperature elevation, and sore throat with cough are typical manifestations of influenza and are treated with supportive care measures such as over-the-counter pain relievers and increased fluid intake.

8. The nurse observes that a patient with respiratory disease experiences a decrease in SpO2 from 93% to 88% while ambulating. What is the priority action of the nurse? a. Notify the health care provider. b. Administer PRN supplemental O2. c. Document the response to exercise. d. Encourage the patient to pace activity.

ANS: B The drop in SpO2 to 85% indicates that the patient is hypoxemic and needs supplemental O2 when exercising. The other actions are also important, but the first action should be to correct the hypoxemia.

12. Which action should the nurse take first when a patient develops epistaxis? a. Pack the affected nare tightly with an epistaxis balloon. b. Apply squeezing pressure to the nostrils for 10 minutes. c. Obtain silver nitrate that may be needed for cauterization. d. Instill a vasoconstrictor medication into the affected nare

ANS: B The first nursing action for epistaxis is to apply direct pressure by pinching the nostrils. Application of cold packs may decrease blood flow to the area but will not be sufficient to stop bleeding. Cauterization, nasal packing, and vasoconstrictors are medical interventions that may be needed if pressure to the nares does not stop the bleeding, but these are not the first actions to take for a nosebleed

10. The nurse assesses that a home hospice patient with terminal cancer who reports severe pain has a respiratory rate of 11 breaths/min. Which action should the nurse take? a. Tell the patient that increasing the morphine will cause the respiratory drive to fail. b. Titrate the prescribed morphine dose up until the patient indicates adequate pain relief. c. Inform the patient that more morphine can be given if the respiratory rate is at least 12. d. Administer a nonsteroidal antiinflammatory drug (NSAID) to improve patient pain control

ANS: B The goal of opioid use in terminally ill patients is effective pain relief regardless of adverse effects such as respiratory depression. A nonopioid analgesic such as ibuprofen would not provide adequate analgesia or be absorbed quickly. The rule of double effect provides ethical justification for administering an increased morphine dose to provide effective pain control even though the morphine may further decrease the patient's respiratory rate.

11. Which abnormality on the skin of an older patient is the priority for the nurse to discuss with the health care provider? a. Dry, scaly patches on the face b. Numerous varicosities on both legs c. Petechiae on the chest and abdomen d. Small dilated blood vessels on the face

ANS: C

31. A pregnant patient with eclampsia is receiving IV magnesium sulfate. Which finding should the nurse report to the health care provider immediately? a. The bibasilar breath sounds are decreased. b. The patellar and triceps reflexes are absent. c. The patient has been sleeping most of the day. d. The patient reports feeling "sick to my stomach."

ANS: B The loss of the deep tendon reflexes indicates that the patient's magnesium level may be reaching toxic levels. Nausea and lethargy are also side effects associated with magnesium elevation and should be reported, but they are not as significant as the loss of deep tendon reflexes. The decreased breath sounds suggest that the patient needs to cough and deep breathe to prevent atelectasis.

2. The nurse plans to teach a patient how to manage allergic rhinitis. Which information should the nurse include in the teaching plan? a. Using oral antihistamines for 2 weeks before the allergy season may prevent reactions. b. Identifying and avoiding environmental triggers are the best way to prevent symptoms. c. Frequent hand washing is the primary way to prevent spreading the condition to others. d. Corticosteroid nasal sprays will reduce inflammation, but systemic effects limit their use.

ANS: B The most important intervention is to assist the patient in identifying and avoiding potential allergens. Intranasal corticosteroids (not oral antihistamines) should be started several weeks before the allergy season. Corticosteroid nasal sprays have minimal systemic absorption. Acute viral rhinopharyngitis (common cold) can be prevented by washing hands, but allergic rhinitis cannot

12. An older adult patient who is malnourished presents to the emergency department with a serum protein level of 5.2 g/dL. Which clinical manifestation should the nurse expect? a. Pallor b. Edema c. Confusion d. Restlessness

ANS: B The normal range for total protein is 6.4 to 8.3 g/dL. Low serum protein levels cause a decrease in plasma oncotic pressure and allow fluid to remain in interstitial tissues, causing edema. Confusion, restlessness, and pallor are not associated with low serum protein levels

3. The nurse discusses management of upper respiratory infections (URIs) with a patient who has acute sinusitis. Which statement by the patient indicates that additional teaching is needed? a. "I will drink lots of juices and other fluids to stay well hydrated." b. "I can use nasal decongestant spray until the congestion is gone." c. "I can take acetaminophen (Tylenol) to treat my sinus discomfort." d. "I will watch for changes in nasal secretions or the sputum that I cough up."

ANS: B The nurse should clarify that nasal decongestant sprays should be used for no more than 3 days to prevent rebound vasodilation and congestion. The other responses indicate that the teaching has been effective.

26. A young adult female patient with cystic fibrosis (CF) tells the nurse that she is considering trying to become pregnant. Which initial response by the nurse is best? a. "Are you aware of the normal lifespan for patients with CF?" b. "Would like more information to help you with that decision?" c. "You should have genetic counseling before making a decision." d. "Many women with CF do not have difficulty conceiving children."

ANS: B The nurse's initial response should be to assess the patient's knowledge level and need for information. Although the life span for patients with CF is likely to be shorter than normal, it would not be appropriate for the nurse to address this as the initial response to the patient's comments. The other responses have accurate information, but the nurse should first assess the patient's understanding about the issues surrounding pregnancy.

20. A patient with newly diagnosed lung cancer tells the nurse, "I don't think I'm going to live to see my next birthday." Which is the best initial response by the nurse? a. "Are you ready to talk with family members about dying?" b. "Can you tell me what makes you think you will die so soon?" c. "Do you think that an antidepressant medication would be helpful?" d. "Would you like to talk to the hospital chaplain about your feelings?"

ANS: B The nurse's initial response should be to collect more assessment data about the patient's statement. The answer beginning "Can you tell me what it is" is the most open-ended question and will offer the best opportunity for obtaining more data. The remaining answers offer interventions that may be helpful to the patient, but more assessment is needed to determine whether these interventions are appropriate.

12. The nurse is caring for a patient with colon cancer who is scheduled for external radiation therapy to the abdomen. Which information obtained by the nurse would indicate a need for patient teaching? a. The patient has a history of dental caries. b. The patient swims several days each week. c. The patient snacks frequently during the day. d. The patient showers each day with mild soap

ANS: B The patient is instructed to avoid swimming in salt water or chlorinated pools during the treatment period. The patient does not need to change habits of eating frequently or showering with a mild soap. A history of dental caries will not impact the patient who is scheduled for abdominal radiation

16. A chemotherapy drug that causes alopecia is prescribed for a patient. Which action should the nurse take to support the patient's self-esteem? a. Suggest that the patient limit social contacts until hair regrowth occurs. b. Encourage the patient to purchase a wig or hat to wear when hair loss begins. c. Teach the patient to wash hair gently with mild shampoo to minimize hair loss. d. Inform the patient that hair usually grows back once chemotherapy is complete

ANS: B The patient is taught to anticipate hair loss and to be prepared with wigs, scarves, or hats. Limiting social contacts is not appropriate at a time when the patient is likely to need a good social support system. The damage occurs at the hair follicles and will occur regardless of gentle washing or use of a mild shampoo. The information that the hair will grow back is not immediately helpful in maintaining the patient's self-estee

5. During a routine health examination, a 40-yr-old patient tells the nurse about a family history of colon cancer. Which action should the nurse take next? a. Schedule a sigmoidoscopy to provide baseline data. b. Obtain more information about the patient's relatives. c. Teach the patient about the need for a colonoscopy at age 50. d. Teach the patient how to do home testing for fecal occult blood.

ANS: B The patient may be at increased risk for colon cancer, but the nurse's first action should be further assessment. The other actions may be appropriate, depending on the information that is obtained from the patient with further questioning

16. The nurse is administering a thrombolytic agent to a patient with an acute myocardial infarction. What patient data indicates that the nurse should stop the drug infusion? a. Bleeding from the gums b. An increase in blood pressure c. Decreased level of consciousness d. A nonsustained episode of ventricular tachycardia

ANS: C

24. A patient with leukemia is considering whether to have hematopoietic stem cell transplantation (HSCT). Which information should the nurse include in the patient's teaching plan? a. Donor bone marrow is transplanted through a sternal or hip incision. b. Hospitalization is required for several weeks after the stem cell transplant. c. The transplant procedure takes place in a sterile operating room to decrease the risk for infection. d. Transplant of the donated cells can be very painful because of the nerves in the tissue lining the bone

ANS: B The patient requires strict protective isolation to prevent infection for 2 to 4 weeks after HSCT while waiting for the transplanted marrow to start producing cells. The transplanted cells are infused through an IV line so the transplant is not painful, nor is an operating room or incision required

7. A patient with new-onset confusion and hyponatremia is being admitted. Which action should the charge nurse take when making room assignments? a. Assign the patient to a semiprivate room. b. Assign the patient to a room near the nurse's station. c. Place the patient in a room nearest to the water fountain. d. Place the patient on telemetry to monitor for peaked T waves.

ANS: B The patient should be placed near the nurse's station if confused for the staff to closely monitor the patient. To help improve serum sodium levels, water intake is restricted. Therefore, a confused patient should not be placed near a water fountain. Peaked T waves are a sign of hyperkalemia, not hyponatremia. A confused patient could be distracting and disruptive for another patient in a semiprivate room.

14. A patient seeks care in the emergency department after sharing needles for heroin injection with a friend who has hepatitis B. To provide immediate protection from infection, what medication will the nurse expect to administer? a. Corticosteroids b. Gamma globulin c. Hepatitis B vaccine d. Fresh frozen plasma

ANS: B The patient should first receive antibodies for hepatitis B from injection of gamma globulin. The hepatitis B vaccination series should be started to provide active immunity. Fresh frozen plasma and corticosteroids will not be effective in preventing hepatitis B in the patient

13. The nurse teaches a patient about pursed-lip breathing. Which action by the patient would indicate to the nurse that further teaching is needed? a. The patient inhales slowly through the nose. b. The patient puffs up the cheeks while exhaling. c. The patient practices by blowing through a straw. d. The patient's ratio of inhalation to exhalation is 1:3

ANS: B The patient should relax the facial muscles without puffing the cheeks while doing pursed-lip breathing. The other actions by the patient indicate a good understanding of pursed-lip breathing.

27. During the teaching session for a patient who has a new diagnosis of acute leukemia, the patient is restless and looks away without making eye contact. The patient asks the nurse to repeat the information about the complications associated with chemotherapy. Based on this assessment, which patient problem should the nurse identify? a. Denial b. Anxiety c. Acute confusion d. Ineffective adherence to treatment

ANS: B The patient who has a new cancer diagnosis is likely to have high anxiety, which may affect learning and require that the nurse repeat and reinforce information about health maintenance. There is no evidence to support confusion. The patient asks for the information to be repeated, indicating that denial is not present. The patient has recently been diagnosed, so adherence has not yet been required.

8. The nurse taking a health history learns that the patient, who has worked in rubber tire manufacturing, has allergic rhinitis and multiple food allergies. Which action by the nurse is correct? a. Recommend that the patient use latex gloves in preventing blood-borne pathogen contact. b. Document the patient's history and teach about clinical manifestations of a type I latex allergy. c. Encourage the patient to carry an epinephrine kit in case a type IV allergic reaction to latex develops. d. Tell the patient to use oil-based hand creams to decrease contact with natural proteins in latex gloves.

ANS: B The patient's allergy history and occupation indicate a risk of developing a latex allergy. Teach the patient about symptoms that may occur. Epinephrine is not an appropriate treatment for contact dermatitis that is caused by a type IV allergic reaction to latex. Using latex gloves increases the chance of developing latex sensitivity. Oil-based creams will increase the exposure to latex from latex gloves.

22. When assessing a patient with a sore throat, the nurse notes anterior cervical lymph node swelling, a temperature of 101.6° F (38.7° C), and yellow patches on the tonsils. Which action will the nurse anticipate taking? a. Teach the patient about the use of expectorants. b. Use a swab to obtain a sample for a rapid strep antigen test. c. Discuss the need to rinse the mouth out after using any inhalers. d. Teach the patient to avoid nonsteroidal antiinflammatory drugs (NSAIDs).

ANS: B The patient's clinical manifestations are consistent with streptococcal pharyngitis, and the nurse will anticipate the need for a rapid strep antigen test or cultures (or both). Because patients with streptococcal pharyngitis usually do not have a cough, use of expectorants will not be anticipated. Rinsing out the mouth after inhaler use may prevent fungal oral infections, but the patient's assessment data are not consistent with a fungal infection. NSAIDs are often prescribed for pain and fever relief with pharyngitis.

23. A patient who has been hospitalized for 2 days has a nasogastric tube to low suction and is receiving normal saline IV at 100 mL/hr. Which assessment finding would be a priority for the nurse to report to the health care provider? a. Oral temperature of 100.1° F b. Decreased alertness since admission c. Weight gain of 2 pounds (1 kg) over 2 days d. Serum sodium level of 138 mEq/L (138 mmol/L)

ANS: B The patient's history and change in LOC could be indicative of fluid and electrolyte disturbances: extracellular fluid (ECF) excess, ECF deficit, hyponatremia, hypernatremia, hypokalemia, or metabolic alkalosis. Further diagnostic information is needed to determine the cause of the change in LOC and the appropriate interventions. The weight gain, elevated temperature, and serum sodium level will be reported but do not indicate a need for rapid action to avoid complications.

13. The nurse is caring for a patient who has diabetes and reports chronic, burning leg pain even when taking oxycodone (OxyContin) twice daily. Which prescribed medication is the best choice for the nurse to administer as an adjuvant to decrease the patient's pain? a. Aspirin b. Amitriptyline c. Celecoxib (Celebrex) d. Acetaminophen (Tylenol)

ANS: B The patient's pain symptoms are consistent with neuropathic pain and the tricyclic antidepressants are effective for treating this type of pain. The other medications are more effective for nociceptive pain.

40. The clinic nurse makes a follow-up telephone call to a patient with asthma. The patient reports having a baseline peak flow reading of 600 L/min, and the current peak flow is 420 L/min. Which action should the nurse take first? a. Tell the patient to go to the hospital emergency department. b. Teach the patient to use the prescribed albuterol (Ventolin HFA). c. Ask about recent exposure to any new allergens or asthma triggers. d. Question the patient about use of the prescribed inhaled corticosteroids

ANS: B The patient's peak flow is 70% of normal, indicating a need for immediate use of short-acting B2-adrenergic SABA medications. Assessing for correct use of medications or exposure to allergens is appropriate but would not address the current decrease in peak flow. Because the patient is currently in the yellow zone, hospitalization is not needed.

6. A patient seen in the asthma clinic has recorded daily peak flowrates that are 75% of the baseline. Which action will the nurse plan to take next? a. Teach the patient about the use of oral corticosteroids. b. Administer a bronchodilator and recheck the spirometry. c. Recommend increasing the dose of the leukotriene inhibitor. d. Instruct the patient to keep the scheduled follow-up appointment

ANS: B The patient's peak flow reading indicates that the condition is worsening (yellow zone). The patient should take the bronchodilator and recheck the peak flow. Depending on whether the patient returns to the green zone, indicating well-controlled symptoms, the patient may be prescribed oral corticosteroids or a change in dosing of other medications. Keeping the next appointment is appropriate, but the patient first needs to be taught how to control symptoms now and use the bronchodilator.

26. A patient who had a transverse colectomy for diverticulosis 18 hours ago has nasogastric suction. The patient reports anxiety and incisional pain. The patient's respiratory rate is 32 breaths/min, and the arterial blood gases (ABGs) indicate respiratory alkalosis. Which action should the nurse take first? a. Check to make sure the nasogastric tube is patent. b. Give the patient the PRN IV morphine sulfate 4 mg. c. Notify the health care provider about the ABG results. d. Teach the patient to take slow, deep breaths when anxio

ANS: B The patient's respiratory alkalosis is caused by the increased respiratory rate associated with pain and anxiety. The nurse's first action should be to medicate the patient for pain. The health care provider may be notified about the ABGs but is likely to instruct the nurse to medicate for pain. The patient will not be able to take slow, deep breaths when experiencing pain. Checking the nasogastric tube can wait until the patient has been medicated for pain.

21. A patient with severe chronic obstructive pulmonary disease (COPD) tells the nurse, "I wish I were dead! I'm just a burden on everybody." Based on this information, which patient problem should the nurse identify? a. Fear of death b. Low self-esteem c. Extended grieving d. Inadequate knowledge

ANS: B The patient's statement about not being able to do anything for himself or herself reflects low self-esteem. Although inadequate knowledge, extended grieving, and fear of death may be problems for some patients who have COPD, the data for this patient do not support these problems.

29. Following a thyroidectomy, a patient reports "a tingling feeling around my mouth." Which assessment should the nurse complete first? a. Verify the serum potassium level. b. Test for presence of Chvostek's sign. c. Observe for blood on the neck dressing. d. Confirm a prescription for thyroid replacement

ANS: B The patient's symptoms indicate possible hypocalcemia, which can occur secondary to parathyroid injury or removal during thyroidectomy. There is no indication of an urgent need to check the potassium level, the thyroid replacement, or for bleeding.

16. A patient who has inhaled cocaine is admitted to the emergency department with palpitations and shortness of breath. What should the nurse do first? a. Infuse normal saline. b. Check oxygen saturation. c. Draw blood for drug screening. d. Obtain a 12-lead echocardiogram (ECG)

ANS: B The priority here is to ensure that oxygenation is adequate. The other orders also should be accomplished as soon as possible but are not the first priority.

21. The nurse is caring for a patient who has a central venous access device (CVAD). Which action by the nurse is appropriate? a. Avoid using friction when cleaning around the CVAD insertion site. b. Use the push-pause method to flush the CVAD after giving medications. c. Position the patient's face toward the CVAD during injection cap changes. d. Obtain a prescription from the health care provider to change CVAD dressing

ANS: B The push-pause enhances the removal of debris from the CVAD lumen and decreases the risk for clotting. To decrease infection risk, friction should be used when cleaning the CVAD insertion site. The dressing should be changed whenever it becomes damp, loose, or visibly soiled. A provider's order is not necessary. The patient should turn away from the CVAD during cap changes.

30. A patient is admitted to the emergency department with severe fatigue and confusion. Laboratory studies are done. Which laboratory value will require the most immediate action by the nurse? a. Arterial blood pH is 7.32. b. Serum calcium is 18 mg/dL. c. Serum potassium is 5.1 mEq/L. d. Arterial oxygen saturation is 91

ANS: B The serum calcium is well above the normal level and puts the patient at risk for cardiac dysrhythmias. The nurse should start cardiac monitoring and notify the health care provider. The potassium, oxygen saturation, and pH are also abnormal, and the nurse should notify the health care provider about these values as well, but they are not immediately life threatening.

14. When admitting a patient with stage 3 pressure injuries on both heels, which information obtained by the nurse will have the most impact on wound healing? a. The patient has had the injuries for 6 months. b. The patient takes oral hypoglycemic agents daily. c. The patient states that the injuries are very painful. d. The patient has several incisions that formed keloids.

ANS: B The use of oral hypoglycemics indicates diabetes, which can interfere with wound healing. The persistence of the injuries over the past 6 months is a concern, but changes in care may be effective in promoting healing. Keloids are not disabling, although the cosmetic effects may be distressing for some patients. Actions to reduce the patient's pain will be implemented, but pain does not directly affect wound healing.

12. Heparin is ordered for a patient with a non-ST-segment-elevation myocardial infarction (NSTEMI). How should the nurse explain the purpose of the heparin to the patient? a. "Heparin enhances platelet aggregation at the plaque site." b. "Heparin decreases the size of the coronary artery plaque." c. "Heparin prevents the development of new clots in the coronary arteries." d. "Heparin dissolves clots that are blocking blood flow in the coronary arteries."

ANS: C

19. A patient presents to the emergency department with a blood alcohol concentration (BAC) of 0.18%. After reviewing the prescribed medications, which drug should the nurse administer first? a. Oral multivitamin daily b. Thiamine (vitamin B1) 100 mg daily c. Lorazepam (Ativan) 1 mg as needed d. Folic acid (Vitamin B9) 0.4 mg daily

ANS: B Thiamine is given to all patients with alcohol intoxication to prevent Wernicke-Korsakoff syndrome. Because Wernicke-Korsakoff syndrome can be precipitated by the administration of glucose solutions, thiamine should be given before or concurrently with a dextrose solution. Lorazepam would not be appropriate while the patient still has an elevated BAC but may be used later for withdrawal symptoms. Folic acid and multivitamins may also be administered but are not as urgent as thiamine.

20. A widowed mother of 4 school-age children is hospitalized with metastatic ovarian cancer. The patient is crying and tells the nurse that she does not know what will happen to her children when she dies. Which response by the nurse is most appropriate? a. "Don't you have any friends that will raise the children for you?" b. "Would you like to talk about options for the care of your children?" c. "For now you need to concentrate on getting well and not worrying about your children." d. "Many patients with cancer live for a long time, so there is time to plan for your children."

ANS: B This response expresses the nurse's willingness to listen and recognizes the patient's concern. The responses beginning "Many patients with cancer live for a long time" and "For now you need to concentrate on getting well" close off discussion of the topic and indicate that the nurse is uncomfortable with the topic. In addition, the patient with metastatic ovarian cancer may not have a long time to plan. Although it is possible that the patient's friends will raise the children, more assessment information is needed before making plans.

19. An hour after a left thoracotomy, a patient reports incisional pain at a level 7 (based on 0 to 10 scale) and has decreased left-sided breath sounds. The pleural drainage system has 100 mL of bloody drainage and a large air leak. Which action should the nurse take? a. Clamp the chest tube in two places. b. Administer the prescribed morphine. c. Milk the chest tube to remove any clots. d. Assist the patient with incentive spirometry.

ANS: B Treat the pain. The patient is unlikely to take deep breaths or cough until the pain level is lower. A chest tube output of 100 mL is not unusual in the first hour after thoracotomy. Milking or stripping chest tubes is no longer recommended because these practices can dangerously increase intrapleural pressures and damage lung tissues. Position tubing so that drainage flows freely to negate need for milking or stripping. An air leak is expected in the initial postoperative period after thoracotomy. Clamping the chest tube is not indicated and may lead to dangerous development of a tension pneumothorax.

11. A patient with Hodgkin's lymphoma who is undergoing external radiation therapy tells the nurse, "I am so tired I can hardly get out of bed in the morning." Which intervention should the nurse add to the plan of care? a. Minimize activity until the treatment is completed. b. Establish time to take a short walk almost every day. c. Consult with a psychiatrist for treatment of depression. d. Arrange for delivery of a hospital bed to the patient's home.

ANS: B Walking programs are used to keep the patient active without excessive fatigue. Having a hospital bed does not necessarily address the fatigue. The better option is to stay as active as possible while combating fatigue. Fatigue is expected during treatment and is not an indication of depression. Minimizing activity may lead to weakness and other complications of immobility.

5. Which action should the nurse plan to prevent aspiration in a high-risk patient? a. Turn and reposition an immobile patient at least every 2 hours. b. Place a patient with altered consciousness in a side-lying position. c. Insert a nasogastric tube for feeding a patient with high-calorie needs. d. Monitor respiratory symptoms in a patient who is immunosuppressed.

ANS: B With loss of consciousness, the gag and cough reflexes are depressed, and aspiration is more likely to occur. The risk for aspiration is decreased when patients with a decreased level of consciousness are placed in a side-lying or upright position. Frequent turning prevents pooling of secretions in immobilized patients but will not decrease the risk for aspiration in patients at risk. Monitoring of parameters such as breath sounds and O2 saturation will help detect pneumonia in immunocompromised patients, but it will not decrease the risk for aspiration. Conditions that increase the risk of aspiration include decreased level of consciousness (e.g., seizure, anesthesia, head injury, stroke, alcohol intake), difficulty swallowing, and nasogastric intubation with or without tube feeding.

17. The nurse is caring for a patient with diabetes who had abdominal surgery 3 days ago. Which finding is most important for the nurse to report to the health care provider? a. Blood glucose of 136 mg/dL b. Separation of proximal wound edges c. Oral temperature of 101° F (38.3° C) d. Patient reports increased incisional pain

ANS: B Wound separation 3 days postoperatively indicates possible wound dehiscence and should be immediately reported to the health care provider. The other findings will also be reported but do not require intervention by the HCP as rapidly.

25. The nurse teaches a patient with liver cancer about high-protein, high-calorie diet choices. Which snack choice by the patient indicates that the teaching has been effective? a. Lime sherbet b. Blueberry yogurt c. Fresh strawberries d. Cream cheese bagel

ANS: B Yogurt has high biologic value because of the protein and fat content. Fruit salad does not have high amounts of protein or fat. Lime sherbet is lower in fat and protein than yogurt. Cream cheese is low in protein.

12. A patient admitted to the emergency department with a sudden onset of shortness of breath is diagnosed with a possible pulmonary embolus. How should the nurse prepare the patient for diagnostic testing to confirm the diagnosis? a. Ensure that the patient has been NPO. b. Start an IV so contrast media may be given. c. Inform radiology that radioactive glucose preparation is needed. d. Instruct the patient to expect to inspire deeply and exhale forcefully.

ANS: B S Spiral computed tomography scans are the most commonly used test to diagnose pulmonary emboli and contrast media may be given IV. Bronchoscopy is used to detect changes in the bronchial tree, not to assess for vascular changes, and the patient should be NPO 6 to 12 hours before the procedure. Positron emission tomography scans are most useful in determining the presence of cancer and a radioactive glucose preparation is used. For spirometry, the patient is asked to inhale deeply and exhale as long, hard, and fast as possible.

13. In preparation for which test should the nurse teach the patient to minimize physical and emotional stress? a. A water deprivation test b. A test for serum T3 and T4 levels c. A 24-hour urine test for free cortisol d. A radioactive iodine (I-131) uptake test

ANS: C

38. A patient who is being treated for stage IV lung cancer tells the nurse about new-onset back pain. Which action should the nurse take first? a. Give the patient the prescribed PRN opioid. b. Assess for sensation and strength in the legs. c. Notify the health care provider about the symptoms. d. Teach the patient how to use relaxation to reduce pain

ANS: B Spinal cord compression, an oncologic emergency, can occur with invasion of tumor into the epidural space. The nurse will need to assess the patient further for symptoms such as decreased leg sensation and strength and then notify the health care provider. Administration of opioids or the use of relaxation may be appropriate but only after the nurse has assessed for possible spinal cord compression.

2. According to the Center for Disease Control and Prevention (CDC) guidelines, which personal protective equipment will the nurse put on before assessing a patient who is on contact precautions for Clostridium difficile diarrhea? (Select all that apply.) a. Mask b. Gown c. Gloves d. Shoe covers e. Eye protection

ANS: B, C Because the nurse will have substantial contact with the patient and bedding when doing an assessment, gloves and gowns are needed. Eye protection and masks are needed for patients in contact precautions only when spraying or splashing is anticipated. Shoe covers are not recommended in the CDC guidelines

1. A nurse is teaching a patient with contact dermatitis of the arms and legs about ways to decrease pruritus. Which information should the nurse include in the teaching plan? (Select all that apply.) a. Add oil to your bath water to moisturize the affected skin. b. Cool, wet clothes or compresses can be used to reduce itching. c. Use an over-the-counter (OTC) antihistamine to reduce itching. d. Take cool or tepid baths several times daily to decrease itching. e. Rub yourself dry with a towel after bathing to prevent skin maceration.

ANS: B, C, D

1. To monitor for complications in a patient with type 2 diabetes, which tests will the nurse in the diabetic clinic schedule at least annually? (Select all that apply.) a. Chest x-ray b. Blood pressure c. Serum creatinine d. Urine for microalbuminuria e. Complete blood count (CBC) f. Monofilament testing of the foot

ANS: B, C, D, F

1. The health care provider orders a patient-controlled analgesia (PCA) machine to provide pain relief for a patient with acute surgical pain who has never received opioids before. Which nursing actions regarding opioid administration are appropriate at this time? (Select all that apply.) a. Assess for signs that the patient is becoming addicted to the opioid. b. Monitor for therapeutic and adverse effects of opioid administration. c. Emphasize that the risk of some opioid side effects increases over time. d. Teach the patient about how analgesics improve postoperative activity levels. e. Provide instructions on decreasing opioid doses by the second postoperative day

ANS: B, D Monitoring for pain relief and teaching the patient about how opioid use will improve postoperative outcomes are appropriate actions when administering opioids for acute pain. Although postoperative patients usually need a decreasing amount of opioids by the second postoperative day, each patient's response is individual. Tolerance may occur, but addiction to opioids will not develop in the acute postoperative period. The patient should use the opioids to achieve adequate pain control, so the nurse should not emphasize the adverse effects.

1. Which information should the nurse include when teaching patients about decreasing the risk for sun damage to the skin? a. Use a sunscreen with an SPF of at least 10 for adequate protection. b. Water-resistant sunscreens provide good protection when swimming. c. Try to stay out of the direct sun between the hours of 10 AM and 2 PM. d. Increase sun exposure by no more than 10 minutes a day to avoid skin damage.

ANS: C

1. Which statement by a nurse to a patient newly diagnosed with type 2 diabetes is accurate? a. Insulin is not used to control blood glucose in patients with type 2 diabetes. b. Complications of type 2 diabetes are less serious than those of type 1 diabetes. c. Changes in diet and exercise may control blood glucose levels in type 2 diabetes. d. Type 2 diabetes is usually diagnosed when a patient is admitted in hyperglycemic coma.

ANS: C

10. A patient is taking spironolactone (Aldactone), a drug that blocks the action of aldosterone on the kidney, for hypertension. What additional effect of the medication should the nurse monitor? a. Increased serum sodium b. Decreased urinary output c. Elevated serum potassium d. Evidence of fluid overload

ANS: C

10. During change-of-shift report, the nurse obtains the following information about a hypertensive patient who received the first dose of nadolol (Corgard) during the previous shift. Which information indicates that the patient needs immediate intervention? a. The patient's pulse has dropped from 68 to 57 beats/min. b. The patient reports that the fingers and toes feel quite cold. c. The patient has developed wheezes throughout the lung fields. d. The patient's blood pressure (BP) reading is now 158/92 mm Hg.

ANS: C

10. Which data indicates to the nurse that the patient with stable angina is experiencing a side effect of metoprolol (Lopressor)? a. Patient is restless and agitated. b. Patient reports feeling anxious. c. Blood pressure is 90/54 mm Hg. d. Heart monitor shows normal sinus rhythm.

ANS: C

16. What is the safest technique for the nurse to use when assisting a blind patient to ambulate to the bathroom? a. Lead the patient slowly to the bathroom, holding on to the patient by the arm. b. Stay beside the patient and describe any obstacles on the path to the bathroom. c. Walk slightly ahead of the patient, allowing the patient to hold the nurse's elbow. d. Have the patient place a hand on the nurse's shoulder and guide the patient forward.

ANS: C

16. When the nurse is taking a health history of a new patient at the ear clinic, the patient states, "I have to sleep with the television on." Which follow-up question is appropriate to obtain more information about possible hearing problems? a. "Do you grind your teeth at night?" b. "What time do you usually fall asleep?" c. "Have you noticed ringing in your ears?" d. "Are you ever dizzy when you are lying down?"

ANS: C

16. Which information obtained by the nurse who is admitting the patient for magnetic resonance imaging (MRI) will be important to report to the health care provider before the MRI? a. The patient has an allergy to shellfish. b. The patient has a history of atherosclerosis. c. The patient has a permanent cardiac pacemaker. d. The patient took the prescribed heart medications today.

ANS: C

17. A patient recovering from a myocardial infarction (MI) develops chest pain on day 3 that increases when taking a deep breath and is relieved by leaning forward. Which action should the nurse take as focused follow-up on this symptom? a. Assess both feet for pedal edema. b. Palpate the radial pulses bilaterally. c. Auscultate for a pericardial friction rub. d. Check the heart monitor for dysrhythmias.

ANS: C

17. When a patient who takes metformin (Glucophage) to manage type 2 diabetes develops an allergic rash from an unknown cause, the health care provider prescribes prednisone. What should the nurse anticipate? a. The patient may need a diet higher in calories while receiving prednisone. b. The patient may develop acute hypoglycemia while taking the prednisone. c. The patient may require administration of insulin while taking prednisone. d. The patient may have rashes caused by metformin-prednisone interactions.

ANS: C

17. When the nurse is monitoring a patient who is undergoing exercise (stress) testing on a treadmill, which assessment finding requires the most rapid action by the nurse? a. Patient reports feeling tired b. Sinus tachycardia at a rate of 110 beats/min c. Inversion of T waves on the electrocardiogram d. Blood pressure (BP) increase from 134/68 to 150/80 mm Hg

ANS: C

18. A hospitalized patient who is diabetic received 38 U of NPH insulin at 7:00 AM. At 1:00 PM, the patient has been away from the nursing unit for 2 hours, missing the lunch delivery while awaiting a chest x-ray. What is the best action by the nurse to prevent hypoglycemia? a. Plan to discontinue the evening dose of insulin. b. Save the lunch tray for the patient's later return. c. Request that if testing is further delayed, the patient will eat lunch first. d. Send a glass of orange juice to the patient in the diagnostic testing area.

ANS: C

18. The nurse recording health histories in the outpatient clinic would plan a focused hearing assessment for adult patients taking which medication? a. Atenolol b. Albuterol c. Ibuprofen d. Acetaminophen

ANS: C

18. The nurse teaches a patient about application of corticosteroid cream to an area of contact dermatitis on the right leg. Which patient action indicates that further teaching is needed? a. The patient takes a tepid bath before applying the cream. b. The patient spreads the cream using a downward motion. c. The patient applies a thick layer of the cream to the affected skin. d. The patient covers the area with a dressing after applying the cream.

ANS: C

2. A patient diagnosed with external otitis is being discharged from the emergency department with an ear wick in place. Which statement by the patient indicates a need for further teaching? a. "I will apply the eardrops to the cotton wick in the ear canal." b. "I can use aspirin or acetaminophen (Tylenol) for pain relief." c. "I will clean the ear canal daily with a cotton-tipped applicator." d. "I can use warm compresses to the outside of the ear for comfort."

ANS: C

2. A patient screened for diabetes at a clinic has a fasting plasma glucose level of 120 mg/dL (6.7 mmol/L). What should the nurse plan to teach the patient? a. Self-monitoring of blood glucose b. Using low doses of regular insulin c. Lifestyle changes to lower blood glucose d. Effects of oral hypoglycemic medications

ANS: C

2. Which nursing intervention is likely to be most effective when assisting the patient with coronary artery disease to make appropriate dietary changes? a. Inform the patient about a diet containing no saturated fat and minimal salt. b. Emphasize the increased cardiac risk unless the patient makes dietary changes. c. Help the patient modify favorite high-fat recipes by using monounsaturated oils. d. Give the patient a list of low-sodium, low-cholesterol foods to include in the diet.

ANS: C

2. Which statement made by a 50-yr-old female patient indicates to the nurse that further assessment of thyroid function may be needed? a. "I am so thirsty that I drink all day long." b. "I get up several times at night to urinate." c. "I feel a lump in my throat when I swallow." d. "I notice my breasts are always tender lately."

ANS: C

20. A patient with diabetes mellitus and chronic stable angina has a new order for captopril. What should the nurse teach this patient about the primary purpose of captopril? a. Decreases the heart rate. b. Controls blood glucose levels. c. Prevents changes in heart muscle. d. Reduces the frequency of chest pain.

ANS: C

20. A transesophageal echocardiogram (TEE) is planned for a patient hospitalized with possible endocarditis. Which action included in the standard TEE orders will the nurse need to accomplish first? a. Start an IV line. b. Start O2 per nasal cannula. c. Place the patient on NPO status. d. Give lorazepam (Ativan) 1 mg IV.

ANS: C

20. The nurse assesses a patient who has just arrived in the postanesthesia recovery area (PACU) after a blepharoplasty. Which assessment data should be reported to the surgeon immediately? a. The patient reports incisional pain. b. The patient's heart rate is 100 beats/min. c. The skin around the incision is pale and cold. d. The patient is unable to sense touch on the eyelids.

ANS: C

20. Which assessment finding should the nurse report to the health care provider? a. Visible cone of light b. Dry skin in the ear canal c. A blue-tinged tympanum d. Cerumen in the auditory canal

ANS: C

22. A 26-yr-old female who has type 1 diabetes develops a sore throat and runny nose after caring for her sick toddler. The patient calls the clinic for advice about her symptoms and reports a blood glucose level of 210 mg/dL despite taking her usual glargine (Lantus) and lispro (Humalog) insulin. What should the nurse advise the patient to do? a. Use only the lispro insulin until the symptoms are resolved. b. Limit intake of calories until the glucose is less than 120 mg/dL. c. Monitor blood glucose every 4 hours and contact the clinic if it rises. d. Decrease carbohydrates until glycosylated hemoglobin is less than 7%.

ANS: C

22. Which action should the nurse take when teaching a patient with mild presbycusis? a. Use patient education handouts rather than discussion. b. Use a high-pitched tone of voice to provide instructions. c. Ask for permission to turn off the television before teaching. d. Wait until family members have left before initiating teaching.

ANS: C

23. A patient with a right retinal detachment had a pneumatic retinopexy procedure. Which information will the nurse include in the discharge teaching plan? a. The use of eye patches to reduce movement of the operative eye b. The need to wear dark glasses to protect the eyes from bright light c. The purpose of maintaining the head resting in a prescribed position d. The procedure for dressing changes when the eye dressing is saturated

ANS: C

23. The health care provider suspects the Somogyi effect in a 50-yr-old patient whose 6:00 AMblood glucose is 230 mg/dL. Which action will the nurse teach the patient to take? a. Avoid snacking right before bedtime. b. Increase the rapid-acting insulin dose. c. Check the blood glucose during the night. d. Administer a larger dose of long-acting insulin.

ANS: C

23. There is one opening in the schedule at the dermatology clinic, and four patients are seeking appointments today. Which patient will the nurse schedule for the available opening? a. 42-yr-old with itching after using topical fluorouracil on the nose b. 50-yr-old with skin redness after having a chemical peel 3 days ago c. 38-year old with a 7-mm nevus on the face that has recently become darker d. 62-yr-old with multiple small, soft, pedunculated papules in both axillary areas

ANS: C

24. A patient with hyperlipidemia has a new order for colesevelam (Welchol). Which nursing action is appropriate when scheduling this medication? a. Administer the medication at the patient's usual bedtime. b. Have the patient take the colesevelam 1 hour before breakfast. c. Give the patient's other medications 2 hours after colesevelam. d. Have the patient take the dose at the same time as the prescribed aspirin.

ANS: C

28. The nurse at the outpatient surgery unit obtains the following information about a patient who is scheduled for cataract extraction and implantation of an intraocular lens. Which information is important to report to the health care provider before the procedure? a. The patient has had blurred vision for 3 years. b. The patient has not eaten anything for 8 hours. c. The patient takes antihypertensive medications. d. The patient gets nauseated with general anesthesia.

ANS: C

29. A patient who has type 2 diabetes is being prepared for an elective coronary angiogram. Which information would the nurse anticipate might lead to rescheduling the test? a. The patient's most recent A1C was 7.5%. b. The patient's blood glucose is 128 mg/dL. c. The patient took the prescribed metformin today. d. The patient took the prescribed enalapril 4 hours ago.

ANS: C

29. The nurse learns that a newly admitted patient has functional blindness and that the spouse has cared for the patient for many years. What is the nurse's most important action during the initial assessment? a. Obtain more information about the cause of the patient's vision loss. b. Obtain information from the spouse about the patient's special needs. c. Make eye contact with the patient and ask about any need for assistance. d. Perform an evaluation of the patient's visual acuity using a Snellen chart.

ANS: C

3. During a physical examination of an older patient, the nurse palpates the point of maximal impulse (PMI) in the sixth intercostal space lateral to the left midclavicular line. What would be the most focused follow-up action for the nurse to take? a. Ask about risk factors for atherosclerosis. b. Determine family history of heart disease. c. Assess for symptoms of left ventricular hypertrophy. d. Auscultate carotid arteries for the presence of a bruit.

ANS: C

3. What should the nurse include when teaching a patient who has undergone a left tympanoplasty? a. "Remain on bed rest." b. "Keep your head elevated." c. "Avoid blowing your nose." d. "Irrigate your left ear canal."

ANS: C

3. Which information should the nurse include in the teaching plan for a patient diagnosed with basal cell carcinoma (BCC)? a. Treatment plans include watchful waiting. b. Screening for metastasis will be important. c. Minimizing sun exposure reduces risk for future BCC. d. Low-dose systemic chemotherapy is used to treat BCC.

ANS: C

30. Which action by a patient indicates that the home health nurse's teaching about glargine and regular insulin has been successful? a. The patient administers the glargine 30 minutes before each meal. b. The patient's family prefills the syringes with the mix of insulins weekly. c. The patient discards the open vials of glargine and regular insulin after 4 weeks. d. The patient draws up the regular insulin and then the glargine in the same syringe.

ANS: C

30. Which electrocardiographic (ECG) change by a patient with chest pain is most important for the nurse to report rapidly to the health care provider? a. Inverted P wave b. Sinus tachycardia c. ST-segment elevation d. First-degree atrioventricular block

ANS: C

31. A patient with acute coronary syndrome has returned to the coronary care unit after having angioplasty with stent placement. Which assessment data indicate the need for immediate action by the nurse? a. Report of chest pain b. Heart rate 102 beats/min c. Pedal pulses 1+ bilaterally d. Blood pressure 103/54 mm Hg

ANS: C

31. A patient with diabetes rides a bicycle to and from work every day. Which site should the nurse teach the patient to use to administer the morning insulin? a. Thigh b. Buttock c. Abdomen d. Upper arm

ANS: C

33. A patient in the emergency department reports being struck in the right eye with a fist. Which finding is a priority for the nurse to communicate to the health care provider? a. The patient reports a right-sided headache. b. The sclera on the right eye has broken blood vessels. c. The patient reports "a curtain" over part of the visual field. d. The area around the right eye is bruised and tender to the touch.

ANS: C

34. Which information about a patient receiving thrombolytic therapy for an acute myocardial infarction is most important for the nurse to communicate to the health care provider? a. An increase in troponin levels from baseline b. A large bruise at the patient's IV insertion site c. No change in the patient's reported level of chest pain d. A decrease in ST-segment elevation on the electrocardiogram

ANS: C

35. The nurse obtains the following data when assessing a patient who experienced an ST-segment-elevation myocardial infarction (STEMI) 2 days previously. Which information is most important to report to the health care provider? a. The troponin level is elevated. b. The patient denies having a heart attack. c. Bilateral crackles in the mid-lower lobes. d. Occasional premature atrial contractions (PACs).

ANS: C

36. A patient who was admitted with diabetic ketoacidosis secondary to a urinary tract infection has been weaned off an insulin drip 30 minutes ago. The patient reports feeling lightheaded and sweaty. Which action should the nurse take first? a. Infuse dextrose 50% by slow IV push. b. Administer 1 mg glucagon subcutaneously. c. Obtain a glucose reading using a finger stick. d. Have the patient drink 4 ounces of orange juice.

ANS: C

36. A patient with a head injury after a motorcycle crash arrives in the emergency department (ED) reporting shortness of breath and severe eye pain. Which action will the nurse take first? a. Assess cranial nerve functions. b. Administer the prescribed analgesic. c. Check the patient's oxygen saturation. d. Examine the eye for evidence of trauma.

ANS: C

37. A patient who has chest pain is admitted to the emergency department (ED), and all of the following items are prescribed. Which one should the nurse arrange to be completed first? a. Chest x-ray b. Troponin level c. Electrocardiogram (ECG) d. Insertion of a peripheral IV

ANS: C

39. A patient who received a corneal transplant 2 weeks ago calls the ophthalmology clinic to report that his vision has not improved with the transplant. Which action should the nurse take? a. Suggest the patient arrange a ride to the clinic immediately. b. Ask about the presence of "floaters" in the patient's visual field. c. Remind the patient it may take months to restore vision after transplant. d. Teach the patient to continue using prescribed pupil-dilating medications.

ANS: C

39. When a patient with type 2 diabetes is admitted for a cholecystectomy, which nursing action can the nurse delegate to a licensed practical/vocational nurse (LPN/VN)? a. Communicate the blood glucose level and insulin dose to the circulating nurse in surgery. b. Discuss the reason for the use of insulin therapy during the immediate postoperative period. c. Administer the prescribed lispro (Humalog) insulin before transporting the patient to surgery. d. Plan strategies to minimize the risk for hypoglycemia or hyperglycemia during the postoperative period.

ANS: C

4. After the nurse teaches the patient with stage 1 hypertension about diet modifications, which diet choice indicates that the teaching has been most effective? a. The patient avoids eating nuts or nut butters. b. The patient restricts intake of chicken and fish. c. The patient drinks low-fat milk with each meal. d. The patient has two cups of coffee in the morning.

ANS: C

4. The home health nurse notices irregular patterns of bruising at different stages of healing on an older patient's body. Which action should the nurse take first? a. Ensure the patient wears shoes with nonslip soles. b. Discourage using throw rugs throughout the house. c. Talk with the patient alone and ask about the bruising. d. Suggest that the health care provider prescribe radiographs.

ANS: C

4. The nurse is assessing a 22-yr-old patient experiencing the onset of symptoms of type 1 diabetes. To which question would the nurse anticipate a positive response? a. "Are you anorexic?" b. "Is your urine dark colored?" c. "Have you lost weight lately?" d. "Do you crave sugary drinks?"

ANS: C

4. Which question from the nurse during a patient interview will provide focused information about a possible thyroid disorder? a. "What methods do you use to help cope with stress?" b. "Have you experienced any blurring or double vision?" c. "Have you had a recent unplanned weight gain or loss?" d. "Do you have to get up at night to empty your bladder?"

ANS: C

40. An active 32-yr-old male who has type 1 diabetes is being seen in the endocrine clinic. Which finding indicates a need for the nurse to discuss a possible a change in therapy with the health care provider? a. Hemoglobin A1C level of 6.2% b. Heart rate at rest of 58 beats/min wait c. Blood pressure of 140/88 mmHg d. High-density lipoprotein (HDL) level of 65 mg/dL

ANS: C

41. A 30-yr-old patient has a new diagnosis of type 2 diabetes. When should the nurse recommend the patient schedule a dilated eye examination? a. Every 2 years b. Every 6 months c. As soon as available d. At the age of 39 years

ANS: C

42. Which patient statement indicates that the nurse's teaching about exenatide (Byetta) has been effective? a. "I may feel hungrier than usual when I take this medicine." b. "I will not need to worry about hypoglycemia with the Byetta." c. "I should take my daily aspirin at least an hour before the Byetta." d. "I will take the pill at the same time I eat breakfast in the morning."

ANS: C

5. A patient diagnosed with hypertension has been prescribed captopril. Which information is most important to teach the patient about this drug? a. Include high-potassium foods such as bananas in the diet. b. Increase fluid intake if dryness of the mouth is a problem. c. Change position slowly to help prevent dizziness and falls. d. Check the blood pressure in both arms before taking the drug.

ANS: C

5. A patient is scheduled in the outpatient clinic for blood cortisol testing. Which instruction should the nurse provide? a. "Avoid adding any salt to your foods for 24 hours before the test." b. "You will need to lie down for 30 minutes before the blood is drawn." c. "Come to the laboratory to have the blood drawn early in the morning." d. "Do not have anything to eat or drink before the blood test is obtained."

ANS: C

5. A patient with dark skin has been admitted to the hospital with acute decompensated heart failure. How would the nurse assess this patient for cyanosis? a. Assess the skin color of the earlobes. b. Apply pressure to the palms of the hands. c. Check the lips and oral mucous membranes. d. Examine capillary refill time of the nail beds.

ANS: C

6. The nurse prepares to obtain a culture from a patient who has a possible fungal infection on the foot. Which items should the nurse gather for this procedure? a. Sterile gloves b. Patch test instruments c. Cotton-tipped applicators d. Syringe and intradermal needle

ANS: C

6. Which statement by the patient to the home health nurse indicates a need for further teaching about self-administering eardrops? a. "I will leave the ear wick in place while administering the drops." b. "I will hold the tip of the dropper above the ear to administer the drops." c. "I will refrigerate the medication until I am ready to administer the drops." d. "I should lie down before and for 5 minutes after administering the drops."

ANS: C

7. A 62-yr-old patient who has no history of hypertension or other health problems suddenly develops a blood pressure (BP) of 198/110 mm Hg. After reconfirming the BP, it is appropriate for the nurse to tell the patient that: a. a BP recheck should be scheduled in a few weeks. b. dietary sodium and fat content should be decreased. c. diagnosis, treatment, and monitoring will be needed. d. there is danger of a stroke, requiring hospitalization.

ANS: C

7. An older patient who is being admitted to the hospital repeatedly asks the nurse to "speak up so that I can hear you." Which action should the nurse take? a. Increase the speaking volume. b. Overenunciate while speaking. c. Speak normally but more slowly. d. Use more facial expressions when talking.

ANS: C

8. A patient in the dermatology clinic is scheduled for removal of a 15-mm multicolored and irregular mole from the upper back. The nurse should prepare the patient for which type of biopsy? a. Shave biopsy b. Punch biopsy c. Incisional biopsy d. Excisional biopsy

ANS: C

8. A teenaged male patient who is on a wrestling team is examined by the nurse in the clinic. Which assessment finding would prompt the nurse to teach the patient about the importance of not sharing headgear to prevent the spread of pediculosis? a. Ringlike rashes with red, scaly borders over the entire scalp b. Red, hivelike papules and plaques with circumscribed borders c. Papular, wheal-like lesions with white deposits on the hair shaft d. Patchy areas of alopecia with small vesicles and excoriated areas

ANS: C

8. The nurse notes that a patient who was admitted with heart failure has jugular venous distention (JVD) when lying flat. Which follow-up action should the nurse take? a. Encourage the patient to drink more liquids. b. Check the apical and radial pulse for a pulse deficit. c. Observe the neck veins with the patient elevated 45 degrees. d. Have the patient bear down to perform the Valsalva maneuver.

ANS: C

8. Which laboratory value should the nurse review to determine whether a patient's hypothyroidism is caused by a problem with the anterior pituitary gland? a. Thyroxine (T4) level b. Triiodothyronine (T3) level c. Thyroid-stimulating hormone (TSH) level d. Thyrotropin-releasing hormone (TRH) level

ANS: C

9. Diltiazem (Cardizem) is prescribed for a patient with newly diagnosed Prinzmetal's (variant) angina. Which action of diltiazem is accurate for the nurse to include in the teaching plan? a. Reduces heart palpitations. b. Prevents coronary artery plaque. c. Decreases coronary artery spasms. d. Increases contractile force of the heart.

ANS: C

9. What information will a review of a patient's glycosylated hemoglobin (A1C) results provide to the nurse? a. Fasting preprandial glucose levels b. Glucose levels 2 hours after a meal c. Glucose control over the past 90 days d. Hypoglycemic episodes in the past 3 months

ANS: C

When assessing a new patient at the outpatient clinic, the nurse notes dry, scaly skin; thin hair; and thick, brittle nails. What is the nurse's most important action? a. Instruct the patient about the importance of nutrition for skin health. b. Make a referral to a podiatrist so that the nails can be safely trimmed. c. Consult with the health care provider about the need for further diagnostic testing. d. Teach the patient about using moisturizing creams and lotions to decrease dry skin

ANS: C

32. The nurse is interviewing a new patient with diabetes who takes rosiglitazone (Avandia). Which information would the nurse anticipate resulting in the health care provider discontinuing the medication? a. The patient's blood pressure is 154/92. b. The patient's blood glucose is 86 mg/dL. c. The patient reports a history of emphysema. d. The patient has chest pressure when walking.

ANS: D

3. Which action should the nurse take to prepare a patient for spirometry? a. Give the rescue medication immediately before testing. b. Administer oral corticosteroids 2 hours before the procedure. c. Withhold bronchodilators for 6 to 12 hours before the examination. d. Ensure that the patient has been NPO for several hours before the test.

ANS: C Bronchodilators are held before spirometry so that a baseline assessment of airway function can be determined. Testing is repeated after bronchodilator use to determine whether the decrease in lung function is reversible. There is no need for the patient to be NPO. Oral corticosteroids should be held before spirometry. Rescue medications (which are bronchodilators) would not be given until after the baseline pulmonary function was assessed.

35. After placement of a centrally inserted IV catheter, a patient reports acute chest pain and dyspnea. Which action should the nurse take first? a. Notify the health care provider. b. Offer reassurance to the patient. c. Auscultate the patient's breath sounds. d. Give prescribed PRN morphine sulfate IV

ANS: C The initial action should be to assess the patient further because the history and symptoms are consistent with several possible complications of central line insertion, including embolism and pneumothorax. The other actions may be appropriate, but further assessment of the patient is needed before notifying the health care provider, offering reassurance, or administration of morphine.

11. A patient is undergoing psoralen plus ultraviolet A light (PUVA) therapy for treatment of psoriasis. What action should the nurse take to prevent adverse effects from this procedure? a. Shield any unaffected areas with lead-lined drapes. b. Apply petroleum jelly to the areas around the lesions. c. Cleanse the skin carefully with antiseptic soap prior to PUVA. d. Have the patient use protective eyewear while receiving PUVA.

ANS: D

7. The nurse teaches a patient who is scheduled for a prostate needle biopsy about the procedure. Which statement by the patient indicates that teaching was effective? a. "The biopsy will remove the cancer in my prostate gland." b. "The biopsy will determine how much longer I have to live." c. "The biopsy will help decide the treatment for my enlarged prostate." d. "The biopsy will indicate whether the cancer has spread to other organ

ANS: C A biopsy is used to determine whether the prostate enlargement is benign or malignant and determines the type of treatment that will be needed. A biopsy does not give information about metastasis, life expectancy, or the impact of cancer on the patient's life.

11. Nadolol (Corgard) is prescribed for a patient with chronic stable angina and left ventricular dysfunction. What data would indicate to the nurse that the drug is effective? a. Decreased blood pressure and heart rate b. Improvement in the strength of the distal pulses c. Fewer complaints of having cold hands and feet d. Participation in daily activities without chest pain

ANS: D

12. The nurse interviews a patient with a new diagnosis of chronic obstructive pulmonary disease (COPD). Which information is specific in confirming a diagnosis of chronic bronchitis? a. The patient relates a family history of bronchitis. b. The patient has a 30 pack-year cigarette smoking history. c. The patient reports a productive cough for 3 months of every winter. d. The patient has respiratory problems that began during the past 12 months.

ANS: C A diagnosis of chronic bronchitis is based on a history of having a productive cough for 3 months for at least 2 consecutive years. There is no family tendency for chronic bronchitis. Although smoking is the major risk factor for chronic bronchitis, a smoking history does not confirm the diagnosis

14. A patient treated for human immunodeficiency virus (HIV) infection for 6 years has developed fat redistribution to the trunk with wasting of the arms, legs, and face. What recommendation will the nurse give to the patient? a. Review foods that are higher in protein. b. Teach about the benefits of daily exercise. c. Discuss a change in antiretroviral therapy. d. Talk about treatment with antifungal agents.

ANS: C A frequent first intervention for metabolic disorders is a change in antiretroviral therapy (ART). Treatment with antifungal agents would not be appropriate because there is no indication of fungal infection. Changes in diet or exercise have not proven helpful for this problem.

25. A patient who is receiving immunotherapy has just received an allergen injection. Which assessment finding is most important to communicate to the health care provider? a. The patient's IgG level is increased. b. The injection site is red and swollen. c. There is a 2-cm wheal at the site of the injection. d. The patient's symptoms did not improve in 2 months

ANS: C A local reaction larger than quarter size may indicate that a decrease in the allergen dose is needed. An increase in IgG shows that the therapy is effective. Redness and swelling at the site are not unusual. Because immunotherapy usually takes 1 to 2 years to achieve an effect, an improvement in the patient's symptoms is not expected after a few months.

40. The nurse notes that a patient has incisional pain, a poor cough effort, and scattered coarse crackles after a thoracotomy. Which action should the nurse take first? a. Assist the patient to sit upright in a chair. b. Splint the patient's chest during coughing. c. Medicate the patient with prescribed morphine. d. Observe the patient use the incentive spirometer.

ANS: C A major reason for atelectasis and poor airway clearance in patients after chest surgery is incisional pain (which increases with deep breathing and coughing). The first action by the nurse should be to medicate the patient to minimize incisional pain. The other actions are all appropriate ways to improve airway clearance but should be done after the morphine is given.

2. The nurse teaches a patient how to administer formoterol (Perforomist) through a nebulizer. Which action by the patient indicates good understanding of the teaching? a. The patient attaches a spacer before using the inhaler. b. The patient coughs vigorously after using the inhaler. c. The patient removes the facial mask when misting stops. d. The patient activates the inhaler at the onset of expiration.

ANS: C A nebulizer is used to administer aerosolized medication. A mist is seen when the medication is aerosolized, and when all the medication has been used, the misting stops. The other options refer to inhaler use. Coughing vigorously after inhaling and activating the inhaler at the onset of expiration are both incorrect techniques when using an inhaler.

9. A patient from a long-term care facility is admitted to the hospital with a sacral pressure injury. The base of the wound involves subcutaneous tissue. How should the nurse classify this pressure injury? a. Stage 1 b. Stage 2 c. Stage 3 d. Stage 4

ANS: C A stage 3 pressure injury has full-thickness skin damage and extends into the subcutaneous tissue. A stage 1 pressure injury has intact skin with some observable damage such as redness or a boggy feel. Stage 2 pressure injuries have partial-thickness skin loss. Stage 4 pressure injuries have full-thickness damage with tissue necrosis, extensive damage, or damage to bone, muscle, or supporting tissues.

22. The nurse reviews the medication orders for an older patient with arthritis in both hips who reports level 3 (0 to 10 scale) hip pain while ambulating. Which medication should the nurse offer as initial therapy? a. Naproxen 200 mg orally b. Oxycodone 5 mg orally c. Acetaminophen 650 mg orally d. Aspirin (acetylsalicylic acid) 650 mg orally

ANS: C Acetaminophen is the best first-choice medication. The principle of "start low, go slow" is used to guide therapy when treating older adults because the ability to metabolize medications is decreased and the likelihood of medication interactions is increased. Nonopioid analgesics are used first for mild to moderate pain, although opioids may be used later. Aspirin and nonsteroidal antiinflammatory drugs are associated with a high incidence of gastrointestinal bleeding in older patients.

3. A patient informed of a positive rapid screening test result for human immunodeficiency virus (HIV) is anxious and does not appear to hear what the nurse is saying. What action by the nurse is most important at this time? a. Inform the patient about the available treatments. b. Teach the patient how to manage a possible drug regimen. c. Remind the patient to return for retesting to verify the results. d. Ask the patient to identify those persons who had intimate contact.

ANS: C After an initial positive antibody test result, the next step is retesting to confirm the results. A patient who is anxious is not likely to be able to take in new information or be willing to disclose information about the HIV status of other individuals.

6. Which information about intradermal skin testing should the nurse teach to a patient with possible allergies? a. "Do not eat anything for about 6 hours before the testing." b. "Take an oral antihistamine about an hour before the testing." c. "Plan to wait in the clinic for 20 to 30 minutes after the testing." d. "Reaction to the testing will take about 48 to 72 hours to occur."

ANS: C Allergic reactions usually occur within minutes after injection of an allergen, and the patient will be monitored for at least 20 minutes for anaphylactic reactions after the testing. Medications that might modify the response, such as antihistamines, should be avoided before allergy testing. There is no reason to be NPO for skin testing. Results with intradermal testing occur within minutes.

11. The nurse in the eye clinic is examining a 67-yr-old patient who says, "I see small spots that move around in front of my eyes." Which action will the nurse take first? a. Immediately have the ophthalmologist evaluate the patient. b. Explain that spots and "floaters" are a normal part of aging. c. Warn the patient that these spots may indicate retinal damage. d. Use an ophthalmoscope to examine the posterior eye chambers.

ANS: D

17. A patient in metabolic alkalosis is admitted to the emergency department and pulse oximetry (SpO2) indicates that the O2 saturation is 94%. Which action should the nurse expect to take next? a. Complete a head-to-toe assessment. b. Administer an inhaled bronchodilator. c. Place the patient on high-flow oxygen. d. Obtain repeat arterial blood gases (ABGs)

ANS: C Although the O2 saturation is adequate, the left shift in the oxyhemoglobin dissociation curve will decrease the amount of O2 delivered to tissues, so high oxygen concentrations should be given. A head-to-toe assessment and repeat ABGs may be implemented later. Bronchodilators are not needed for metabolic alkalosis and there is no indication that the patient is having difficulty with airflow

4. The home health nurse cares for an alert and oriented older adult patient with a history of dehydration. Which instructions should the nurse give this patient related to fluid intake? a. "Drink more fluids in the late evening." b. "More fluids are needed if you feel thirsty." c. "Increase the fluids if your mouth feels dry." d. "If you feel confused, you need more fluids.

ANS: C An alert older patient will be able to self-assess for signs of oral dryness such as thick oral secretions or dry-appearing mucosa. The thirst mechanism decreases with age and is not an accurate indicator of volume depletion. Many older patients prefer to restrict fluids slightly in the evening to improve sleep quality. The patient will not be likely to notice and act appropriately when changes in level of consciousness occur.

23. The health care provider asks the nurse to evaluate whether a patient's angioedema has responded to prescribed therapies. Which assessment should the nurse perform? a. Obtain the patient's blood pressure and heart rate. b. Question the patient about any clear nasal discharge. c. Observe for swelling of the patient's lips and tongue. d. Assess the patient's extremities for wheal and flare lesions.

ANS: C Angioedema is characterized by swelling of the eyelids, lips, and tongue. Wheal and flare lesions, clear nasal drainage, and hypotension and tachycardia are characteristic of other allergic reactions.

31. The nurse takes an admission history on a patient with possible asthma who has new-onset wheezing and shortness of breath. Which information may indicate a need for a change in therapy? a. The patient has chronic inflammatory bowel disease. b. The patient has a history of pneumonia 6 months ago. c. The patient takes propranolol (Inderal) for hypertension. d. The patient uses acetaminophen (Tylenol) for headaches.

ANS: C B-Blockers such as propranolol can cause bronchospasm in some patients with asthma. The other information will be documented in the health history but does not indicate a need for a change in therapy.

27. Which finding indicates to the nurse that the administered nifedipine (Procardia) was effective for a patient who has idiopathic pulmonary arterial hypertension (IPAH)? a. Heart rate is between 60 and 100 beats/min. b. Patient's chest x-ray indicates clear lung fields. c. Patient reports a decrease in exertional dyspnea. d. Blood pressure (BP) is less than 140/90 mm Hg.

ANS: C Because a major symptom of IPAH is exertional dyspnea, an improvement in this symptom would indicate that the medication was effective. Nifedipine will affect BP and heart rate, but these parameters would not be used to monitor the effectiveness of therapy for a patient with IPAH. The chest x-ray will show clear lung fields even if the therapy is not effective

14. A 73-yr-old patient is admitted with pancreatitis. Which tool would be the most appropriate for the nurse to use during the admission assessment? a. Mini-Mental State Examination b. Drug Abuse Screening Test (DAST-10) c. Short Michigan Alcoholism Screening Test-Geriatric Version (SMAST-G) d. Clinical Institute Withdrawal Assessment of Alcohol Scale, Revised (CIWA-Ar)

ANS: C Because alcohol use is a common factor associated with the development of pancreatitis, the first assessment step is to screen for alcohol use using a validated screening questionnaire. The SMAST-G is a short-form alcoholism screening instrument tailored specifically to the needs of the older adult. If the patient scores positively on the SMAST-G, then the CIWA-Ar would be a useful tool for determining treatment. The DAST-10 provides more general information about substance use. The Mini-Mental State Examination screens for cognitive impairment.

35. The nurse receives change-of-shift report on the oncology unit. Which patient should the nurse assess first? a. A 35-yr-old patient who has wet desquamation associated with abdominal radiation b. A 42-yr-old patient who is sobbing after receiving a new diagnosis of ovarian cancer c. A 24-yr-old patient who received neck radiation and has blood oozing from the neck d. A 56-yr-old patient who developed a new pericardial friction rub after chest radiation

ANS: C Because neck bleeding may indicate possible carotid artery rupture in a patient who is receiving radiation to the neck, this patient should be seen first. The diagnoses and clinical manifestations for the other patients are not immediately life threatening.

19. A patient has inadequate nutrition due to painful oral ulcers. Which nursing action will be most effective in improving oral intake? a. Offer the patient frequent small snacks between meals. b. Assist the patient to choose favorite foods from the menu. c. Apply prescribed anesthetic gel to oral lesions before meals. d. Teach the patient about the importance of nutritional intake.

ANS: C Because the cause of the patient's poor nutrition is the painful oral ulcers, the best intervention is to apply anesthetic gel to the lesions before the patient eats. The other actions might be helpful for other patients with impaired nutrition but would not be as helpful for this patient

18. After the nurse has received change-of-shift report, which patient should the nurse assess first? a. A patient with pneumonia who has crackles in the right lung base. b. A patient with chronic bronchitis who has a low forced vital capacity. c. A patient with possible lung cancer who has just returned after bronchoscopy. d. A patient with hemoptysis and a 16-mm induration after tuberculin skin testing.

ANS: C Because the cough and gag are decreased after bronchoscopy, this patient should be assessed for airway patency. The other patients do not have clinical manifestations or procedures that require immediate assessment by the nurse.

12. A patient with an enlarging, irregular mole that is 7 mm in diameter is scheduled for outpatient treatment. The nurse should plan to prepare the patient for which procedure? a. Curettage b. Cryosurgery c. Punch biopsy d. Surgical excision

ANS: D

12. An 18-yr-old male patient with small stature is scheduled for a growth hormone stimulation test. What should the nurse obtain in preparation for the test? a. Ice in a basin b. Glargine insulin c. A cardiac monitor d. 50% dextrose solution

ANS: D

5. A patient with a tracheostomy has a new order for a fenestrated tracheostomy tube. Which action should the nurse include in the plan of care in collaboration with the speech therapist? a. Leave the tracheostomy inner cannula inserted at all times. b. Place the decannulation cap in the tube before cuff deflation. c. Assess the ability to swallow before using the fenestrated tube. d. Inflate the tracheostomy cuff during use of the fenestrated tube.

ANS: C Because the cuff is deflated when using a fenestrated tube, the patient's risk for aspiration should be assessed before changing to a fenestrated tracheostomy tube. The decannulation cap is never inserted before cuff deflation because to do so would obstruct the patient's airway. The cuff is deflated and the inner cannula removed to allow air to flow across the patient's vocal cords when using a fenestrated tube.

7. The nurse reviewing a clinic patient's medical record notes that the patient missed the previous appointment for weekly immunotherapy. Which action by the nurse is appropriate? a. Schedule an additional dose the following week. b. Administer the scheduled dosage of the allergen. c. Consult with the health care provider about giving a lower allergen dose. d. Reevaluate the patient's sensitivity to the allergen with a repeat skin test.

ANS: C Because there is an increased risk for adverse reactions after a patient misses a scheduled dose of allergen, the nurse should check with the health care provider before administration of the injection. A skin test is used to identify the allergen and would not be used at this time. An additional dose for the week may increase the risk for a reaction.

19. Postural drainage with percussion and vibration is ordered twice daily for a patient with chronic obstructive pulmonary disease. Which intervention should the nurse include in the plan of care? a. Schedule the procedure 1 hour after the patient eats. b. Maintain the patient in the lateral position for 20 minutes. c. Give the prescribed albuterol (Ventolin HFA) before the therapy. d. Perform percussion before assisting the patient to the drainage position

ANS: C Bronchodilators are administered before chest physiotherapy. Postural drainage, percussion, and vibration should be done 1 hour before or 3 hours after meals. Patients remain in each postural drainage position for 5 minutes. Percussion is done while the patient is in the postural drainage position.

9. The nurse is caring for a patient with chronic obstructive pulmonary disease (COPD). Which information would prompt the nurse to consult with the health care provider before administering the prescribed theophylline? a. The patient reports a recent 15 pound weight gain. b. The patient denies shortness of breath at present. c. The patient takes cimetidine (Tagamet HB) daily. d. The patient reports coughing up some green mucus.

ANS: C Cimetidine interferes with the metabolism of theophylline, and concomitant administration may lead rapidly to theophylline toxicity. The other patient information would not affect whether the theophylline should be administered or not

15. The nurse is caring for a patient with cor pulmonale. The nurse should monitor the patient for which expected finding? a. Chest pain b. Finger clubbing c. Peripheral edema d. Elevated temperature

ANS: C Cor pulmonale causes clinical manifestations of right ventricular failure, such as peripheral edema. The other clinical manifestations may occur in the patient with other complications of chronic obstructive pulmonary disease but are not indicators of cor pulmonale.

43. A patient has acute bronchitis with a nonproductive cough and wheezes. Which topic should the nurse plan to include in the teaching plan? a. Purpose of antibiotic therapy b. Ways to limit oral fluid intake c. Appropriate use of cough suppressants d. Safety concerns with home O2 therapy

ANS: C Cough suppressants are frequently prescribed for acute bronchitis. Because most acute bronchitis is viral in origin, antibiotics are not prescribed unless there are systemic symptoms. Fluid intake is encouraged. Home O2 is not prescribed for acute bronchitis, although it may be used for chronic bronchitis.

3. A patient with bacterial pneumonia has coarse crackles and thick sputum. Which action should the nurse plan to promote airway clearance? a. Restrict oral fluids during the day. b. Encourage pursed-lip breathing technique. c. Help the patient to splint the chest when coughing. d. Encourage the patient to wear the nasal O2 cannula.

ANS: C Coughing is less painful and more likely to be effective when the patient splints the chest during coughing. Fluids should be encouraged to help liquefy secretions. Nasal O2 will improve gas exchange but will not improve airway clearance. Pursed-lip breathing can improve gas exchange in patients with chronic obstructive pulmonary disease but will not improve airway clearance

12. A patient diagnosed with active tuberculosis (TB) is homeless and has a history of chronic alcohol use. Which intervention by the nurse will be most effective in ensuring adherence with the treatment regimen? a. Repeat warnings about the high risk for infecting others several times. b. Give the patient written instructions about how to take the medications. c. Arrange for a daily meal and drug administration at a community center. d. Arrange for the patient's friend to administer the medication on schedule.

ANS: C Directly observed therapy is the most effective means for ensuring compliance with the treatment regimen. Arranging a daily meal will help ensure that the patient is available to receive the medication. The other nursing interventions may be appropriate for some patients but are not likely to be as helpful for this patient's situation.

11. A patient with chronic obstructive pulmonary disease (COPD) has been eating very little and has lost weight. Which intervention would be most appropriate for the nurse to include in the plan of care? a. Encourage increased intake of whole grains. b. Increase the patient's menu order of fruits and fruit juices. c. Offer high-calorie protein snacks between meals and at bedtime. d. Assist the patient in choosing foods with high vegetable content.

ANS: C Eating small amounts more often (as occurs with snacking) will increase caloric intake by decreasing the fatigue and feelings of fullness associated with large meals. Patients with COPD should rest before meals. Foods that have a lot of texture such as whole grains may take more energy to eat and get absorbed and lead to decreased intake. Although fruits, juices, and minerals are not contraindicated, foods high in protein are a better choice

12. The nurse notes darker skin pigmentation in the skinfolds of a middle-aged patient who has a body mass index of 40 kg/m2 . What is the nurse's appropriate action? a. Discuss the use of drying agents to minimize infection risk. b. Instruct the patient about the use of mild soap to clean skinfolds. c. Teach the patient about treating fungal infections in the skinfolds. d. Ask the patient about a personal or family history of type 2 diabetes.

ANS: D

14. The long-term care nurse is evaluating the effectiveness of protein supplements for an older resident who has a low serum total protein level. Which assessment finding indicates that the patient's condition has improved? a. Hematocrit 28% b. Absence of skin tenting c. Decreased peripheral edema d. Blood pressure 110/72 mm Hg

ANS: C Edema is caused by low oncotic pressure in individuals with low serum protein levels. The decrease in edema indicates an improvement in the patient's protein status. Good skin turgor is an indicator of fluid balance, not protein status. A low hematocrit could be caused by poor protein intake. Blood pressure does not provide a useful clinical tool for monitoring protein status.

41. After change-of-shift report on the oncology unit, which patient should the nurse assess first? a. Patient who has a platelet count of 82,000/µL after chemotherapy. b. Patient who has xerostomia after receiving head and neck radiation. c. Patient who is neutropenic and has a temperature of 100.5° F (38.1° C). d. Patient who is worried about getting the prescribed long-acting opioid on time

ANS: C Fever is an emergency in neutropenic patients because of the risk for rapid progression to severe infections and sepsis. The other patients also require assessments or interventions but do not need to be assessed as urgently. Patients with thrombocytopenia do not have spontaneous bleeding until the platelets are 20,000/µL. Xerostomia does not require immediate intervention. Although breakthrough pain needs to be addressed rapidly, the patient does not appear to have breakthrough pain.

7. The nurse completes a shift assessment on a patient admitted in the early phase of heart failure. Which sounds would the nurse most likely hear on auscultation? a. Continuous rumbling, snoring, or rattling sounds mainly on expiration b. Continuous high-pitched musical sounds on inspiration and expiration c. Discontinuous high-pitched sounds of short duration during inspiration d. A series of long-duration, discontinuous, low-pitched sounds during inspiration

ANS: C Fine crackles are likely to be heard in the early phase of heart failure. Fine crackles are discontinuous, high-pitched sounds of short duration heard on inspiration. Course crackles are a series of long-duration, discontinuous, low-pitched sounds during inspiration. Wheezes are continuous high-pitched musical sounds on inspiration and expiration.

21. A patient who has severe pain with terminal pancreatic cancer is being cared for at home by family members. Which finding by the nurse indicates that teaching about pain management has been effective? a. The patient uses the ordered opioid pain medication whenever the pain is greater than 5 (0 to 10 scale). b. The patient agrees to take the medications by the IV route to improve analgesic effectiveness. c. The patient takes opioids around the clock on a regular schedule and uses additional doses when breakthrough pain occurs. d. The patient states that nonopioid analgesics may be used if the maximal dose of the opioid is reached without adequate pain relief

ANS: C For chronic cancer pain, analgesics should be taken on a scheduled basis, with additional doses as needed for breakthrough pain. Taking the medications only when pain reaches a certain level does not provide effective pain control. Although nonopioid analgesics may also be used, there is no maximum dose of opioid. Opioids are given until pain control is achieved. The IV route is not more effective than the oral route, and usually the oral route is preferred

9. The nurse teaches a patient about pulmonary spirometry testing. Which statement by the patient indicates teaching was effective? a. "I should use my inhaler right before the test." b. "I won't eat or drink anything 8 hours before the test." c. "I will inhale deeply and blow out hard during the test." d. "My blood pressure and pulse will be checked every 15 minutes."

ANS: C For spirometry, the patient should inhale deeply and exhale as long, hard, and fast as possible. The other actions are not needed. The administration of inhaled bronchodilators should be avoided 6 hours before the procedure.

25. The nurse provides preoperative instruction for a patient scheduled for a left pneumonectomy. Which information should the nurse include about the patient's postoperative care? a. Bed rest for the first 24 hours b. Positioning only on the right side c. Frequent use of an incentive spirometer d. Chest tube placement to continuous suction

ANS: C Frequent deep breathing and coughing are needed after chest surgery to prevent atelectasis. To promote gas exchange, patients after pneumonectomy are positioned on the surgical side. Early mobilization decreases the risk for postoperative complications such as pneumonia and deep vein thrombosis. In a pneumonectomy, chest tubes may or may not be placed in the space from which the lung was removed. If a chest tube is used, it is clamped and only released by the surgeon to adjust the volume of serosanguineous fluid that will fill the space vacated by the lung. If the cavity overfills, it could compress the remaining lung and compromise the cardiovascular and pulmonary function. Daily chest x-rays can be used to assess the volume and space.

16. An older adult patient has a prescription for cyclosporine following a kidney transplant. Which information in the patient's health history has implications for planning patient teaching about the safe use of cyclosporine? a. The patient restricts salt to 2 grams per day. b. The patient eats green leafy vegetables daily. c. The patient drinks grapefruit juice every day. d. The patient drinks 3 to 4 quarts of fluid each day.

ANS: C Grapefruit juice can increase the toxicity of cyclosporine. The patient should be taught to avoid grapefruit juice. Normal fluid and sodium intake or eating green leafy vegetables will not affect cyclosporine levels or renal function

11. A patient who has a history of chronic obstructive pulmonary disease (COPD) was hospitalized for increasing shortness of breath and chronic hypoxemia (SaO2 levels of 89% to 90%). In planning for discharge, which action by the nurse will be most effective in improving compliance with discharge teaching? a. Have the patient repeat the instructions immediately after teaching. b. Accomplish the patient teaching just before the scheduled discharge. c. Arrange for the patient's caregiver to be present during the teaching. d. Start giving the patient discharge teaching during the admission process.

ANS: C Hypoxemia interferes with the patient's ability to learn and retain information, so having the patient's caregiver present will increase the likelihood that discharge instructions will be followed. Having the patient repeat the instructions will indicate that the information is understood at the time, but it does not guarantee retention of the information. Because the patient is likely to be distracted just before discharge, giving discharge instructions just before discharge is not ideal. The patient is likely to be anxious and even more hypoxemic than usual on the day of admission, so teaching about discharge should be postpone

4. A nurse assesses a patient with chronic cancer pain who is receiving imipramine (Tofranil) in addition to long-acting morphine (MS Contin). Which statement, if made by the patient, indicates to the nurse that the patient is receiving adequate pain control? a. "I'm not anxious during the day." b. "Every night I get 8 hours of sleep." c. "I can accomplish activities without much discomfort." d. "I feel less depressed since I've been taking the Tofranil."

ANS: C Imipramine is being used in this patient to manage chronic pain and improve functional ability. Although the medication is also prescribed for patients with depression, insomnia, and anxiety, the evaluation for this patient is based on improved pain control and activity level.

24. A nurse is assessing a newly admitted patient with chronic heart failure who forgot to take prescribed medications. The patient seems confused and short of breath with peripheral edema. Which assessment should the nurse complete first? a. Skin turgor b. Heart sounds c. Mental status d. Capillary refill

ANS: C Increases in extracellular fluid (ECF) can lead to swelling of cells in the central nervous system, initially causing confusion, which may progress to coma or seizures. Although skin turgor, capillary refill, and heart sounds may also be affected by increases in ECF, these are signs that do not have as immediate impact on patient outcomes as cerebral edema.

10. A patient who had a total laryngectomy has previously expressed hopelessness about the loss of control over personal care. Which information obtained by the nurse indicates that this identified problem is resolving? a. The patient allows the nurse to suction the tracheostomy. b. The patient's spouse provides the daily tracheostomy care. c. The patient asks to learn how to clean the tracheostomy stoma. d. The patient uses a communication board to request "No Visitors."

ANS: C Independently caring for the laryngectomy tube indicates that the patient has regained control of personal care and hopelessness is at least partially resolved. Letting the nurse and spouse provide care and requesting no visitors may indicate that the patient is still experiencing hopelessness.

29. The nurse is caring for a patient diagnosed with stage I colon cancer. When assessing the need for psychologic support, which question by the nurse will provide the most information? a. "How long ago were you diagnosed with this cancer?" b. "Do you have any concerns about body image changes?" c. "Can you tell me what has been helpful when coping with past stressful events?" d. "Are you familiar with the stages of emotional adjustment to cancer of the colon?"

ANS: C Information about how the patient has coped with past stressful situations helps the nurse determine usual coping mechanisms and their effectiveness. The length of time since the diagnosis will not provide much information about the patient's need for support. The patient's knowledge of typical stages in adjustment to a critical diagnosis does not provide insight into patient needs for assistance. Because surgical interventions for stage I cancer of the colon may not cause any body image changes, this question is not appropriate at this time.

4. A young adult patient receiving antibiotics for an infected leg wound has a temperature of 101.8° F (38.7° C). The patient denies any discomfort. Which action by the nurse is appropriate? a. Apply a cooling blanket. b. Notify the health care provider. c. Check the patient's temperature again in 4 hours. d. Give acetaminophen prescribed as-needed for pain.

ANS: C Mild to moderate temperature elevations (less than 103° F) do not harm young adult patients and may benefit host defense mechanisms. Continue to monitor the temperature. Antipyretics are not indicated unless the patient has fever-related symptoms, and the patient does not require analgesics if not reporting discomfort. There is no need to notify the patient's health care provider of a fever in a patient who is already being treated for the infection or to use a cooling blanket for a moderate temperature elevation

18. A lobectomy is scheduled for a patient with stage I non-small cell lung cancer. The patient tells the nurse, "I would rather have chemotherapy than surgery." Which response by the nurse is most appropriate? a. "Are you afraid that the surgery will be very painful?" b. "Did you have bad experiences with previous surgeries?" c. "Tell me what you know about the treatments available." d. "Surgery is the treatment of choice for stage I lung cancer."

ANS: C More assessment of the patient's concerns about surgery is indicated. An open-ended response will elicit the most information from the patient. The answer beginning, "Surgery is the treatment of choice" is accurate, but it discourages the patient from sharing concerns about surgery. The remaining two answers indicate that the nurse has jumped to conclusions about the patient's reasons for not wanting surgery. Chemotherapy is the primary treatment for small cell lung cancer. In non-small cell lung cancer, chemotherapy may be used in the treatment of nonresectable tumors or as adjuvant therapy to surgery.

4. A patient who is diagnosed with acquired immunodeficiency syndrome (AIDS) tells the nurse, "I feel obsessed with morbid thoughts about dying." Which response by the nurse is appropriate? a. "Thinking about dying will not improve the course of AIDS." b. "Do you think that taking an antidepressant might be helpful?" c. "Can you tell me more about the thoughts that you are having?" d. "It is important to focus on the good things about your life now."

ANS: C More assessment of the patient's psychosocial status is needed before taking any other action. The statements, "Thinking about dying will not improve the course of AIDS" and "It is important to focus on the good things in life" or suggesting an antidepressant discourage the patient from sharing any further information with the nurse and decrease the nurse's ability to develop a trusting relationship with the patient.

3. The nurse teaches a student nurse about the action of ibuprofen. Which statement, if made by the student, indicates that teaching was effective? a. "The drug decreases pain impulses in the spinal cord." b. "The drug decreases sensitivity of the brain to painful stimuli." c. "The drug decreases production of pain-sensitizing chemicals." d. "The drug decreases the modulating effect of descending nerves."

ANS: C Nonsteroidal antiinflammatory drugs (NSAIDs) provide analgesic effects by decreasing the production of pain-sensitizing chemicals such as prostaglandins at the site of injury. Transmission of impulses through the spinal cord, brain sensitivity to pain, and the descending nerve pathways are not affected by NSAIDs.

15. The health care provider has prescribed the following medications for a middle-aged patient who uses long-acting morphine (MS Contin) for chronic back pain but still has ongoing pain. Which medication should the nurse question? a. Morphine b. Dexamethasone c. Pentazocine (Talwin) d. Celecoxib (Celebrex)

ANS: C Opioid agonist-antagonists can precipitate withdrawal if used in a patient who is physically dependent on mu agonist drugs such as morphine. The other medications are appropriate for chronic back pain.

10. A patient who is taking rifampin (Rifadin) for tuberculosis calls the clinic and reports having orange discolored urine and tears. Which response by the nurse reflects accurate knowledge about the medication and the patient's illness? a. Ask the patient about any visual changes in red-green color discrimination. b. Question the patient about experiencing shortness of breath, hives, or itching. c. Explain that orange discolored urine and tears are normal while taking this medication. d. Advise the patient to stop the drug and report the symptoms to the health care provider.

ANS: C Orange-colored body secretions are a side effect of rifampin. The patient does not have to stop taking the medication. The findings are not indicative of an allergic reaction. Alterations in red-green color discrimination commonly occur when taking ethambutol, which is a different tuberculosis medication

22. A patient has a chest wall contusion as a result of being struck in the chest with a baseball bat. Which initial assessment finding is of most concern to the emergency department nurse? a. Report of chest wall pain b. Heart rate of 110 beats/min c. Paradoxical chest movement d. Large bruised area on the chest

ANS: C Paradoxical chest movement indicates that the patient may have flail chest, which can severely compromise gas exchange and can rapidly lead to hypoxemia. Chest wall pain, a slightly elevated pulse rate, and chest bruising all require further assessment or intervention, but the priority concern is poor gas exchange.

11. A patient who uses injectable illegal drugs asks the nurse how to prevent acquired immunodeficiency syndrome (AIDS). Which response by the nurse is most accurate? a. "Clean drug injection equipment before each use." b. "Ask those who share equipment to be tested for HIV." c. "Consider participating in a needle-exchange program." d. "Avoid sexual intercourse when using injectable drugs."

ANS: C Participation in needle-exchange programs has been shown to decrease and control the rate of HIV infection. Cleaning drug equipment before use also reduces risk, but it might not be consistently practiced. HIV antibodies do not appear for several weeks to months after exposure, so testing drug users would not be very effective in reducing risk for HIV exposure. HIV can be transmitted through both intercourse and injection.

9. The nurse palpates enlarged cervical lymph nodes on a patient diagnosed with acute human immunodeficiency virus (HIV) infection. Which action would be appropriate for the nurse to take? a. Instruct the patient to apply ice to the neck. b. Tell the patient a secondary infection is present. c. Explain to the patient that this is an expected finding. d. Request that an antibiotic be prescribed for the patient

ANS: C Persistent generalized lymphadenopathy is common in the early stages of HIV infection. No antibiotic is needed because the enlarged nodes are probably not caused by bacteria. Lymphadenopathy is common with acute HIV infection and is therefore not likely to represent an additional infection. Ice will not decrease the swelling in persistent generalized lymphadenopathy

24. A nurse has obtained donor tissue typing information about a patient who is waiting for a kidney transplant. Which results should be reported to the transplant surgeon? a. Patient is Rh positive and donor is Rh negative. b. Six antigen matches are present in HLA typing. c. Results of patient-donor crossmatching are positive. d. Panel of reactive antibodies (PRA) percentage is low

ANS: C Positive crossmatching is an absolute contraindication to kidney transplantation because a hyperacute rejection will occur after the transplant. The other information shows that the tissue match between the patient and potential donor is acceptable.

20. After the home health nurse teaches a patient's family member about how to care for a sacral pressure injury, which finding indicates that additional teaching is needed? a. The family member uses a lift sheet to reposition the patient. b. The family member uses clean tap water to clean the wound. c. The family member dries the wound using a hair dryer on a low setting. d. The family member places contaminated dressings in a plastic grocery bag.

ANS: C Pressure injuries need to be moist to facilitate wound healing. The other actions indicate a good understanding of pressure ulcer care. The use of lift sheets prevents shearing forces. Clean tap water is acceptable for home use on chronic pressure wounds. Proper disposal of contaminated dressings prevents the spread of infection.

16. An occupational health nurse works at a manufacturing plant where there is potential exposure to inhaled dust. Which action recommended by the nurse is intended to prevent lung disease? a. Treat workers with pulmonary fibrosis. b. Teach about symptoms of lung disease. c. Require the use of protective equipment. d. Monitor workers for coughing and wheezing.

ANS: C Prevention of lung disease requires the use of appropriate protective equipment such as masks. The other actions will help in recognition or early treatment of lung disease but will not be effective in prevention of lung damage. Repeated exposure eventually results in diffuse pulmonary fibrosis. Fibrosis is the result of tissue repair after inflammation

6. Which exposure by the nurse is most likely to require postexposure prophylaxis when the patient's human immunodeficiency virus (HIV) status is unknown? a. Bite to the arm that does not result in open skin b. Splash into the eyes while emptying a bedpan containing stool c. Needle stick with a needle and syringe used for a venipuncture d. Contamination of open skin lesions with patient vaginal secretions

ANS: C Puncture wounds are the most common means for workplace transmission of blood-borne diseases, and a needle with a hollow bore that had been contaminated with the patient's blood would be a high-risk situation. The other situations described would be much less likely to result in transmission of the virus

10. External-beam radiation is planned for a patient with cervical cancer. What instructions should the nurse give to the patient to prevent complications from the effects of the radiation? a. Test all stools for the presence of blood. b. Maintain a high-residue, high-fiber diet. c. Clean the perianal area carefully after every bowel movement. d. Inspect the mouth and throat daily for the appearance of thru

ANS: C Radiation to the abdomen will affect organs in the radiation path, such as the bowel, and cause frequent diarrhea. Careful cleaning of this area will help decrease the risk for skin breakdown and infection. Stools are likely to have occult blood from the inflammation associated with radiation, so routine testing of stools for blood is not indicated. Radiation to the abdomen will not cause stomatitis. A low-residue diet is recommended to avoid irritation of the bowel when patients receive abdominal radiation

7. The nurse teaches a patient who has asthma about peak flowmeter use. Which action by the patient indicates that teaching was successful? a. The patient inhales rapidly through the peak flowmeter mouthpiece. b. The patient takes montelukast (Singulair) for peak flows in the red zone. c. The patient uses albuterol (Ventolin HFA) for peak flows in the yellow zone. d. The patient calls the health care provider when the peak flow is in the green zone.

ANS: C Readings in the yellow zone indicate a decrease in peak flow. The patient should use short-acting B2-adrenergic (SABA) medications. Readings in the green zone indicate good asthma control. The patient should exhale quickly and forcefully through the peak flowmeter mouthpiece to obtain the readings. Readings in the red zone do not indicate good peak flow, and the patient should take a fast-acting bronchodilator and call the health care provider for further instructions. Singulair is not indicated for acute attacks but is used for maintenance therapy

6. A patient with a chronic cough is scheduled to have a bronchoscopy with biopsy. Which intervention will the nurse implement directly after the procedure? a. Encourage the patient to drink clear liquids. b. Place the patient on bed rest for at least 4 hours. c. Keep the patient NPO until the gag reflex returns. d. Maintain the head of the bed elevated 90 degrees.

ANS: C Risk for aspiration and maintaining an open airway is the priority. Because a local anesthetic is used to suppress the gag and cough reflexes during bronchoscopy, the nurse should monitor for the return of these reflexes before allowing the patient to take oral fluids or food. The patient does not need to be on bed rest, and the head of the bed does not need to be in the high-Fowler's position.

22. The nurse designs a program to decrease the incidence of human immunodeficiency virus (HIV) infection in the adolescent and young adult populations. Which information should the nurse assign as the highest priority? a. Methods to prevent perinatal HIV transmission. b. Ways to sterilize needles used by injectable drug users. c. Prevention of HIV transmission between sexual partners. d. Means to prevent transmission through blood transfusions.

ANS: C Sexual transmission is the most common way that HIV is transmitted. The nurse should also provide teaching about perinatal transmission, needle sterilization, and blood transfusion, but the rate of HIV infection associated with these situations is lower.

23. When preparing a clinic patient who has chronic obstructive pulmonary disease (COPD) for pulmonary spirometry, what question the nurse should ask? a. "Are you claustrophobic?" b. "Are you allergic to shellfish?" c. "Have you taken any bronchodilators today?" d. "Do you have any metal implants or prostheses?"

ANS: C Spirometry will help establish the COPD diagnosis. Bronchodilators should be avoided at least 6 hours before the test. Spirometry does not involve being placed in an enclosed area such as for magnetic resonance imaging (MRI). Contrast dye is not used for spirometry. The patient may still have spirometry done if metal implants or prostheses are present because they are contraindications for an MRI

7. The health care provider writes an order for bacteriologic testing for a patient who has a positive tuberculosis skin test. Which action should the nurse take? a. Teach about the reason for the blood tests. b. Schedule an appointment for a chest x-ray. c. Teach the patient about providing specimens for 3 consecutive days. d. Instruct the patient to collect several separate sputum specimens today

ANS: C Sputum specimens are obtained on 2 to 3 consecutive days for bacteriologic testing for Mycobacterium tuberculosis. The patient should not provide all the specimens at once. Blood cultures are not used to test for tuberculosis. A chest x-ray is not bacteriologic testing. Although the findings on chest x-ray examination are important, it is not possible to make a diagnosis of TB solely based on chest x-ray findings because other diseases can mimic the appearance of TB.

9. The nurse teaches a patient about the transmission of pulmonary tuberculosis (TB). Which statement, if made by the patient, indicates that teaching was effective? a. "I will take the bus instead of driving." b. "I will stay indoors whenever possible." c. "My spouse will sleep in another room." d. "I will keep the windows closed at home."

ANS: C Teach the patient how to minimize exposure to close contacts and household members. Homes should be well ventilated, especially the areas where the infected person spends a lot of time. While still infectious, the patient should sleep alone, spend as much time as possible outdoors, and minimize time in congregate settings or on public transportation.

2. The nurse is caring for a patient receiving intravesical bladder chemotherapy. The nurse should monitor for which adverse effect? a. Nausea b. Alopecia c. Hematuria d. Xerostomia

ANS: C The adverse effects of intravesical chemotherapy are confined to the bladder. The other adverse effects are associated with systemic chemotherapy.

12. Which nursing action will be most useful in assisting a young adult to adhere to a newly prescribed antiretroviral therapy (ART) regimen? a. Give the patient detailed information about possible medication side effects. b. Remind the patient of the importance of taking the medications as scheduled. c. Help the patient develop a schedule to decide when the drugs should be taken. d. Encourage the patient to join a support group for adults who are HIV positive.

ANS: C The best approach to improve adherence is to learn about important activities in the patient's life and adjust the ART around those activities. The other actions are also useful, but they will not improve adherence as much as individualizing the ART to the patient's schedule.

17. A patient who has ovarian cancer is crying and tells the nurse, "My husband rarely visits. He just doesn't care." The husband tells the nurse that he does not know what to say to his wife. Which problem is appropriate for the nurse to address in the plan of care? a. Anxiety b. Death anxiety c. Difficulty coping d. Lack of knowledge

ANS: C The data indicate that difficulty coping with the situation may be present reflected by the poor communication among the family members. The data given does not suggest death anxiety, anxiety, or lack of knowledge as an etiology.

22. An older adult patient receiving iso-osmolar continuous enteral nutrition develops restlessness, agitation, and weakness. Which laboratory result should the nurse report to the health care provider immediately? a. K + 3.4 mEq/L (3.4 mmol/L) b. Ca+2 7.8 mg/dL (1.95 mmol/L) c. Na+ 154 mEq/L (154 mmol/L) d. PO4 ?2-3 4.8 mg/dL (1.55 mmol/L)

ANS: C The elevated serum sodium level is consistent with the patient's neurologic symptoms and indicates a need for immediate action to prevent further serious complications such as seizures. The potassium, phosphate, and calcium levels vary slightly from normal and should be reported, but do not require immediate action by the nurse

14. After a laryngectomy, a patient coughs violently during suctioning and dislodges the tracheostomy tube. Which action should the nurse take first? a. Arrange for arterial blood gases to be drawn immediately. b. Cover stoma with sterile gauze and ventilate through stoma. c. Attempt to reinsert the tracheostomy tube with the obturator in place. d. Assess the patient's oxygen saturation and notify the health care provider

ANS: C The first action should be to attempt to reinsert the tracheostomy tube to maintain the patient's airway. Covering the stoma with a dressing and manually ventilating the patient may be an appropriate action if the nurse is unable to reinsert the tracheostomy tube. Assessing the patient's oxygenation is an important action, but it is not as appropriate until there is an established airway.

13. After 2 months of tuberculosis (TB) treatment with isoniazid, rifampin (Rifadin), pyrazinamide, and ethambutol, a patient continues to have positive sputum smears for acid-fast bacilli (AFB). Which action should the nurse take next? a. Teach about drug-resistant TB. b. Schedule directly observed therapy. c. Ask the patient whether medications have been taken as directed. d. Discuss the need for an injectable antibiotic with the health care provider.

ANS: C The first action should be to determine whether the patient has been compliant with drug therapy because negative sputum smears would be expected if the TB bacillus is susceptible to the medications and if the medications have been taken correctly. Assessment is the first step in the nursing process. Depending on whether the patient has been compliant or not, different medications or directly observed therapy may be indicated. The other options are interventions based on assumptions until an assessment has been completed.

33. A patient with cystic fibrosis has blood glucose levels that are consistently between 180 to 250 mg/dL. Which action will the nurse expect to implement? a. Discuss the role of diet in blood glucose control. b. Evaluate the patient's use of pancreatic enzymes. c. Teach the patient about administration of insulin. d. Give oral hypoglycemic medications before meals.

ANS: C The glucose levels indicate that the patient has developed cystic fibrosis (CF) related diabetes, and insulin therapy is required. Because the etiology of diabetes in CF is inadequate insulin production, oral hypoglycemic agents are not effective. Patients with CF need a high-calorie diet. Inappropriate use of pancreatic enzymes would not be a cause of hyperglycemia in a patient with CF

41. The nurse is caring for a patient with idiopathic pulmonary arterial hypertension (IPAH). Which assessment information requires the most immediate action by the nurse? a. The O2 saturation is 90%. b. The blood pressure is 98/56 mm Hg. c. The epoprostenol (Flolan) infusion is disconnected. d. The international normalized ratio (INR) is prolonged.

ANS: C The half-life of this drug is 6 minutes, so the nurse will need to restart the infusion as soon as possible to prevent rapid clinical deterioration. The other data also indicate a need for ongoing monitoring or intervention, but the priority action is to reconnect the infusion.

44. Which action by the nurse will be most effective in decreasing the spread of pertussis in a community setting? a. Providing supportive care to patients diagnosed with pertussis b. Teaching family members about the need for careful hand washing c. Teaching patients about the need for adult pertussis immunizations d. Encouraging patients to complete the prescribed course of antibiotics

ANS: C The increased rate of pertussis in adults is thought to be caused by decreasing immunity after childhood immunization. Immunization is the most effective method of protecting communities from infectious diseases. Hand washing should be taught, but pertussis is spread by droplets and contact with secretions. Supportive care does not shorten the course of the disease or the risk for transmission. Taking antibiotics as prescribed does assist with decreased transmission, but patients are likely to have already transmitted the disease by the time the diagnosis is made.

20. A patient is anxious and reports difficulty breathing after being stung by a wasp. What is the nurse's priority action? a. Provide high-flow oxygen. b. Administer antihistamines. c. Assess the patient's airway. d. Remove the stinger from the site.

ANS: C The initial action with any patient with difficulty breathing is to assess and maintain the airway. The patient's symptoms of anxiety and difficulty breathing may have other causes than anaphylaxis, so additional assessment is warranted. The other actions are part of the emergency management protocol for anaphylaxis, but the priority is airway assessment and maintenance.

21. The nurse monitors a patient in the emergency department after chest tube placement for a hemopneumothorax. Which assessment finding is of most concern? a. A large air leak in the water-seal chamber b. Report of pain with each deep inspiration c. 400 mL of blood in the collection chamber d. Subcutaneous emphysema at the insertion site

ANS: C The large amount of blood may indicate that the patient is in danger of developing hypovolemic shock. An air leak would be expected after chest tube placement for a pneumothorax. Initially, brisk bubbling of air occurs in this chamber when a pneumothorax is evacuated. The pain should be treated but is not as urgent a concern as the possibility of continued hemorrhage. Subcutaneous emphysema should be monitored but is not unusual in a patient with pneumothorax. A small amount of subcutaneous air is harmless and will be reabsorbed.

36. After receiving change-of-shift report, which patient should the nurse assess first? a. Patient with serum sodium level of 145 mEq/L who is asking for water b. Patient with serum potassium level of 5.0 mEq/L who reports abdominal cramping c. Patient with serum magnesium level of 1.1 mEq/L who has tremors and hyperactive reflexes d. Patient with serum phosphorus level of 4.5 mg/dL who has soft tissue calcium-phosphate precipitates

ANS: C The low magnesium level and neuromuscular irritability suggest that the patient may be at risk for seizures. The other patients have mild electrolyte disturbances or symptoms that require action, but they are not at risk for life-threatening complications.

1. A patient who is scheduled for a breast biopsy asks the nurse the difference between a benign tumor and a malignant tumor. Which answer by the nurse is accurate? a. "Benign tumors do not cause damage to other tissues." b. "Benign tumors are likely to recur in the same location." c. "Malignant tumors may spread to other tissues or organs." d. "Malignant cells reproduce more rapidly than normal cells."

ANS: C The major difference between benign and malignant tumors is that malignant tumors invade adjacent tissues and spread to distant tissues and benign tumors do not metastasize. The other statements are inaccurate. Both types of tumors may cause damage to adjacent tissues. Malignant cells do not reproduce more rapidly than normal cells. Benign tumors do not usually recur.

13. A nurse is caring for a patient who has had a total laryngectomy and radical neck dissection. During the first 24 hours after surgery what is the priority nursing action? a. Monitor the incision for bleeding. b. Maintain adequate IV fluid intake. c. Keep the patient in semi-Fowler's position. d. Teach the patient to suction the tracheostomy.

ANS: C The most important goals after a laryngectomy and radical neck dissection are to maintain the airway and ensure adequate oxygenation. Keeping the patient in a semi-Fowler's position will decrease edema and limit tension on the suture lines to help ensure an open airway. Maintenance of IV fluids and monitoring for bleeding are important, but maintaining an open airway is the priority. During the immediate postoperative period, the patient with a laryngectomy requires frequent suctioning of the tracheostomy tube. The patient may be taught to suction after the tracheostomy is stable, if needed, but not during the immediate postoperative period

10. A young male patient with paraplegia who has a stage 2 sacral pressure injury is being cared for at home by his family. To prevent further tissue damage, what instructions are most important for the nurse to teach the patient and family? a. Change the patient's bedding frequently. b. Apply a hydrocolloid dressing over the injury. c. Change the patient's position every 1 to 2 hours. d. Record the size and appearance of the injury weekly.

ANS: C The most important intervention is to avoid prolonged pressure on bony prominences by frequent repositioning. The other interventions may also be included in family teaching.

6. A patient with right lower-lobe pneumonia has been treated with IV antibiotics for 3 days. Which assessment data obtained by the nurse indicates that the treatment is effective? a. Bronchial breath sounds are heard at the right base. b. The patient coughs up small amounts of green mucus. c. The patient's white blood cell (WBC) count is 6000/µL. d. Increased tactile fremitus is palpable over the right chest.

ANS: C The normal WBC count indicates that the antibiotics have been effective. All the other data suggest that a change in treatment is needed.

19. Which patient who has arrived at the human immunodeficiency virus (HIV) clinic should the nurse assess first? a. Patient whose rapid HIV-antibody test is positive. b. Patient whose latest CD4+ count has dropped to 250/µL. c. Patient who has had 10 liquid stools in the last 24 hours. d. Patient who has nausea from prescribed antiretroviral drugs

ANS: C The nurse should assess the patient for dehydration and hypovolemia. The other patients also will require assessment and possible interventions, but do not require immediate action to prevent complications such as hypovolemia and shock.

18. A 25-yr-old patient comes to the emergency department with severe chest pain and agitation. Which action should the nurse take first? a. Ask about habitual use of stimulant drugs. b. Assess orientation to person, place, and time. c. Check blood pressure, pulse, and respirations. d. Start an IV for administering antipsychotic drugs.

ANS: C The patient has symptoms consistent with the use of cocaine or amphetamines and is at risk for fatal tachydysrhythmias or complications of hypertension such as stroke or myocardial infarction. The nurse also will ask about drug use and assess orientation, but these are not the priority actions. Antipsychotics may be used if the patient develops hallucinations.

26. A patient with cancer is eating very little due to altered taste sensation. Which nursing action would address the cause of the patient problem? a. Add protein powder to foods such as casseroles. b. Tell the patient to eat foods that are high in nutrition. c. Avoid giving the patient foods that are strongly disliked. d. Add spices to enhance the flavor of foods that are served.

ANS: C The patient will eat more if disliked foods are avoided and foods that the patient likes are included instead. Additional spice is not usually an effective way to enhance taste. Adding protein powder does not address the issue of taste. The patient's poor intake is not caused by a lack of information about nutrition.

17. After change-of-shift report, which patient should the nurse assess first? a. A 40-yr-old with a pleural effusion who reports severe stabbing chest pain b. A 72-yr-old with cor pulmonale who has 4+ bilateral edema in his legs and feet c. A 64-yr-old with lung cancer and tracheal deviation after subclavian catheter insertion d. A 28-yr-old with a history of a lung transplant 1 month ago and a fever of 101° F (38.3° C)

ANS: C The patient's history and symptoms suggest possible tension pneumothorax, a medical emergency. The other patients also require assessment as soon as possible, but tension pneumothorax will require immediate treatment to avoid death from inadequate cardiac output or hypoxemia

19. A patient with terminal cancer-related pain and a history of opioid abuse reports breakthrough pain 2 hours before the next dose of sustained-release morphine sulfate (MS Contin) is due. Which action should the nurse take first? a. Use distraction by talking about things the patient enjoys. b. Suggest the use of alternative therapies such as heat or cold. c. Administer the prescribed PRN immediate-acting morphine. d. Consult with the doctor about increasing the MS Contin dose.

ANS: C The patient's pain requires rapid treatment, and the nurse should administer the immediate-acting morphine. Increasing the MS Contin dose and use of alternative therapies and distraction may also be needed, but the initial action should be to use the prescribed analgesic medications.

3. A patient with a systemic bacterial infection feels cold and has a shaking chill. Which assessment finding will the nurse expect next? a. Skin flushing b. Muscle cramps c. Rising body temperature d. Decreasing blood pressure

ANS: C The patient's report of feeling cold and shivering indicate that the hypothalamic set point for temperature has increased and the temperature will be increasing. Because associated peripheral vasoconstriction and sympathetic nervous system stimulation will occur, skin flushing and hypotension are not expected. Muscle cramps are not expected with chills and shivering or with a rising temperature.

16. A patient who had abdominal surgery yesterday is receiving morphine through patient-controlled analgesia (PCA). What action by the nurse is a priority? a. Assessing for nausea b. Auscultating bowel sounds c. Checking the respiratory rate d. Evaluating for sacral redness

ANS: C The patient's respiratory rate is the highest priority of care while using PCA medication because of the possible respiratory depression. The other areas also require assessment but do not reflect immediately life-threatening complications.

6. A patient with alcohol dependence is admitted to the hospital with back pain after a fall. Twenty-four hours after admission, the patient becomes tremulous and anxious. Which action by the nurse is appropriate? a. Insert an IV line and infuse fluids. b. Administer opioids to provide sedation. c. Provide a quiet and well-lit environment. d. Encourage increased liquid and food intake.

ANS: C The patient's symptoms suggest acute alcohol withdrawal, and a quiet and well-lit environment will help decrease agitation, delusions, and hallucinations. There is no indication that the patient is dehydrated or underfed. Benzodiazepines, rather than opioids, are used to prevent withdrawal. IV lines are avoided whenever possible.

23. The nurse on a surgical inpatient unit is caring for several patients. Which patient should the nurse assess first? a. Patient with postoperative pain who received morphine sulfate IV 15 minutes ago. b. Patient who received hydromorphone (Dilaudid) 1 hour ago and is currently asleep c. Patient who was treated for pain just prior to return from the postanesthesia care unit. d. Patient with neuropathic pain who is scheduled to receive a dose of hydrocodone (Lortab) now.

ANS: C The risk for oversedation is greatest in the first 4 hours after transfer from the postanesthesia care unit. Patients should be reassessed 30 minutes after receiving IV opioids for pain. A scheduled oral medication does not need to be administered exactly at the scheduled time. A patient who falls asleep after pain medication can be allowed to rest.

8. The nurse teaches a postmenopausal patient with stage III breast cancer about the expected outcomes of cancer treatment. Which patient statement indicates that the teaching has been effective? a. "After cancer has not recurred for 5 years, it is considered cured." b. "The cancer will be cured if the entire tumor is surgically removed." c. "I will need follow-up examinations for many years after treatment before I can be considered cured." d. "Cancer is never cured, but the tumor can be controlled with surgery, chemotherapy, and radiation."

ANS: C The risk of recurrence varies by the type of cancer. Some cancers are considered cured after a shorter time span or after surgery, but stage III breast cancer will require additional therapies and ongoing follow-up

6. A patient who is receiving sustained-release morphine sulfate (MS Contin) every 12 hours for chronic pain experiences level 9 (0 to 10 scale) breakthrough pain and anxiety. Which action by the nurse is appropriate for treating this change in assessment? a. Administer lorazepam (Ativan) 1 mg orally. b. Give ibuprofen 400 to 800 mg orally. c. Offer immediate-release morphine 30 mg orally. d. Suggest the patient take amitriptyline 10 mg orally.

ANS: C The severe breakthrough pain indicates that the initial therapy should be a rapidly acting opioid, such as the immediate-release morphine. Lorazepam and amitriptyline may be appropriate to use as adjuvant therapy, but they are not likely to block severe breakthrough pain. Use of antianxiety agents for pain control is inappropriate because this patient's anxiety is caused by the pain.

39. The nurse is caring for a patient with left-sided lung cancer. Which finding would be most important for the nurse to report to the health care provider? a. Hematocrit of 32% b. Pain with deep inspiration c. Serum sodium of 126 mEq/L d. Decreased breath sounds on left side

ANS: C The syndrome of inappropriate antidiuretic hormone (and the resulting hyponatremia) is an oncologic metabolic emergency and requires rapid treatment to prevent complications such as seizures and coma. The other findings also require intervention but are common in patients with lung cancer and not immediately life threatening.

5. A patient's 4 x 3-cm leg wound has a 0.4-cm black area in the center of the wound surrounded by yellow-green semiliquid material. Which dressing should the nurse apply to the wound? a. Dry gauze dressing b. Nonadherent dressing c. Hydrocolloid dressing d. Transparent film dressing

ANS: C The wound requires debridement of the necrotic areas and absorption of the yellow-green slough. A hydrocolloid dressing, such as DuoDerm, would accomplish these goals. Transparent film dressings are used for clean wounds or approximated surgical incisions. Dry dressings will not debride the necrotic areas. Nonadherent dressings will not absorb wound drainage or debride the wound.

18. The charge nurse is assigning semiprivate rooms for new admissions. Which patient could safely be assigned as a roommate for a patient who has acute rejection of an organ transplant? a. A patient who has viral pneumonia b. A patient with second-degree burns c. A patient with an anaphylactic reaction d. A patient with graft-versus-host disease

ANS: C There is no increased exposure to infection from a patient who had an anaphylactic reaction. Treatment for a patient with acute rejection includes administration of additional immunosuppressants and the patient should not be exposed to increased risk for infection as would occur from patients with viral pneumonia, graft-versus-host disease, and burns.

4. The nurse has just auscultated coarse crackles bilaterally on a patient with a tracheostomy tube in place. If the patient is unsuccessful in coughing up secretions, what action should the nurse take? a. Encourage increased incentive spirometer use. b. Encourage the patient to increase oral fluid intake. c. Put on sterile gloves and use a sterile catheter to suction. d. Preoxygenate the patient for 3 minutes before suctioning.

ANS: C This patient needs suctioning to secure a patent airway. Sterile gloves and a sterile catheter are used when suctioning a tracheostomy. Preoxygenation for 3 minutes is not necessary; 30 seconds is recommended. Incentive spirometer use opens alveoli and can induce coughing, which can mobilize secretions. However, the patient with a tracheostomy may not be able to use an incentive spirometer. Increasing oral fluid intake would not moisten and help mobilize secretions in a timely manner.

17. The nurse is caring for a patient who has a calcium level of 12.1 mg/dL. Which nursing action should the nurse include on the care plan? a. Maintain the patient on bed rest. b. Auscultate lung sounds every 4 hours. c. Encourage fluid intake up to 4000 mL daily. d. Monitor for Trousseau's and Chvostek's signs.

ANS: C To decrease the risk for renal calculi, the patient should have a fluid intake of 3000 to 4000 mL daily. Ambulation helps decrease the loss of calcium from bone and is encouraged in patients with hypercalcemia. Trousseau's and Chvostek's signs are monitored when there is a possibility of hypocalcemia. There is no indication that the patient needs frequent assessment of lung sounds, although these would be assessed every shift.

15. The nurse is interviewing a patient with contact dermatitis. Which finding indicates a need for patient teaching? a. The patient applies corticosteroid cream to pruritic areas. b. The patient adds oilated oatmeal to the bath water every day. c. The patient takes diphenhydramine at night for persistent itching. d. The patient uses bacitracin-neomycin-polymyxin on minor abrasions.

ANS: D

14. A patient with metastatic colon cancer has severe vomiting after each administration of chemotherapy. Which action by the nurse is appropriate? a. Have the patient eat large meals when nausea is not present. b. Offer dry crackers and carbonated fluids during chemotherapy. c. Administer prescribed antiemetics 1 hour before the treatments. d. Give the patient a glass of a citrus fruit beverage during treatments.

ANS: C Treatment with antiemetics before chemotherapy may help prevent nausea. The patient should eat small, frequent meals. Offering food and beverages during chemotherapy is likely to cause nausea. The acidity of citrus fruits may be further irritating to the stomach.

9. A patient receiving epidural morphine has not voided for over 10 hours. What action should the nurse take first? a. Place an indwelling urinary catheter. b. Monitor for signs of narcotic overdose. c. Ask if the patient feels the need to void. d. Encourage the patient to drink more fluids.

ANS: C Urinary retention is a common side effect of epidural opioids. Assess whether the patient feels the need to void. Because urinary retention is a possible side effect, there is no reason for concern of overdose symptoms. Placing an indwelling catheter requires an order from the health care provider. Usually an in-and-out catheter is performed to empty the bladder if the patient is unable to void because of the risk of infection with an indwelling catheter. Encouraging oral fluids may lead to bladder distention if the patient is unable to void but might be useful if a patient who is able to void has a fluid deficit

29. Which instruction should the nurse include in an exercise teaching plan for a patient with chronic obstructive pulmonary disease (COPD)? a. "Avoid upper body exercises to prevent dyspnea." b. "Stop exercising if you start to feel short of breath." c. "Use the bronchodilator before you start to exercise." d. "Breathe in and out through the mouth while exercising."

ANS: C Use of a bronchodilator before exercise improves airflow for some patients and is recommended. Shortness of breath is normal with exercise and not a reason to stop. Patients should be taught to breathe in through the nose and out through the mouth (using a pursed-lip technique). Upper-body exercise can improve the mechanics of breathing in patients with COPD.

34. Which assessment finding for a patient with a history of asthma indicates that the nurse should take immediate action? a. Pulse oximetry reading of 91% b. Respiratory rate of 26 breaths/min c. Use of accessory muscles in breathing d. Peak expiratory flow rate of 240 L/min

ANS: C Use of accessory muscle indicates that the patient is experiencing respiratory distress, and rapid intervention is needed. The other data indicate the need for ongoing monitoring and assessment but do not suggest that immediate treatment is required

9. A patient scheduled for a total laryngectomy and radical neck dissection for cancer of the larynx asks the nurse, "Will I be able to talk normally after surgery?" What is the most accurate response by the nurse? a. "You will breathe through a permanent opening in your neck, but you will not be b. "You won't be able to talk right after surgery, but you will be able to speak again after the tracheostomy tube is removed." c. "You will have a permanent opening into your neck, and you will need rehabilitation for some type of voice restoration." d. "You won't be able to speak as you used to, but there are artificial voice devices that will give you the ability to speak normally."

ANS: C Voice rehabilitation is planned after a total laryngectomy, and a variety of assistive devices are available to restore communication. Although the ability to communicate orally is changed, it would not be lost. Artificial voice devices do not permit normal-sounding speech. In a total laryngectomy, the vocal cords are removed, so normal speech is impossible.

1. A patient with acute shortness of breath is admitted to the hospital. Which action should the nurse take during the initial assessment of the patient? a. Ask the patient to lie down for complete a full physical assessment. b. Complete the health history and check for allergies before treatment. c. Briefly ask specific questions about this episode of respiratory distress. d. Delay the physical assessment to first complete pulmonary function tests.

ANS: C When a patient has severe respiratory distress, only information pertinent to the current episode is obtained, and a more thorough assessment is deferred until later. Obtaining a comprehensive health history or full physical examination is unnecessary until the acute distress has resolved. Brief questioning and a focused physical assessment should be done rapidly to help determine the cause of the distress and suggest treatment. Checking for allergies is important, but it is not appropriate to complete the entire admission database at this time. The initial respiratory assessment must be completed before any diagnostic tests or interventions can be ordered

28. Which action should the nurse take to prepare a patient with a pleural effusion for a thoracentesis? a. Remind the patient not to eat or drink 6 hours. b. Start a peripheral IV line to administer sedation. c. Position the patient sitting up on the side of the bed. d. Obtain a collection device to hold 3 liters of pleural fluid

ANS: C When the patient is sitting up, fluid accumulates in the pleural space at the lung bases and can more easily be located and removed. The patient does not usually require sedation for the procedure, and there are no restrictions on oral intake because the patient is not sedated or unconscious. Usually only 1000 to 1200 mL of pleural fluid is removed at one time. Rapid removal of a large volume can result in hypotension, hypoxemia, or pulmonary edema.

12. Which medication should the nurse administer for a patient with cancer who describes the pain as "deep, aching and at a level 8 on a 0 to 10 scale"? a. Ketorolac tablets b. Fentanyl (Duragesic) patch c. Hydromorphone (Dilaudid) IV d. Acetaminophen (Tylenol) suppository

ANS: C The patient's pain level indicates that a rapidly acting medication such as an IV opioid is needed. The other medications may also be appropriate to use at other times but will not work as rapidly or as effectively as the IV hydromorphone.

1. An older adult patient who has just arrived in the emergency department has a pulse deficit of 46 beats. The nurse should expect that the patient may require: a. cardiac catheterization. b. emergent cardioversion. c. hourly blood pressure checks. d. electrocardiographic monitoring.

ANS: D

1. To decrease the risk for future hearing loss, which action should the nurse implement with college students at the on-campus health clinic? a. Perform tympanometry. b. Schedule otoscopic examinations. c. Administer influenza immunizations. d. Discuss exposure to amplified music.

ANS: D

16. An older adult patient with a squamous cell carcinoma (SCC) on the lower arm has a Mohs procedure in the dermatology clinic. Which nursing action will be included in the postoperative plan of care? a. Schedule daily appointments for dressing changes. b. Describe the use of topical fluorouracil on the incision. c. Instruct how to use sterile technique to clean the suture line. d. Teach the use of cold packs to reduce bruising and swelling.

ANS: D

16. The nurse has been teaching a patient with type 2 diabetes about managing blood glucose levels and taking glipizide (Glucotrol). Which patient statement indicates a need for additional teaching? a. "If I overeat at a meal, I will still take the usual dose of medication." b. "Other medications besides the Glucotrol may affect my blood sugar." c. "When I am ill, I may have to take insulin to control my blood sugar." d. "My diabetes won't cause complications because I don't need insulin."

ANS: D

19. The charge nurse is observing a new nurse who is caring for a patient with vestibular disease. For what action by the nurse should the charge nurse intervene immediately? a. Facing the patient directly when speaking b. Speaking slowly and distinctly to the patient c. Administering both the Rinne and Weber tests d. Encouraging the patient to ambulate independently

ANS: D

19. The nurse is caring for a patient diagnosed with furunculosis. Which action could the nurse delegate to unlicensed assistive personnel (UAP)? a. Applying antibiotic cream to the groin b. Obtaining cultures from ruptured lesions c. Evaluating the patient's personal hygiene d. Cleaning the skin with antimicrobial soap

ANS: D

19. The registered nurse (RN) is caring for a patient with a hypertensive crisis who is receiving sodium nitroprusside. Which nursing action can the nurse delegate to an experienced licensed practical/vocational nurse (LPN/VN)? a. Evaluate effectiveness of nitroprusside therapy on blood pressure (BP). b. Assess the patient's environment for adverse stimuli that might increase BP. c. Titrate nitroprusside to decrease mean arterial pressure (MAP) to 115 mm Hg. d. Set up the automatic noninvasive BP machine to take readings every 15 minutes.

ANS: D

19. Which teaching point should the nurse plan to include when caring for a patient whose vision is corrected to 20/200? a. How to access audio books? b. How to use a white cane safely? c. Where Braille instruction is available? d. Where to obtain hand-held magnifiers?

ANS: D

2. The nurse is reviewing the 12-lead electrocardiograph (ECG) for a healthy older adult patient who is having an annual physical examination. What finding should be of most concern to the nurse? a. A right bundle branch block. b. The PR interval is 0.21 seconds. c. The QRS duration is 0.13 seconds. d. The heart rate (HR) is 41 beats/min.

ANS: D

2. Which finding by the nurse performing an eye examination indicates that the patient has normal accommodation? a. After covering one eye for 1 minute, the pupil constricts as the cover is removed. b. Shining a light into the patient's eye causes pupil constriction in the opposite eye. c. A blink reaction occurs after touching the patient's pupil with a piece of sterile cotton. d. The pupils constrict while fixating on an object being moved toward the patient's eyes.

ANS: D

21. A patient who has severe refractory psoriasis on the face, neck, and extremities is socially withdrawn because of the appearance of the lesions. Which action should the nurse take first? a. Discuss the possibility of taking part in an online support group. b. Encourage the patient to volunteer to work on community projects. c. Suggest that the patient use cosmetics to cover the psoriatic lesions. d. Ask the patient to describe the impact of psoriasis on quality of life.

ANS: D

22. In reviewing a patient's medical record, the nurse notes that the last eye examination revealed an intraocular pressure of 28 mm Hg. What should the nurse plan to assess? a. Visual acuity b. Pupil reaction c. Color perception d. Peripheral vision

ANS: D

24. A patient with age-related macular degeneration (AMD) has just had photodynamic therapy. Which statement by the patient indicates that the discharge teaching has been effective? a. "I will use drops to keep my pupils dilated until my appointment." b. "I will need to use brighter lights to read for at least the next week." c. "I will not use facial lotions near my eyes during the recovery period." d. "I will cover up with long-sleeved shirts and pants for the next 5 days."

ANS: D

27. Which finding indicates a need to contact the health care provider before the nurse administers metformin (Glucophage)? a. The patient's blood glucose level is 174 mg/dL. b. The patient is scheduled for a chest x-ray in an hour. c. The patient has gained 2 lb (0.9 kg) in the past 24 hours. d. The patient's estimated glomerular filtration rate is 42 mL/min.

ANS: D

3. A 28-yr-old male patient with type 1 diabetes reports how he manages his exercise and glucose control. Which behavior indicates that the nurse should implement additional teaching? a. The patient always carries hard candies when engaging in exercise. b. The patient goes for a vigorous walk when his glucose is 200 mg/dL. c. The patient has a peanut butter sandwich before going for a bicycle ride. d. The patient increases daily exercise when ketones are present in the urine.

ANS: D

3. Which action should the nurse take when giving the first dose of oral labetalol to a patient with hypertension? a. Encourage the use of hard candy to prevent dry mouth. b. Teach the patient that headaches often occur with this drug. c. Instruct the patient to call for help if heart palpitations occur. d. Ask the patient to request assistance before getting out of bed.

ANS: D

32. A patient admitted to the coronary care unit (CCU) with an ST-segment-elevation myocardial infarction (STEMI) is restless and anxious. The blood pressure is 86/40 mm Hg, and heart rate is 132 beats/min. Based on this information, which patient problem is the priority? a. Anxiety b. Acute pain c. Stress management d. Decreased cardiac output

ANS: D

32. The nurse at the eye clinic made a follow-up telephone call to a patient who underwent cataract extraction and intraocular lens implantation the previous day. Which information is the priority to communicate to the health care provider? a. The patient reports that the vision has not improved. b. The patient requests a prescription refill for next week. c. The patient feels uncomfortable wearing an eye patch. d. The patient reports eye pain rated 5 (on a 0 to 10 scale).

ANS: D

38. After receiving change-of-shift report about the following four patients on the cardiac care unit, which patient should the nurse assess first? a. A 39-year-old patient with pericarditis who is complaining of sharp, stabbing chest pain. b. A 56-year-old patient with variant angina who is scheduled to receive nifedipine (Procardia). c. A 65-year-old patient who had a myocardial infarction (MI) 4 days ago and is anxious about today's planned discharge. d. A 59-year-old patient with unstable angina who has just returned after a percutaneous coronary intervention (PCI).

ANS: D

4. Which information from a patient helps the nurse confirm the previous diagnosis of chronic stable angina? a. "The pain wakes me up at night." b. "The pain is level 3 to 5 (0 to 10 scale)." c. "The pain has gotten worse over the last week." d. "The pain goes away after a nitroglycerin tablet."

ANS: D

41. The nurse is working in an urgent care clinic that has standardized treatment protocols for implementation by nursing staff. After reviewing the history, physical assessment, and vital signs for a 60-yr-old patient as shown in the accompanying figure, which action should the nurse take first? a. Check the patient's blood glucose level. b. Take the blood pressure on the left arm. c. Use an irrigating syringe to clean the ear canals. d. Report a vision change to the health care provider.

ANS: D

46. After change-of-shift report, which patient will the nurse assess first? a. A 19-yr-old patient with type 1 diabetes who was admitted with dawn phenomenon b. A 35-yr-old patient with type 1 diabetes whose most recent blood glucose reading was 230 mg/dL c. A 68-yr-old patient with type 2 diabetes who has severe peripheral neuropathy and reports burning foot pain d. A 60-yr-old patient with hyperosmolar hyperglycemic syndrome who has poor skin turgor and dry oral mucosa

ANS: D

5. Which laboratory test result will the nurse review to determine the effects of therapy for a patient being treated for heart failure? a. Troponin b. Homocysteine (Hcy) c. Low-density lipoprotein (LDL) d. B-type natriuretic peptide (BNP)

ANS: D

5. Which patient statement indicates that the nurse's teaching about sublingual nitroglycerin (Nitrostat) has been effective? a. "I can expect nausea as a side effect of nitroglycerin." b. "I should only take nitroglycerin when I have chest pain." c. "Nitroglycerin helps prevent a clot from forming and blocking blood flow to my heart." d. "I will call an ambulance if I have pain after taking 3 nitroglycerin 5 minutes apart."

ANS: D

6. A patient admitted with pneumonia has a total serum calcium level of 13.3 mg/dL. What should the nurse anticipate will be tested next? a. Calcitonin b. Catecholamine c. Thyroid hormone d. Parathyroid hormone

ANS: D

6. Which statement made by a patient with coronary artery disease after the nurse has completed teaching about the therapeutic lifestyle changes (TLC) diet indicates that further teaching is needed? a. "I will switch from whole milk to 1% milk." b. "I like salmon and I will plan to eat it more often." c. "I can have a glass of wine with dinner if I want one." d. "I will miss being able to eat peanut butter sandwiches."

ANS: D

6. While doing the hospital admission assessment for a thin older adult, the nurse observes pulsation of the abdominal aorta in the epigastric area. Which action should the nurse take next? a. Teach the patient about aneurysms. b. Notify the hospital rapid response team. c. Instruct the patient to remain on bed rest. d. Document the finding in the patient chart.

ANS: D

8. A patient who has had chest pain for several hours is admitted with a diagnosis of rule out acute myocardial infarction (AMI). Which laboratory test is most specific for the nurse to monitor in determining whether the patient has had an AMI? a. Myoglobin b. Homocysteine c. C-reactive protein d. Cardiac-specific troponin

ANS: D

8. Which action will be included in the plan of care for a patient who is receiving nicardipine (Cardene) to treat a hypertensive emergency? a. Keep the patient NPO to prevent aspiration caused by nausea and possible vomiting. b. Organize nursing activities so that the patient has 8 hours of undisturbed sleep at night. c. Assist the patient up in the chair for meals to avoid complications associated with immobility. d. Use an automated noninvasive blood pressure machine to obtain frequent measurements.

ANS: D

9. A patient will be evaluated for rhythm disturbances with a Holter monitor. What should the nurse teach the patient to do? a. Connect the recorder to a computer once daily. b. Exercise more than usual while the monitor is in place. c. Remove the electrodes when taking a shower or tub bath. d. Keep a diary of daily activities while the monitor is worn.

ANS: D

9. Which information will the nurse include for a patient considering a cochlear implant? Cochlear implants: a. are not useful for patients with congenital deafness. b. are most helpful as an early intervention for presbycusis. c. improve hearing in patients with conductive hearing loss. d. require extensive training in order to reach the full benefit.

ANS: D

32. A patient newly diagnosed with asthma is being discharged. The nurse anticipates including which topic in the discharge teaching? a. Complications associated with O2 therapy b. Use of long-acting B-adrenergic medications c. Side effects of sustained-release theophylline d. Self-administration of inhaled corticosteroids

ANS: D Inhaled corticosteroids are more effective in improving asthma than any other drug and are indicated for all patients with persistent asthma. The other therapies would not typically be first-line treatments for newly diagnosed asthma.

19. The laboratory technician calls with arterial blood gas (ABG) results on four patients. Which result is most important for the nurse to report immediately to the health care provider? a. pH 7.34, PaO2 82 mm Hg, PaCO2 40 mm Hg, and O2 sat 97% b. pH 7.35, PaO2 85 mm Hg, PaCO2 50 mm Hg, and O2 sat 95% c. pH 7.46, PaO2 90 mm Hg, PaCO2 32 mm Hg, and O2 sat 98% d. pH 7.31, PaO2 91 mm Hg, PaCO2 50 mm Hg, and O2 sat 96%

ANS: D ABGs with a decreased pH and increased PaCO2 indicate uncompensated respiratory acidosis and should be reported to the health care provider. The other values are normal, close to normal, or compensated.

15. The nurse analyzes the results of a patient's arterial blood gases (ABGs). Which finding requires immediate action? a. The bicarbonate level (HCO3 ?2- ) is 31 mEq/L. b. The arterial oxygen saturation (SaO2) is 92%. NURSINGTB.COM Lewis's Medical Surgical Nursing 11th Edition Harding Test BankNU RS IN GT B.CO M c. The partial pressure of CO2 in arterial blood (PaCO2) is 31 mm Hg. d. The partial pressure of oxygen in arterial blood (PaO2) is 62 mm Hg

ANS: D All the values are abnormal, but the low PaO2 indicates that the patient is at the point on the oxyhemoglobin dissociation curve where a small change in the PaO2 will cause a large drop in the O2 saturation and a decrease in tissue oxygenation. The nurse should intervene immediately to improve the patient's oxygenation.

34. A patient with pneumonia has a fever of 101.4° F (38.6° C), a nonproductive cough, and an O2 saturation of 88%. The patient is weak and needs assistance to get out of bed. Which patient problem should the nurse assign as the priority? a. Fatigue b. Hyperthermia c. Impaired mobility d. Impaired gas exchange

ANS: D All these problems are appropriate for the patient, but the patient's O2 saturation indicates that all body tissues are at risk for hypoxia unless the gas exchange is improved.

13. A patient undergoing external radiation has developed a dry desquamation of the skin in the treatment area. The nurse teaches the patient about the management of the skin reaction. Which statement, if made by the patient, indicates the teaching was effective? a. "I can use ice packs to relieve itching." b. "I will scrub the area with warm water." c. "I will expose my skin to a sun lamp each day." d. "I can buy some aloe vera gel to use on my skin

ANS: D Aloe vera gel and cream may be used on the radiated skin area. Ice and sunlamps may injure the skin. Treatment areas should be cleaned gently to avoid further injury

31. The nurse obtains information about a hospitalized patient who is receiving chemotherapy for colorectal cancer. Which information about the patient alerts the nurse to discuss a possible change in cancer therapy with the health care provider? a. Frequent loose stools b. Nausea and vomiting c. Elevated white blood count (WBC) d. Increased carcinoembryonic antigen (CEA

ANS: D An increase in CEA indicates that the chemotherapy is not effective for the patient's cancer and may need to be modified. Gastrointestinal adverse effects are common with chemotherapy. The nurse may need to address these, but they would not necessarily indicate a need for a change in therapy. An elevated WBC may indicate infection but does not reflect the effectiveness of the colorectal cancer therapy.

17. The nurse is caring for a patient who is living with human immunodeficiency virus (HIV) and taking antiretroviral therapy (ART). Which information is most important for the nurse to address when planning care? a. The patient reports feeling "constantly tired." b. The patient reports having no side effects from the medications. c. The patient is unable to explain the effects of atorvastatin (Lipitor). d. The patient reports missing doses of tenofovir AF/emtricitabine (Descovy).

ANS: D Because missing doses of ART can lead to drug resistance, this patient statement indicates the need for interventions such as teaching or changes in the drug scheduling. Fatigue is a common side effect of ART. The nurse should discuss medication actions and side effects with the patient, but this is not as important as addressing the skipped doses of Descovy.

6. Spironolactone (Aldactone), an aldosterone antagonist, is prescribed for a patient. Which statement by the patient indicates that the teaching about this medication has been effective? a. "I will try to drink at least 8 glasses of water every day." b. "I will use a salt substitute to decrease my sodium intake." c. "I will increase my intake of potassium-containing foods." d. "I will drink apple juice instead of orange juice for breakfast."

ANS: D Because spironolactone is a potassium-sparing diuretic, teach patients to choose low-potassium foods (e.g., apple juice) rather than foods that have higher levels of potassium (e.g., citrus fruits). Because the patient is using spironolactone as a diuretic, the nurse would not encourage the patient to increase fluid intake. Teach patients to avoid salt substitutes, which are high in potassium

20. A patient arrives in the ear, nose, and throat clinic with foul-smelling nasal drainage from the right nare, reporting a piece of tissue being "stuck up my nose." Which action should the nurse take first? a. Notify the clinic health care provider. b. Obtain aerobic culture specimens of the drainage. c. Ask the patient about how the cotton got into the nose. d. Have the patient occlude the left nare and blow the nose.

ANS: D Because the highest priority action is to remove the foreign object from the nare, the nurse's first action should be to assist the patient to remove the object. The other actions are also appropriate but should be done after attempting to clear the nose.

23. After the nurse receives report, which patient should the nurse assess first? a. Patient with a history of daily alcohol use who reports insomnia and diaphoresis. b. Patient who is having hallucinations and extreme anxiety after the use of marijuana. c. Patient who has a respiratory rate of 14 after overdosing on oxycodone (OxyContin). d. Patient admitted with cocaine use who has an irregular heart rate of 142 beats/min and BP 186/92 mm Hg.

ANS: D Because the patient with cocaine use has symptoms suggestive of a possible fatal dysrhythmia, this patient should be assessed immediately. The other patients should also be seen as soon as possible, but their clinical manifestations do not suggest that life-threatening complications may be occurring.

16. Which assessment finding for an older patient indicates that the nurse should take immediate action? a. Weak cough effort b. Barrel-shaped chest c. Dry mucous membranes d. Bilateral basilar crackles

ANS: D Crackles in the lower half of the lungs indicate that the patient may have an acute problem such as heart failure. The nurse should immediately accomplish further assessments, such as O2 saturation, and notify the health care provider. A barrel-shaped chest, and a weak cough effort are associated with aging and immediate action is not indicated. An older patient has a less forceful cough and fewer and less functional cilia. Mucous membranes tend to be drier

2. A patient who has a positive test for human immunodeficiency virus (HIV) antibodies is admitted to the hospital with Pneumocystis jiroveci pneumonia (PCP) and a CD4+ count of less than 200 cells/L. Based on diagnostic criteria established by the Centers for Disease Control and Prevention (CDC), which statement by the nurse is correct? a. "The patient meets the criteria for a diagnosis of acute HIV infection." b. "The patient will be diagnosed with asymptomatic chronic HIV infection." c. "The patient will likely develop symptomatic HIV infection within 1 year." d. "The patient has developed acquired immunodeficiency syndrome (AIDS)."

ANS: D Development of PCP meets the diagnostic criteria for AIDS. The other responses indicate earlier stages of HIV infection than is indicated by the PCP infection.

33. A patient is diagnosed with both human immunodeficiency virus (HIV) and active tuberculosis (TB) disease. Which information obtained by the nurse is most important to communicate to the health care provider? a. The Mantoux test had an induration of 7 mm. b. The chest x-ray showed infiltrates in the lower lobes. c. The patient has a cough that is productive of blood-tinged mucus. d. The patient is being treated with antiretrovirals for HIV infection.

ANS: D Drug interactions can occur between the antiretrovirals used to treat HIV infection and the medications used to treat TB. The other data are expected in a patient with HIV and TB

7. A patient who is human immunodeficiency virus (HIV)-infected has a CD4+ cell count of 400/µL. Which factor is most important for the nurse to determine before the initiation of antiretroviral therapy (ART) for this patient? a. CD4+ cell count b. How the patient obtained HIV c. Patient's tolerance for potential medication side effects d. Patient's ability to follow a complex medication regimen

ANS: D Drug resistance develops quickly unless the patient takes ART medications on a strict, regular schedule. In addition, drug resistance endangers both the patient and community. The other information is also important to consider, but patients who are unable to manage and follow a complex drug treatment regimen should not be considered for ART.

32. A patient who has just been admitted with community-acquired pneumococcal pneumonia has a temperature of 101.6° F with a frequent cough and severe pleuritic chest pain. Which prescribed medication should the nurse give first? a. Codeine b. Guaifenesin c. Acetaminophen (Tylenol) d. Piperacillin/tazobactam (Zosyn

ANS: D Early initiation of antibiotic therapy has been shown to reduce mortality. The other medications are also appropriate and should be given as soon as possible, but the priority is to start antibiotic therapy.

20. The nurse develops a teaching plan to help increase activity tolerance at home for an older adult with severe chronic obstructive pulmonary disease (COPD). Which instructions would be appropriate for the nurse to include in the plan of care? a. Walk until pulse rate exceeds 130 beats/min. b. Stop exercising when you feel short of breath. c. Walk 15 to 20 minutes a day at least 3 times/wk. d. Limit exercise to activities of daily living (ADLs)

ANS: D Encourage the patient to walk 15 to 20 minutes a day at least three times a week with gradual increases. Shortness of breath is normal with exercise and not an indication that the patient should stop. Limiting exercise to ADLs will not improve the patient's exercise tolerance. A 70-yr-old patient should have a pulse rate of 120 beats/min or less with exercise (80% of the maximal heart rate of 150 beats/min).

5. A patient who is taking a potassium-wasting diuretic for treatment of hypertension reports generalized weakness. Which action is appropriate for the nurse to take? a. Assess for facial muscle spasms. b. Ask the patient about loose stools. c. Recommend the patient avoid drinking orange juice with meals. d. Suggest that the health care provider order a basic metabolic panel.

ANS: D Generalized weakness is a manifestation of hypokalemia. After the health care provider orders the metabolic panel, the nurse should check the potassium level. Facial muscle spasms might occur with hypocalcemia. Orange juice is high in potassium and would be advisable to drink if the patient is hypokalemic. Loose stools are associated with hyperkalemia.

15. The nurse prepares to administer the following medications to a hospitalized patient with human immunodeficiency (HIV). Which medication is most important to administer at the scheduled time? a. Nystatin tablet b. Oral acyclovir (Zovirax) c. Aerosolized pentamidine (NebuPent) d. Oral tenofovir AF/emtricitabine/bictegravir (Biktarvy)

ANS: D It is important that antiretrovirals be taken at the prescribed time every day to avoid developing drug-resistant HIV. The other medications should also be given as close as possible to the correct time, but they are not as essential to receive at the same time every day.

16. The nurse could delegate care of which patient to a licensed practical/vocational nurse (LPN/VN)? a. The patient who was just admitted after suturing of a full-thickness arm wound. b. The patient who just reported increased tenderness and swelling in a leg wound. c. The patient who requires teaching about home care for an open draining abdominal wound. d. The patient who needs a hydrocolloid dressing change for a stage 3 sacral pressure injury.

ANS: D LPN/VN education and scope of practice include sterile dressing changes for stable patients. Initial wound assessments, patient teaching, and evaluation for possible poor wound healing or infection should be done by the registered nurse (RN).

30. The nurse completes an admission assessment on a patient with asthma. Which information indicates a need for a change in therapy? a. The patient uses albuterol (Ventolin HFA) before aerobic exercise. b. The patient says that the asthma symptoms are worse every spring. c. The patient's heart rate increases slightly after using the albuterol inhaler. d. The patient's only medications are albuterol (Ventolin HFA) and salmeterol

ANS: D Long-acting 2-agonists should be used only in patients who also are using an inhaled corticosteroid for long-term control. Salmeterol should not be used as the first-line therapy for long-term control. Using a bronchodilator before exercise is appropriate. The other information given by the patient requires further assessment by the nurse but is not unusual for a patient with asthma.

46. Which intervention will the nurse include in the plan of care for a patient who is diagnosed with a lung abscess? a. Assist the patient with chest physiotherapy and postural drainage. b. Teach the patient to avoid the use of over-the-counter expectorants. c. Notify the health care provider immediately about any bloody or foul-smelling sputum. d. Teach about the need for prolonged antibiotic therapy after discharge from the hospital.

ANS: D Long-term antibiotic therapy is needed for effective eradication of the infecting organisms in lung abscess. Chest physiotherapy and postural drainage are not recommended for lung abscess because they may lead to spread of the infection. Foul-smelling and bloody sputum are common clinical manifestations in lung abscess. Expectorants may be used because the patient is encouraged to cough.

29. The nurse completes discharge teaching for a patient who has had a lung transplant. Which patient statement indicates that the teaching has been effective? a. "I will make an appointment to see the doctor every year." b. "I will stop taking the prednisone if I experience a dry cough." c. "I will not worry if I feel a little short of breath with exercise." d. "I will call the health care provider right away if I develop a fever."

ANS: D Low-grade fever may indicate infection or acute rejection, so the patient should notify the health care provider immediately if the temperature is elevated. Patients require frequent follow-up visits with the transplant team. Annual health care provider visits would not be sufficient. Home O2 use is not an expectation after lung transplant. Shortness of breath should be reported. Low-grade fever, fatigue, dyspnea, dry cough, and O2 desaturation are signs of rejection. Immunosuppressive therapy, including prednisone, needs to be continued to prevent rejection.

1. The nurse provides discharge instructions after a rhinoplasty. Which statement by the patient indicates that the teaching was successful? a. "My nose will look normal after 24 to 48 hours." b. "I can take 800 mg ibuprofen every 6 hours for pain." c. "I will remove and reapply the nasal packing every day." d. "I will elevate my head for 48 hours to minimize swelling."

ANS: D Maintaining the head in an elevated position will decrease the amount of nasal swelling. Nonsteroidal antiinflammatory drugs, such as ibuprofen, increase the risk for postoperative bleeding and should not be used postoperatively. The patient would not remove or reapply nasal packing, which is usually removed by the surgeon on the day after surgery. Although return to a preinjury appearance is the goal of the surgery, it is not always possible to achieve this result, especially in the first few weeks after surgery.

11. The nurse is completing the medication reconciliation form for a patient admitted with chronic cancer pain. Which medication is of most concern to the nurse? a. Amitriptyline 50 mg at bedtime . Ibuprofen 800 mg 3 times daily c. Oxycodone (OxyContin) 80 mg twice daily d. Meperidine (Demerol) 25 mg every 4 hours

ANS: D Meperidine is contraindicated for chronic pain because it forms a metabolite that is neurotoxic and can cause seizures when used for prolonged periods. The ibuprofen, amitriptyline, and oxycodone are appropriate medications for long-term pain management

24. Which nursing activity can the nurse delegate to unlicensed assistive personnel (UAP) who are working in a family practice clinic? a. Make referrals to community substance use treatment centers. b. Teach patients about the use of nicotine replacement products. c. Obtain patient histories about alcohol, tobacco, and other substance use. d. Administer and score the Alcohol Use Disorders Identification Test (AUDIT

ANS: D No clinical judgment is needed to administer the AUDIT, which is a written questionnaire that is given to patients for self-administration and scored based on patient answers. Making appropriate referrals, patient teaching, and obtaining a patient history all require critical thinking and RN education and scope of practice.

10. The nurse should assess the patient undergoing plasmapheresis for which clinical manifestation? a. Shortness of breath b. High blood pressure c. Transfusion reaction d. Extremity numbness

ANS: D Numbness and tingling may occur as the result of the hypocalcemia caused by the citrate used to prevent coagulation. The other clinical manifestations are not associated with plasmapheresis.

21. The nurse is caring for a patient who has acute pharyngitis caused by Candida albicans. Which action is appropriate for the nurse to include in the plan of care? a. Assess patient for allergies to penicillin antibiotics. b. Teach the patient to sleep in a warm, dry environment. c. Avoid giving the patient warm food or warm liquids to drink. d. Teach patient to "swish and swallow" prescribed oral nystatin.

ANS: D Oral or pharyngeal fungal infections are treated with nystatin solution. The goal of the "swish and swallow" technique is to expose all the oral mucosa to the antifungal agent. Warm liquids may be soothing to a sore throat. The patient should be taught to use a cool mist humidifier. There is no need to assess for penicillin allergy because C. albicans infection is treated with antifungals.

2. A patient who has had good control for chronic pain using a fentanyl (Duragesic) patch reports rapid onset pain at a level 9 (0 to 10 scale) and requests "something for pain that will work quickly." How will the nurse document the type of pain reported by this patient? a. Somatic pain b. Referred pain c. Neuropathic pain d. Breakthrough pain

ANS: D Pain that occurs beyond the chronic pain already being treated by appropriate analgesics is termed breakthrough pain. Neuropathic pain is caused by damage to peripheral nerves or the central nervous system. Somatic pain is localized and arises from bone, joint, muscle, skin, or connective tissue. Referred pain is pain that is localized in uninjured tissue.

5. An alcohol-intoxicated patient with a penetrating wound to the abdomen is undergoing emergency surgery. What will the nurse expect the patient to need during the perioperative period? a. An increased dose of the general anesthetic medication b. Interventions to prevent withdrawal symptoms within 2 hours c. Stimulation hourly to prevent prolonged postoperative sedation d. Frequent monitoring for bleeding and respiratory complications

ANS: D Patients who are intoxicated at the time of surgery are at increased risk for problems with bleeding and respiratory complications such as aspiration. In an intoxicated patient, a lower dose of anesthesia is used because of the synergistic effect of the alcohol. Withdrawal is likely to occur later in the postoperative course because the medications used for anesthesia, sedation, and pain will delay withdrawal symptoms. The patient should be monitored frequently for oversedation but does not need to be stimulated.

38. The nurse is performing tuberculosis (TB) skin tests in a clinic that has many patients who have immigrated to the United States. Which question is important for the nurse to ask before the skin test? a. "Do you take any over-the-counter (OTC) medications?" b. "Do you have any family members with a history of TB?" c. "How long has it been since you moved to the United States?" d. "Did you receive the bacille Calmette-Guérin (BCG) vaccine for TB?"

ANS: D Patients who have received the BCG vaccine will have a positive Mantoux test. Another method for screening (e.g., chest x-ray) will need to be used in determining whether the patient has a TB infection. The other information also may be valuable but is not as pertinent to the decision about doing TB skin testing.

9. What instructions about plasmapheresis should the nurse include in the teaching plan for a patient diagnosed with systemic lupus erythematosus (SLE)? a. Plasmapheresis counteracts recovery of IgG production. b. Plasmapheresis removes eosinophils and basophils from the blood. c. Plasmapheresis decreases the damage to organs from T lymphocytes. d. Plasmapheresis prevents inflammatory mediators from injuring tissues.

ANS: D Plasmapheresis is used in SLE to remove antibodies, antibody-antigen complexes, and inflammatory mediators, such as complement, from the blood. T lymphocytes, foreign antibodies, eosinophils, and basophils do not directly contribute to the tissue damage in SLE. Immunosuppressive therapy is used to prevent recovery of IgG production

35. The nurse supervises unlicensed assistive personnel (UAP) providing care for a patient who has right lower lobe pneumonia. Which action by the UAP requires the nurse to intervene? a. UAP assists the patient to ambulate to the bathroom. b. UAP helps splint the patient's chest during coughing. c. UAP transfers the patient to a bedside chair for meals. d. UAP lowers the head of the patient's bed to 15 degrees

ANS: D Positioning the patient with the head of the bed lowered will decrease ventilation. The other actions are appropriate for a patient with pneumonia.

47. The nurse provides discharge teaching for a patient who has two fractured ribs from an automobile accident. Which patient statement indicates that teaching has been effective? a. "I am going to buy a rib binder to wear during the day." b. "I can take shallow breaths to prevent my chest from hurting." c. "I should plan on taking the pain pills only at bedtime so I can sleep." d. "I will use the incentive spirometer every hour or two during the day."

ANS: D Prevention of the complications of atelectasis and pneumonia is a priority after rib fracture. This can be ensured by deep breathing and coughing. Use of a rib binder, shallow breathing, and taking pain medications only at night are likely to result in atelectasis.

40. An older adult patient who has colorectal cancer is receiving IV fluids at 175 mL/hr in conjunction with the prescribed chemotherapy. Which finding by the nurse is most important to report to the health care provider? a. Patient reports having severe fatigue. b. Patient voids every hour during the day. c. Patient takes only 50% of meals and refuses snacks. d. Patient has crackles up to the midline posterior chest.

ANS: D Rapid fluid infusions may cause heart failure, especially in older patients. The other findings are common in patients who have cancer or are receiving chemother

8. A patient is hospitalized with active tuberculosis (TB). Which assessment finding indicates to the nurse that prescribed airborne precautions are likely to be discontinued? a. Chest x-ray shows no upper lobe infiltrates. b. TB medications have been taken for 6 months. c. Mantoux testing shows an induration of 10 mm. d. Sputum smears for acid-fast bacilli are negative.

ANS: D Repeated negative sputum smears indicate that M. tuberculosis is not present in the sputum, and the patient cannot transmit the bacteria by the airborne route. Chest x-rays are not used to determine whether treatment has been successful. Taking medications for 6 months is necessary, but the multidrug-resistant forms of the disease might not be eradicated after 6 months of therapy. Repeat Mantoux testing would not be done because the result will not change even with effective treatment.

24. A young adult patient with cystic fibrosis (CF) is admitted to the hospital with increased dyspnea. Which intervention should the nurse include in the plan of care? a. Schedule a sweat chloride test. b. Arrange for a hospice nurse visit. c. Place the patient on a low-sodium diet. d. Perform chest physiotherapy every 4 hours.

ANS: D Routine scheduling of airway clearance techniques is an essential intervention for patients with CF. A sweat chloride test is used to diagnose CF, but it does not provide any information about the effectiveness of therapy. There is no indication that the patient is terminally ill. Patients with CF lose excessive sodium in their sweat and require high amounts of dietary sodium.

3. The nurse is caring for a patient who smokes 2 packs/day. Which action by the nurse could help reduce the patient's risk of lung cancer? a. Teach the patient about the seven warning signs of cancer. b. Plan to monitor the patient's carcinoembryonic antigen (CEA) level. c. Teach the patient about annual chest x-rays for lung cancer screening. d. Discuss risks associated with cigarettes during each patient encounter.

ANS: D Teaching about the risks associated with cigarette smoking is recommended at every patient encounter because cigarette smoking is associated with multiple health problems. The other options may detect lung cancer that is already present but do not reduce the risk.

39. A patient is admitted to the emergency department with an open stab wound to the left chest. What action should the nurse take? a. Keep the head of the patient's bed positioned flat. b. Cover the wound tightly with an occlusive dressing. c. Position the patient so that the left chest is dependent. d. Tape a nonporous dressing on three sides over the wound.

ANS: D The dressing taped on three sides will allow air to escape when intrapleural pressure increases during expiration, but it will prevent air from moving into the pleural space during inspiration. Placing the patient on the left side or covering the chest wound with an occlusive dressing will allow trapped air in the pleural space and cause tension pneumothorax. The head of the bed should be elevated to 30 to 45 degrees to facilitate breathing.

7. A patient with rheumatoid arthritis has been taking oral corticosteroids for 2 years. Which nursing action is most likely to detect early signs of infection in this patient? a. Monitor white blood cell counts. b. Check the skin for areas of redness. c. Measure the temperature every 2 hours. d. Ask about feelings of fatigue or malaise

ANS: D The earliest manifestation of an infection may be "just not feeling well." Common clinical manifestations of inflammation and infection are frequently not present when patients receive immunosuppressive medications.

7. A patient with chronic neck pain is seen in the clinic for follow-up. To evaluate whether the pain management is effective, which question is best for the nurse to ask? a. "Has there been a change in pain location?" b. "Can you describe the quality of your pain?" c. "How would you rate your pain on a 0 to 10 scale?" d. "Does pain keep you from activities that you enjoy?"

ANS: D The goal for the treatment of chronic pain usually is to enhance function and quality of life. The other questions are also appropriate to ask, but information about patient function is more useful in evaluating effectiveness.

13. The nurse admits a patient who has a diagnosis of acute asthma. Which statement indicates that the patient may need teaching regarding medication use? a. "I have not had any acute asthma attacks during the past year." b. "I became short of breath an hour before coming to the hospital." c. "I've been taking acetaminophen every 6 hours for chest wall pain." d. "I've been using my albuterol inhaler frequently over the last 4 days."

ANS: D The increased need for a rapid-acting bronchodilator should alert the patient that an acute attack may be imminent and that a change in therapy may be needed. The patient should be taught to contact a health care provider if this occurs. The other data do not indicate any need for additional teaching.

32. The nurse reviews the laboratory results of a patient who is receiving chemotherapy. Which laboratory result is most important to report to the health care provider? a. Hematocrit 30% b. Platelets 95,000/µL c. Hemoglobin 10 g/L d. White blood cells (WBC) 2700/µL

ANS: D The low WBC count places the patient at risk for severe infection and is an indication that the chemotherapy dose may need to be lower or that WBC growth factors such as filgrastim (Neupogen) are needed. Although the other laboratory data indicate decreased levels, they do not indicate any immediate life-threatening adverse effects of the chemotherapy.

13. A patient with human immunodeficiency virus (HIV) infection has developed Cryptosporidium parvum infection. Which outcome would be appropriate for the nurse to include in the plan of care? a. The patient will be free from injury. b. The patient will receive immunizations. c. The patient will have adequate oxygenation. d. The patient will maintain intact perineal skin

ANS: D The major manifestation of C. pravum infection is loose, watery stools, which would increase the risk for perineal skin breakdown. The other outcomes would be appropriate for other complications (e.g., pneumonia, dementia, influenza) associated with HIV infection

10. A patient who was involved in a motor vehicle crash has had a tracheostomy placed to allow for continued mechanical ventilation. How should the nurse interpret the following arterial blood gas results: pH 7.48, PaO2 85 mm Hg, PaCO2 32 mm Hg, and HCO3 25 mEq/L? a. Metabolic acidosis b. Metabolic alkalosis c. Respiratory acidosis d. Respiratory alkalosis

ANS: D The pH indicates that the patient has alkalosis and the low PaCO2 indicates a respiratory cause. The other responses are incorrect based on the pH and the normal HCO3.

36. A patient with a possible pulmonary embolism reports chest pain and difficulty breathing. The nurse finds a heart rate of 142 beats/min, blood pressure of 100/60 mm Hg, and respirations of 42 breaths/min. Which action should the nurse take first? a. Administer anticoagulant drug therapy. b. Notify the patient's health care provider. c. Prepare patient for a spiral computed tomography (CT). d. Elevate the head of the bed to a semi-Fowler's position.

ANS: D The patient has symptoms consistent with a pulmonary embolism (PE). Elevating the head of the bed will improve ventilation and gas exchange. The other actions can be performed after the head is elevated and O2 is started. The health care provider may order a spiral CT to identify PE. Anticoagulants may be ordered after confirmation of the diagnosis of PE.

12. A patient admitted with shortness of breath and chest pain who is a pack-a-day smoker tells the nurse, "I am just not ready to quit smoking yet." Which response by the nurse is appropriate for the patient's stage of change? a. "This would be a really good time to quit." b. "Your smoking is the cause of your chest pain." c. "Are you familiar with nicotine replacement products?" d. "What health problems do you think smoking has caused?"

ANS: D The patient is in the precontemplation stage of change, and the nurse's role is to assist the patient to become motivated to quit. The current Clinical Practice Guidelines indicate that the nurse should ask the patient to identify any negative consequences from smoking. The responses "This would be a really good time to quit" and "Your smoking is the cause of your chest pain" express judgmental feelings by the nurse and are not likely to motivate the patient. Providing information about the various nicotine replacement options would be appropriate for a patient who has expressed a desire to quit smoking.

15. An older adult patient who has been taking alprazolam (Xanax) calls the clinic asking for a refill of the prescription 1 month before the alprazolam should need to be refilled. Which response by the nurse is best? a. "The prescription cannot be refilled for another month. What happened to all of your pills?" b. "Do you have muscle cramps and tremors if you don't take the medication frequently?" c. "I will ask the health care provider to prescribe more pills, but you will not be able to have them until next month." d. "I am concerned that you may be overusing those. I'll make an appointment for you with the health care provider."

ANS: D The patient should be assessed for problems that are causing overuse of alprazolam, such as anxiety or memory loss. The other responses by the nurse will not allow for the needed assessment and possible referral for support services or treatment of drug dependence.

18. A patient receiving head and neck radiation for larynx cancer has ulcerations over the oral mucosa and tongue and thick, ropey saliva. Which instructions should the nurse give to this patient? a. Remove food debris from the teeth and oral mucosa with a stiff toothbrush. b. Use cotton-tipped applicators dipped in hydrogen peroxide to clean the teeth. c. Gargle and rinse the mouth several times a day with an antiseptic mouthwash. d. Rinse the mouth before and after each meal and at bedtime with a saline solution.

ANS: D The patient should rinse the mouth with a saline solution frequently. A soft toothbrush is used for oral care. Hydrogen peroxide may damage tissues. Antiseptic mouthwashes may irritate the oral mucosa and are not recommended

4. A patient admitted to the hospital after an automobile crash is alert and does not appear to be highly intoxicated. The blood alcohol concentration (BAC) is 110 mg/dL (0.11 %). Which action by the nurse is appropriate? a. Restrict oral and intravenous fluids. b. Maintain the patient on NPO status. c. Administer acetaminophen for headache. d. Monitor for hyperreflexia and diaphoresis.

ANS: D The patient's assessment data indicate probable physiologic dependence on alcohol, and the patient is likely to develop acute withdrawal such as anxiety, hyperreflexia, and sweating, which could be life threatening. Acetaminophen is not recommended because it is metabolized by the liver. Alcohol has a dehydrating effect so fluids should not be restricted and there is no indication that the patient should be NPO

23. The emergency department nurse notes tachycardia and absent breath sounds over the right thorax of a patient who has just arrived after an automobile accident. For which intervention will the nurse prepare the patient? a. Emergency pericardiocentesis b. Stabilization of the chest wall c. Bronchodilator administration d. Chest tube connected to suction

ANS: D The patient's history and absent breath sounds suggest a right-sided pneumothorax or hemothorax, which will require treatment with a chest tube and drainage to suction. The other therapies would be appropriate for an acute asthma attack, flail chest, or cardiac tamponade, but the patient's clinical manifestations are not consistent with these problems.

22. After receiving change-of-shift report on 4 patients who are undergoing substance use treatment, which patient will the nurse assess first? a. A patient who has just arrived for alcohol use treatment and states that the last drink was 2 hours ago. b. A patient who is receiving treatment for cocaine addiction and is irritable with a pulse rate of 112 beats/min. c. A patient who is agitated and experiencing nausea, occasional vomiting, and diarrhea while withdrawing from heroin. d. A patient who with new tremors from benzodiazepine withdrawal and whose last benzodiazepine use was 4 days ago

ANS: D The patient's tremors indicate risk for seizures and possible cardiac/respiratory arrest, which can occur with withdrawal from sedative-hypnotics. The greatest risk for these complications occurs 3 to 5 days after stopping the drug. Opioid and stimulant withdrawal are uncomfortable but not life threatening. Symptoms of alcohol withdrawal do not occur until 4 to 6 hours after the last drink.

11. A patient who was admitted with diabetic ketoacidosis has rapid, deep respirations. Which action should the nurse take? a. Give the prescribed PRN lorazepam (Ativan). b. Encourage the patient to take deep slow breaths. c. Start the prescribed PRN oxygen at 2 to 4 L/min. d. Administer the prescribed fluid bolus and insulin

ANS: D The rapid, deep (Kussmaul) respirations indicate a metabolic acidosis and the need for correction of the acidosis with a saline bolus to prevent hypovolemia followed by insulin administration to allow glucose to reenter the cells. Oxygen therapy is not indicated because there is no indication that the increased respiratory rate is related to hypoxemia. The respiratory pattern is compensatory, and the patient will not be able to slow the respiratory rate. Lorazepam administration will slow the respiratory rate and increase the level of acidosis.

2. The nurse prepares a patient who has a left-sided pleural effusion for a thoracentesis. How should the nurse position the patient? a. High-Fowler's position with the left arm extended b. Supine with the head of the bed elevated 30 degrees c. On the right side with the left arm extended above the head d. Sitting upright with the arms supported on an over bed table

ANS: D The upright position with the arms supported increases lung expansion, allows fluid to collect at the lung bases, and expands the intercostal space so that access to the pleural space is easier. The other positions would increase the work of breathing for the patient and make it more difficult for the health care provider performing the thoracentesis.

5. The nurse palpates the posterior chest and notes absent fremitus while the patient says "99". Which action should the nurse take next? a. Palpate the anterior chest and observe for barrel chest. b. Encourage the patient to turn, cough, and deep breathe. c. Review the chest x-ray report for evidence of pneumonia. d. Auscultate anterior and posterior breath sounds bilaterally.

ANS: D To assess for tactile fremitus, the nurse uses the palms of the hands to palpate for vibration while the patient repeats a word or phrase such as "99." After noting absent fremitus, the nurse should then auscultate the lungs to assess for the presence or absence of breath sounds. Absent fremitus may be noted with pneumothorax or atelectasis. The vibration is increased in conditions such as pneumonia, lung tumors, thick bronchial secretions, and pleural effusion. Turning, coughing, and deep breathing are appropriate interventions for atelectasis, but the nurse needs to first assess breath sounds. Fremitus is decreased if the hand is farther from the lung or the lung is hyperinflated (barrel chest). The anterior of the chest is more difficult to palpate for fremitus because of the presence of large muscles and breast tissue

4. Which information will the nurse include in the teaching plan for a patient newly diagnosed with asthma? a. Use the inhaled corticosteroid when shortness of breath occurs. b. Inhale slowly and deeply when using the dry powder inhaler (DPI). c. Hold your breath for 5 seconds after using the bronchodilator inhaler. d. Tremors are an expected side effect of rapidly acting bronchodilators.

ANS: D Tremors are a common side effect of short-acting B2-adrenergic (SABA) medications and not a reason to avoid using the SABA inhaler. Inhaled corticosteroids do not act rapidly to reduce dyspnea. Rapid inhalation is needed when using a DPI. The patient should hold the breath for 10 seconds after using inhalers.

6. The nurse notes that a patient's open abdominal wound widens as it extends deeper into the abdomen. How would the nurse document this characteristic? a. Eschar b. Slough c. Maceration d. Undermining

ANS: D Undermining is evident when a cotton-tipped applicator is placed in the wound and there is a narrower "lip" around the wound, which widens as the wound deepens. Eschar is a crusted cover over a wound. Slough and maceration refer to loosening friable tissue.

8. The nurse should plan to use a wet-to-dry dressing for which patient? a. A patient who has a pressure injury with pink granulation tissue. b. A patient who has a surgical incision with pink, approximated edges. c. A patient who has a full-thickness burn filled with dry, black material. d. A patient who has a wound with purulent drainage and dry brown areas.

ANS: D Wet-to-dry dressings are used when there is minimal eschar to be removed. A full-thickness wound filled with eschar will require interventions such as surgical debridement to remove the necrotic tissue. Wet-to-dry dressings are not needed on approximated surgical incisions. Wet-to-dry dressings are not used on uninfected granulating wounds because of the damage to the granulation tissue.

1. The nurse is developing a health promotion plan for an older adult who worked in the landscaping business for 40 years. The nurse will plan to teach the patient about how to self-assess for which clinical manifestations? (Select all that apply.) a. Vitiligo b. Alopecia c. Intertrigo d. Erythema e. Actinic keratosis

ANS: D, E

1. A patient is admitted to the emergency department for treatment of a possible opioid overdose. Rank the nursing activities in the priority order from first activity to last activity. (Put a comma and a space between each answer choice [A, B, C, D, E]). a. Initiate IV access. b. Take a health history. c. Administer naloxone. d. Obtain a urine toxicology screen. e. Support respirations with a bag-valve-mas

ANS: E, A, C, D, B Maintenance of the airway and respirations is the first priority for patients with possible depressant overdose. Opioid antagonists are given before toxicology testing is done because reversal of the opioid will prevent respiratory arrest. However, this will require IV access. The toxicology report will help guide further treatment for possible multiple substance ingestions. The health history will guide care after the initial emergency treatment phase.


Conjuntos de estudio relacionados

chapter 5 life insurance policy Provisions, options, and riders

View Set

UNCC: ARCH 5202: Architectural History II Final

View Set

[FINANCIAL REPORTING] CFA II LOS 18

View Set

Social Responsibility n Ethics Unit 1: chap 3

View Set